SlideShare a Scribd company logo
1 of 51
Download to read offline
---------------------------------------------------------------------------------------------------------------------------
1
Lý thuyết Graph và ứng dụng
Một trong những kết quả đầu tiên trong lý thuyết đồ thị (graph) xuất hiện
trong bài báo của Leonhard Euler về Bảy cây cầu ở Königsberg, xuất bản
năm 1736. Bài báo này cũng được xem như một trong những kết quả topo đầu
tiên trong hình học, tức là, nó không hề phụ thuộc vào bất cứ độ đo nào. Nó
diễn tả mối liên hệ sâu sắc giữa lý thuyết đồ thị và tôpô học.
Năm 1845, Gustav Kirchhoff đưa ra Định luật Kirchhoff cho mạch
điện để tính điện thế và cường độ dòng điện trong mạch điện.
Năm 1852 Francis Guthrie đưa ra bài toán bốn màu về vấn đề liệu chỉ với
bốn màu có thể tô màu một bản đồ bất kì sao cho không có hai nước nào cùng biên
giới được tô cùng màu. Bài toán này được xem như đã khai sinh ra lý thuyết đồ thị,
và chỉ được giải sau một thế kỉ vào năm 1976 bởi Kenneth Appel và Wolfgang
Haken. Trong khi cố gắng giải quyết bài toán này, các nhà toán học đã phát minh
ra nhiều thuật ngữ và khái niệm nền tảng cho lý thuyết đồ thị.
Năm 1933 nhà toán học Dénes Kőnig đã xuất bản cuốn sách giáo khoa
đầu tiên về đồ thị và phát biểu định lí Kőnig rất nổi tiếng. Năm 1959 xuất bản
CuuDuongThanCong.com https://fb.com/tailieudientucntt
---------------------------------------------------------------------------------------------------------------------------
2
cuốn sách The Theory of Graphs and its Applications được coi như cuốn sách
giáo khoa thứ 2 về vấn đề graph.
Mặc dù Lý thuyết đồ thị đã được khoa học phát triển từ rất lâu nhưng lại
có nhiều ứng dụng hiện đại. Đặc biệt trong khoảng vài mươi năm trở lại đây,
cùng với sự ra đời của máy tính điện tử và sự phát triển nhanh chóng của Tin
học, Lý thuyết đồ thị càng được quan tâm đến nhiều hơn. Đặc biệt là các thuật
toán trên đồ thị đã có nhiều ứng dụng trong nhiều lĩnh vực khác nhau như:
Mạng máy tính, Lý thuyết mã, Tối ưu hoá, Kinh tế học v.v... Chẳng hạn như trả
lời câu hỏi: Hai máy tính trong mạng có thể liên hệ được với nhau hay không ?;
hay vấn đề phân biệt hai hợp chất hoá học có cùng công thức phân tử nhưng lại
khác nhau về công thức cấu tạo cũng được giải quyết nhờ mô hình đồ
thị. Hiện nay, môn học này là một trong những kiến thức cơ sở của bộ môn
khoa học máy tính.
Trong toán học và tin học, lý thuyết đồ thị nghiên cứu các tính chất
của đồ thị. Một cách không chính thức, đồ thị là một tập các đối tượng được gọi
là các đỉnh (hoặc nút) nối với nhau bởi các cạnh (hoặc cung). Cạnh có thể có
hướng hoặc vô hướng. Đồ thị thường được vẽ dưới dạng một tập các điểm (các
đỉnh nối với nhau bằng các đoạn thẳng (các cạnh).
Đồ thị biểu diễn được rất nhiều cấu trúc, nhiều bài toán thực tế có thể
được biểu diễn bằng đồ thị. Ví dụ, cấu trúc liên kết của một website có thể được
biểu diễn bằng một đồ thị có hướng như sau: các đỉnh là các trang web hiện có
tại website, tồn tại một cạnh có hướng nối từ trang A tới trang B khi và chỉ
khi A có chứa 1 liên kết tới B. Do vậy, sự phát triển của các thuật toán xử lý đồ
thị là một trong các mối quan tâm chính của khoa học máy tính.
Cấu trúc đồ thị có thể được mở rộng bằng cách gán trọng số cho mỗi
cạnh. Có thể sử dụng đồ thị có trọng số để biểu diễn nhiều khái niệm khác nhau.
Ví dụ, nếu đồ thị biểu diễn một mạng đường giao thông, các trọng số có thể là
độ dài của mỗi con đường. Một cách khác để mở rộng đồ thị cơ bản là quy định
hướng cho các cạnh của đồ thị (như đối với các trang web, A liên kết tới B,
nhưng B không nhất thiết cũng liên kết tới A). Loại đồ thị này được gọi là đồ thị
có hướng. Một đồ thị có hướng với các cạnh có trọng số được gọi là một lưới.
Trong những năm gần đây, vấn đề đồ thị được đưa vào áp dụng trong
nhiều bài toán tổ hợp thi học sinh giỏi (Quốc gia và quốc tế). Theo xu hướng
của quốc tế, vấn đề tổ hợp nói chung và vấn đề đồ thị nói riêng cần được quan
tâm phát triển mạnh hơn nữa, tạo tiền đề tốt cho học sinh khi tham gia các kì thi
học sinh giỏi. Trong chuyên đề này chúng tôi trình bày lý thuyết từ đầu của
phương pháp đồ thị (các khái niệm và kết quả) cùng một số phương pháp áp
dụng vào giải các bài toán thi học sinh giỏi.
CuuDuongThanCong.com https://fb.com/tailieudientucntt
---------------------------------------------------------------------------------------------------------------------------
3
1. Định nghĩa mở đầu
Định nghĩa 1. Một graph (hay một đồ thị) là tập các đỉnh và các cạnh nối một
số đỉnh với nhau. Kí hiệu  ,G V E với V là tập đỉnh và E V V  là tập
cạnh.
Ví dụ: Có 11 graph khác nhau với tập đỉnh
có 4 phần tử. Biểu diễn của các graph như
hình vẽ bên.
Định nghĩa 2. Hai đỉnh được gọi là kề nhau nếu có 1 cạnh nối 2 đỉnh này.
Kí hiệu cạnh nối 2 đỉnh A, B là  AB , nói chung  AB khác  BA , nếu ta coi 2
cạnh này là 1 thì ta có graph vô hướng, nếu coi chúng khác nhau thì ta có graph
có hướng.
Có thể tồn tại cạnh nối 1 điểm với chính nó, cạnh này gọi là
khuyên.
Có thể tồn tại nhiều cạnh nối 2 điểm phân biệt, các cạnh
này gọi là cạnh kép.
Một graph đơn nếu nó không có khuyên và không có cạnh kép. Các bài toán ta
gặp chủ yếu là graph đơn.
Định nghĩa 3. Một graph đầy đủ với n đỉnh, kí hiệu là nK , là graph mà 2 đỉnh
bất kì đều có cạnh nối giữa chúng, khi đó có tất cả 2
nC cạnh.
Ví dụ : 4K và 5K
Định nghĩa 4. Với U là tập con của tập các đỉnh, kí hiệu  G U là graph con
của G, thu được khi ta xóa tất cả các đỉnh nằm ngoài U, chỉ giữ lại các cạnh mà
cả 2 đầu mút thuộc U.
Các định nghĩa và kết quả dưới đây có giả thiết là graph đơn và vô hướng.
CuuDuongThanCong.com https://fb.com/tailieudientucntt
---------------------------------------------------------------------------------------------------------------------------
4
2. Bậc của đỉnh
Định nghĩa 5. Kí hiệu  d v hoặc  deg v cho bậc của đỉnh v, là số cạnh mà v là
đầu mút. Một khuyên được tính 2 lần cho đỉnh. Một điểm gọi là chẵn nếu nó có
bậc chẵn và được gọi là lẻ nếu nó có bậc lẻ.
Ví dụ: Graph bên có      1 2 34, 6, 1,d v d v d v  
   4 53, 4d v d v  .
Kết quả 1. Trong một graph có nhiều hơn 1 đỉnh luôn có 2 đỉnh có cùng bậc.
Chứng minh
Xét  ,G V E có n đỉnh, khi đó bậc của mỗi đỉnh sẽ là số tự nhiên nhỏ hơn n,
hơn nữa không tồn tại 2 đỉnh mà bậc của chúng tương ứng là 0 và 1n  (Có
đỉnh bậc 1n  có nghĩa nó nối với tất cả các đỉnh khác nên không còn đỉnh bậc
0).
Nếu không tồn tại 2 đỉnh cùng bậc thì bậc của các đỉnh này nhận tất cả các giá
trị 0,1,2,..., 1n  , mâu thuẫn với nhận xét trên. Ta có điều chứng minh.
Kết quả 2. Trong một Graph vô hướng G tùy ý tổng bậc của tất cả các đỉnh gấp
đôi số cạnh của Graph.
Chứng minh: Trong mỗi graph tổng bậc các đỉnh của một graph thì mỗi
cạnh được tính đúng hai lần bởi hai đỉnh của nó. Do đó tổng này gấp đôi số
cạnh của graph
Hệ quả 1: Trong một Graph vô hướng G tùy ý số đỉnh bậc lẻ luôn là một số
chẵn.
Chứng minh: Theo định lý trên thì tổng các bậc của các đỉnh luôn là một số
chẵn do vậy số các đỉnh bậc lẻ luôn là một số chẵn.
Hệ quả 2: Trong một graph vô hướng G có số lẻ đỉnh luôn có một số lẻ các
đỉnh có bậc chẵn.
Chứng minh: Theo hệ quả trên thì số đỉnh bậc lẻ trong graph G là một số
chẵn. Do trong graph G có số lẻ đỉnh, nên số các đỉnh bậc chẵn phải là số lẻ.
CuuDuongThanCong.com https://fb.com/tailieudientucntt
---------------------------------------------------------------------------------------------------------------------------
5
Ví dụ: Trong bữa tiệc có 51 người. Khi đó:
1. Có 1 người quen với chẵn người khác trong bữa tiệc;
2. Có 2 người có cùng số người quen;
3. Nếu mỗi người tính số người quen của mình trong bữa tiệc thì tổng của các số
này là số chẵn.
3. Đường đi, chu trình trong graph, graph liên thông.
Định nghĩa 6. Đường đi trong graph là một dãy các cạnh liên tiếp (hai cạnh liên
tiếp nếu chúng có chung đỉnh). Với đồ thị đơn, nếu đường đi đi qua các đỉnh
1 2, ,..., nv v v theo thứ tự thì ta kí hiệu  1 2, ,..., nv v v , nếu đường đi đi qua các cạnh
1 2, ,.., ne e e theo thứ tự thì ta kí hiệu  1 2, ,.., ne e e .
Ví dụ với graph bên, ta có đường đi
 1 2 6 5 3, , , ,v v v v v , đường đi này có thể kí hiệu là
 1 6 5 4, , ,e e e e .
Độ dài của đường đi là số cạnh của đường đi này.
Ví dụ: Đường đi ở ví dụ trên có độ dài là 4.
Khoảng cách giữa 2 đỉnh a và b là độ dài của đường đi ngắn nhất nối 2 đỉnh
này, kí hiệu  ,d a b . Quy ước  , 0d a a  . Nếu không có đường đi nối a, b thì
quy ước  ,d a b   .
Ví dụ: Ở graph trên ta có    1 3 3 4, 4, , 1d v v d v v  .
Đường kính của graph G là khoảng cách lớn nhất giữa 2 đỉnh của graph, kí hiệu
 d G . Nếu graph có 2 điểm a, b mà  ,d a b   thì quy ước  d G  .
Ví dụ: Graph trên có đường kính là 4.
Định nghĩa 7. Graph gọi là liên thông nếu với 2 đỉnh bất kì luôn tìm được 1
đường đi nối chúng.
Kết quả 3. Với  ,G V E là liên thông thì 1E V  .
Chứng minh
e2
e3
e5
e6
e1
e4
v3
v4
v5
v6v2
v1
CuuDuongThanCong.com https://fb.com/tailieudientucntt
---------------------------------------------------------------------------------------------------------------------------
6
Ta chứng minh quy nạp theo số đỉnh n của đồ thị.
Với 1,2n  ta thấy điều cần chứng minh.
Giả sử bài toán đúng với graph N đỉnh, nghĩa là có ít nhất 1N  cạnh.
Xét  ,G V E có 1N  đỉnh và liên thông, suy ra bậc của mỗi đỉnh đều lớn hơn
0. Ta xét 2 trường hợp
Trường hợp 1: Bậc của mỗi đỉnh đều lớn hơn 1.
Ta có  2 2
A V
E d A N

  , suy ra E N , có điều chứng minh.
Trường hợp 2 : Giả sử có đỉnh A của đồ thị có bậc 1, xét graph con
 'G G A  (bỏ A và cạnh mà A là đầu mút). Dễ thấy 'G là liên thông và có N
đỉnh và 1E  cạnh. Theo giả thiết quy nạp với 'G ta có
 1 1 1 1E N E N       , có điều chứng minh.
Kết quả 4. Các đỉnh có thể phân hoạch thành các tập 1 2, ,..., rV V V mà các graph
con  iG V là liên thông và không có cạnh nào nối cặp điểm ở 2 tập khác nhau.
Chúng được gọi là các thành phần liên thông của G.
Định nghĩa 8. Chu trình của đồ thị là 1 đường đi đóng (điểm đầu và điểm cuối
trùng nhau). Độ dài của chu trình là số cạnh trong chu trình này.
Định nghĩa 9. Đường đi Ole là 1 đường đi qua tất cả các cạnh, mỗi cạnh đúng 1
lần.
Ví dụ trong graph bên, đường đi 1, 2, 3, 4, 5, 6, 7 là 1 đường
đi Ole.
Đường đi này có thể đi qua 1 đỉnh nhiều lần, cần chú ý điều
này để so sánh với đường đi Hamilton ở phần sau.
Định nghĩa 10. Đường đi Ole được gọi là chu trình Ole nếu điểm đầu và điểm
cuối trùng nhau.
Ví dụ graph bên có đường đi 1, 2, 3, 4, 5, 6, 7, 8 là chu trình Ole.
Kết quả 5. Một graph có chu trình Ole thì tất cả các đỉnh là chẵn.
Chứng minh
CuuDuongThanCong.com https://fb.com/tailieudientucntt
---------------------------------------------------------------------------------------------------------------------------
7
Chú ý rằng, tại mỗi đỉnh nếu có cạnh vào thì sẽ có cạnh ra nên bậc luôn là chẵn.
Kết quả 6. Graph liên thông bất kì với tất cả các đỉnh là chẵn có 1 chu trình Ole.
Chứng minh
Bắt đầu từ đỉnh 1v không lặp lại bất kì cạnh nào và chú ý rằng do điều kiện bậc
mỗi đỉnh đều chẵn, đường đi này sẽ kết thúc ở 1v , đây là 1 chu trình. Nếu vẫn
còn cạnh chưa đi qua, xét đỉnh 2v thuộc đường đi trên và là đầu mút của 1 cạnh
chưa đi qua, xét đường đi bắt đầu từ 2v và qua cạnh chưa sử dụng này, đường đi
này cũng sẽ kết thúc ở 2v . Liên hết 2 đường đi này tại đỉnh 2v , nếu chưa được
chu trình thì tiếp tục quá trình. Ta có điều cần chứng minh.
Kết quả 7. Nếu 1 graph có đường đi Ole thì có nhiều nhất 2 đỉnh lẻ.
Chứng minh
Tương tự như kết quả 5.
Kết quả 8. Một graph liên thông có đúng 2 đỉnh lẻ chứa 1 đường đi Ole.
Chứng minh
Nối 1 cạnh mới giữa 2 đỉnh lẻ, khi đó tất cả các đỉnh là chẵn. Theo kết quả trên,
graph này có chu trình Ole. Xét chu trình bắt đầu từ 1 đỉnh lẻ lúc đầu, cạnh mới
thêm đi qua sau cùng. Khi đó nếu xóa cạnh mới thêm này thì chu trình này trở
thành đường đi Ole. Ta có điều chứng minh.
4. Cây
Định nghĩa 11. Một rừng là 1 graph không nhất thiết liên thông và không có
chu trình.
Định nghĩa 12. Một cây là 1 graph liên thông và không có chu trình.
Kết quả 9. Một cây bất kì luôn chứa đỉnh có bậc 1, đỉnh này được gọi là lá.
Chứng minh
Giả sử tất cả các đỉnh đều có bậc không nhỏ hơn 2. Xét đường đi bất kì
  1 2, ,..., 2nv v v n  . Nếu nv nối với 1 trong các đỉnh 1 2,..., nv v  thì ta có 1 chu
trình, mâu thuẫn.
CuuDuongThanCong.com https://fb.com/tailieudientucntt
---------------------------------------------------------------------------------------------------------------------------
8
Nếu không thì nv phải nối với 1 đỉnh khác 1nv  . Tiếp tục quá trình thì graph sẽ
có vô hạn đỉnh, mâu thuẫn.
Hệ quả. Từ kết quả trên suy ra nếu graph có bậc mỗi đỉnh đều lớn hơn hoặc
bằng 2 thì có chu trình.
Kết quả 10. Một cây có thể tạo thành từ 1 graph liên thông bằng cách bỏ đi 1 số
cạnh.
Thật vậy, ta có thể tạo ra bằng cách bỏ đi 1 cạnh trong 1 chu trình, làm với tất
cả các chu trình ta sẽ thu được cây.
Kết quả 11. Một graph liên thông là 1 cây nếu và chỉ nếu nó chứa đúng 1V 
cạnh.
Chứng minh
Giả sử G là liên thông, có n đỉnh và có 1n  cạnh và G có chu trình, chẳng hạn
1 2 1... kA A A A . Nhận thấy nếu loại bỏ cạnh 1 2A A đồ thị mới vẫn liên thông, tuy
nhiên lúc này nó chỉ có 2n  cạnh, mâu thuẫn với 1E V  , do G liên thông.
Ngược lại, do mọi cây đều có đỉnh bậc 1, bỏ đỉnh này khỏi đồ thị, khi đó ta có 1
cây mới mà số đỉnh và số cạnh cùng giảm đi 1. Bằng quy nạp theo số đỉnh ta có
điều chứng minh.
Kết quả 12. Nếu bỏ đi bất kì 1 cạnh của cây thì nó không liên thông.
Chứng minh
Giả sử cây có n đỉnh, theo kết quả trên thì nó có 1n  cạnh. Nếu bỏ đi 1 cạnh thì
nó còn 2n  cạnh. Cũng theo kết quả trên thì nó không còn là cây. Việc bỏ đi 1
cạnh sẽ không làm xuất hiện chu trình nào, suy ra nó không liên thông.
Kết quả 13. Nếu graph G không có chu trình, có n đỉnh và 1n  cạnh thì nó là 1
cây.
Chứng minh
Cần chứng minh G liên thông.
Phân hoạch G thành các thành phần liên thông, giả sử có k thành phần liên
thông. Ta tạo ra 1k  cạnh mới bằng cách nối thành phần liên thông thứ 1 với
thứ 2 (lấy 1 đỉnh ở 1 nối với 1 đỉnh ở 2), làm đến thành phần liên thông thứ k.
CuuDuongThanCong.com https://fb.com/tailieudientucntt
---------------------------------------------------------------------------------------------------------------------------
9
Khi đó ta sẽ được graph mới liên thông và không có chu trình (mỗi thành phần
liên thông không có chu trình và cách nối không tạo ra chu trình), suy ra graph
mới này là 1 cây. Theo kết quả trên, số cạnh của cây này là 1n  , suy ra 1k  ,
nghĩa là graph ban đầu liên thông, ta có điều chứng minh.
Kết quả 14. Giữa 2 đỉnh A, B bất kì trong 1 cây có đúng 1 đường đi.
Chứng minh
Do 1 cây là liên thông nên giữa A, B có ít nhất 1 đường đi.
Giả sử còn 1 đường đi nữa nối A, B. Ta thấy chỉ xảy ra 1 trong 3 trường hợp như
hình dưới đây, khi đó cây này có chu trình, mâu thuẫn.
Vậy giả sử sai, có điều chứng minh.
Nhận xét:
1. Nếu ta nối 2 đỉnh không kề nhau trong cây thì sẽ thu được 1 chu trình. Khi đó
nếu graph G có n đỉnh và ít nhất n cạnh thì nó có chu trình.
2. Kết quả mạnh hơn được đưa ra bởi Erdos: Một graph có n đỉnh và số cạnh ít
nhất là
 1
2
n k
thì tồn tại 1 chu trình có độ dài ít nhất là 1k  .
3. Một graph có n đỉnh và 1n  cạnh thì có ít nhất 2 chu trình.
Thật vậy, trước hết khẳng định graph có 1 chu trình, ta xóa 1 cạnh của chu trình
này thì graph còn lại có n cạnh, khi đó có 1 chu trình khác. Vậy nó có ít nhất 2
chu trình.
4. Nếu tất cả các đỉnh có bậc ít nhất là d thì có 1 đường đi có độ dài ít nhất là
1d  .
Định nghĩa 13. Đường đi Hamilton là đường đi đi qua tất cả các đỉnh, mỗi đỉnh
đúng 1 lần. Một đường đi Hamilton có điểm đầu và cuối trùng nhau được gọi là
chu trình Hamiltonian.
CuuDuongThanCong.com https://fb.com/tailieudientucntt
---------------------------------------------------------------------------------------------------------------------------
10
Ví dụ 1: Graph bên có đường đi 1, 2, 3, 4, 5 là 1 đường đi
Hamilton.
Đường đi 1, 2, 3, 4, 5, 6 là 1 chu trình Hamilton.
Ví dụ 2: Trong các graph dưới đây, graph nào có chu trình Ole, chu trình
Hamilton?
Hình 1: Có cả chu trình Ole và Hamilton.
Hình 2: Có chu trình Hamilton và không có chu trình Ole.
Hình 3: Có chu trình Ole và không có chu trình Hamilton.
Hình 4: Không có cả chu trình Ole và chu trình Hamilton.
5. Tô màu
Số màu (sắc số) của graph là số nhỏ nhất của số màu cần thiết để tô các đỉnh sao
cho không có 2 đỉnh kề nhau được tô cùng màu.
Số màu của graph luôn nhỏ hơn hoặc bằng đỉnh lớn nhất cộng 1.
Hiển nhiên theo định nghĩa trên.
2
5
4
1
3 6
CuuDuongThanCong.com https://fb.com/tailieudientucntt
---------------------------------------------------------------------------------------------------------------------------
11
Một số bài toán áp dụng
Bài 1. Cho số nguyên dương 2n  và n thành phố, có 1 đường đi trực tiếp nối 2
thành phố bất kì, đường đi này thuộc sở hữu của 1 hãng hàng không. Biết rằng
không có hãng hàng không nào có quá 2n  đường đi. Chứng minh tồn tại 3
thành phố mà đường đi nối chúng đôi một thuộc 3 hãng hàng không khác nhau.
Giải
Đưa bài toán về ngôn ngữ đồ thị như sau: Cho đồ thị đầy đủ có n đỉnh, tô màu
mỗi cạnh bởi 1 màu sao cho không có màu nào được dùng quá 2n  lần. Chứng
minh tồn tại một tam giác mà 3 cạnh được tô 3 màu khác nhau.
Chứng minh bài toán bằng nguyên lí cựu hạn
Giả sử phản chứng, nghĩa là không tồn tại tam giác mà 3 cạnh được tô 3 màu
khác nhau.
Giả sử ta sử dụng các màu 1, 2, 3, …, k.
Với mỗi i, kí hiệu iC là tập tất cả các đỉnh mà với 2 đỉnh bất kì trong đó có
đường đi được tô cùng màu i (Đường đi trong đồ thị là dãy các cạnh liên tiếp).
Đặt
1,...,
max i
i k
C C

 , giả sử các đỉnh trong C được tô màu 1.
Do điều kiện mỗi màu được dùng không quá 2n  lần nên tồn tại đỉnh v không
thuộc C.
Xét 2 đỉnh bất kì u, w trong C, khi đó cả 2 cạnh vu, vw được tô màu khác màu 1
(vì nếu có cạnh được tô màu 1 thì v thuộc C).
Nếu 2 cạnh này được tô 2 màu khác nhau thì tam giác uvw thỏa mãn điều kiện,
mâu thẫu giả sử.
Nếu 2 cạnh này được tô cùng màu, giả sử màu 2. Khi đó tất cả các cạnh nối v và
đỉnh trong C cùng được tô màu 2 (do u, w được lấy bất kì).
Suy ra tập  C v chứa các đỉnh mà với 2 đỉnh bất kì trong đó có đường đi
được tô cùng màu 2, tập này có nhiều phần tử hơn tập C, mâu thuẫn cách chọn
C.
Vậy điều giả sử là sai, ta có điều phải chứng minh.
CuuDuongThanCong.com https://fb.com/tailieudientucntt
---------------------------------------------------------------------------------------------------------------------------
12
Bài 2 (IMO Shortlist 1990). Có 10 thành phố và 2 hãng hàng không cung cấp
tất cả các đường bay trực tiếp từ 2 thành phố bất kì, giữa 2 thành phố có đúng 1
đường bay. Một hãng hàng không chỉ tổ chức được chuyến bay trên đường bay
mà mình quản lí. Chứng minh có 1 hãng hàng không có thể tổ chức 2 chuyến du
lịch, mỗi chuyến là đường đi vòng tròn qua lẻ thành phố và không có thành phố
chung của 2 chuyến du lịch này.
Giải
Đưa bài toán về ngôn ngữ graph: Xét graph đầy đủ 10K . Mỗi cạnh của graph
được tô bởi màu xanh hoặc đỏ. Chứng minh tồn tại 2 chu trình rời nhau, cùng
màu và có độ dài lẻ.
Trước hết cần có 2 bổ đề:
Bổ đề 1. Mỗi cạnh của graph 6K được tô bởi 1 trong 2 màu, khi đó tồn tại tam
giác đơn sắc.
Chứng minh
Dễ chứng minh bằng nguyên lí Dirichle.
Bổ đề 2. Mỗi cạnh của graph 5K được tô bởi 1 trong 2 màu, giả sử không tồn tại
tam giác đơn sắc. Khi đó tồn tại 2 chu trình đơn sắc có độ dài 5.
Chứng minh
Xét các đỉnh của 5K là 1 2 3 4 5, , , ,v v v v v và các cạnh 1 2 1 3 1 4 1 5, , ,v v v v v v v v .
Nếu cả 4 cạnh này cùng màu thì sẽ tồn tại tam giác đơn sắc, mâu thuẫn giả thiết.
Nếu có 3 cạnh, giả sử là 1 2 1 3 1 4, ,v v v v v v cùng màu, giả sử là màu đỏ.
Xét 3 cạnh 2 3 3 4 4 2, ,v v v v v v , nếu có cạnh màu đỏ thì có tam giác đỏ, nếu không thì
có tam giác xanh, mâu thuẫn giả sử.
Vậy mỗi đỉnh là đầu mút của đúng 2 cạnh xanh và 2 cạnh đỏ.
Xét graph sinh ra từ graph trên mà chỉ giữ lại cạnh được tô đỏ, graph này có 5
đỉnh, mỗi đỉnh đều có bậc 2, khi đó nó là hợp của các chu trình rời nhau, do chỉ
có 5 đỉnh nên nó chỉ có 1 chu trình, chu trình này có độ dài là 5. Tương tự với
graph khi ta chỉ giữ lại các cạnh được tô xanh.
CuuDuongThanCong.com https://fb.com/tailieudientucntt
---------------------------------------------------------------------------------------------------------------------------
13
Ta có điều phải chứng minh.
Quay lại bài toán
Xét 10 đỉnh của graph là 1 2 10, ,...,v v v .
Theo bổ đề 1, tồn tại tam giác đơn sắc, giả sử là 1 2 3v v v .
Tiếp tục bổ đề 1 với graph con  10 1 2 3, ,K v v v , ta có tam giác đơn sắc, giả sử là
4 5 6v v v .
Nếu 2 tam giác này cùng màu ta có điều phải chứng minh.
Ngược lại, giả sử 1 2 3v v v màu xanh và 4 5 6v v v màu đỏ.
Xét 9 cạnh ,1 3,4 6i jv v i j    , theo nguyên lí Dirichle có 5 cạnh trong số
chúng cùng màu, giả sử là màu xanh. Khi đó tồn tại 0 0,4 6j j  sao cho 2
trong 3 cạnh 0 0 01 2 3, ,j j jv v v v v v cùng màu xanh, ta có 1 tam giác xanh và 1 tam
giác đỏ chung nhau đúng 1 đỉnh 0jv , giả sử ta có 1 2 3v v v màu đỏ và 3 4 5v v v màu
xanh.
Xét graph con  10 1 5,...,K v v có 5 đỉnh.
Nếu graph này có tam giác đơn sắc thì có điều chứng minh.
Ngược lại, theo bổ đề 2 thì tồn tại 1 chu trình đơn sắc có độ dài 5.
Chu trình này cùng với 1 trong 2 tam giác ở trên thỏa mãn điều kiện bài toán.
Vậy bài toán được chứng minh.
Bài 3. Một viện hàn lâm có 999 viện sĩ, mỗi đề tài khoa học có đúng 3 viện sĩ
nghiên cứu và 2 viện sĩ bất kì có chung nhau đúng 1 đề tài cùng nghiên cứu.
Chứng minh trong số các đề tài nghiên cứu, tồn tại 250 đề tài mà mỗi viện sĩ chỉ
nghiên cứu tối đa 1 đề tài trong số 250 đề tài đó.
Giải
Giả sử các đề tài là 1, 2, …, n tương ứng với các màu 1, 2, …, n.
Xây dựng đồ thị G có 999 đỉnh, 2 đỉnh nối với nhau nếu 2 viện sĩ tương ứng
nghiên cứu cùng 1 đề tài, đồng thời tô màu cạnh này tương ứng với đề tài đó.
CuuDuongThanCong.com https://fb.com/tailieudientucntt
---------------------------------------------------------------------------------------------------------------------------
14
Khi đó ta được G là đầy đủ, hơn nữa với mỗi màu có đúng 3 cạnh được tô màu
này và 3 cạnh này tạo thành 1 tam giác đơn sắc.
Cần chứng minh tồn tại 250 tam giác đơn sắc đôi một rời nhau.
Gọi k là số lớn nhất các tam giác đơn sắc đôi một không có điểm chung.
Nếu 250k  ta chọn k đề tài ứng với k màu của k tam giác trên thì bài toán
được chứng minh.
Nếu 250k  , trong  999 3k đỉnh còn lại không thể tìm được 1 tam giác đơn
sắc nào nữa (nếu có thì k không lớn nhất).
Gọi S là tập chứa  999 3k đỉnh này, T là tập các đỉnh không thuộc S.
Suy ra với 2 điểm X, Y bất kì trong S, tồn tại duy nhất điểm Z trong T sao cho
XYZ đơn sắc.
Ta có 999 3.249 252, 249.3 747.S T    
Có tất cả 2
S
C cặp điểm trong S.
Theo nguyên lí Dirichle, tồn tại
2 2
1 1
999
3
S S
C C
Tk
 
   
     
    
  
cặp điểm trong S mà
mỗi cặp điểm này đều kề với 1 đỉnh của tam giác ABC trong T.
Dễ dàng chứng minh được
2
1 ,252 999
999 2
3
S
C S
S
T
 
 
    
 
  
nên trong số các
cặp điểm kề với các đỉnh của tam giác ABC sẽ có 2 cặp kề với 2 đỉnh khác nhau
(vì nếu không thì sẽ có 2 cặp điểm chung nhau 1 đỉnh cùng nối với 1 đỉnh của
tam giác, suy ra có 4 đỉnh tương ứng với 4 người có cùng đề tài, mâu thuẫn).
Suy ra tồn tại 2 cặp điểm  1 1,X Y và  2 2,X Y trong S tương ứng kề với 2 điểm
,A B là 2 đỉnh của tam giác ABC đơn sắc trong T.
CuuDuongThanCong.com https://fb.com/tailieudientucntt
---------------------------------------------------------------------------------------------------------------------------
15
Khi đó nếu bỏ đi tam giác ABC và thay bởi 2 tam giác đơn sắc 1 1X Y A và
2 2X Y B thì số tam giác đơn sắc nhiều hơn k, trái với tính lớn nhất của k. Vậy
250k  không thỏa mãn.
Ta có điều chứng minh.
Bài 4. Cho 1 nhóm người mà mỗi người quen không quá m người khác trong
nhóm. Chứng minh có thể chia nhóm người này thành 1m  nhóm sao cho 2
người trong cùng 1 nhóm thì không quen nhau.
Giải
Xây dựng graph G có số đỉnh ứng với số người trong nhóm, 2 đỉnh được nối với
nhau nếu 2 người tương ứng quen nhau.
Tô màu 1m  đỉnh bất kì bởi 1m  màu khác nhau. Thực hiện tô màu các đỉnh
còn lại sao cho 2 đỉnh kề nhau được tô màu khác nhau. Do mỗi đỉnh kề với
không quá m đỉnh nên việc tô màu thực hiện được.
Khi đó tạo được 1m  nhóm mà trong mỗi nhóm các đỉnh được tô cùng màu,
hay chúng không kề với nhau. Ta có điều chứng minh.
Bài 5 (IMO 1992, P3). Cho G là graph đầy đủ với 9 đỉnh. Mỗi cạnh được tô bởi
1 trong 2 màu xanh, đỏ hoặc không tô màu. Tìm n nhỏ nhất sao cho nếu tô màu
n cạnh bất kì thì tồn tại ít nhất 1 tam giác đơn sắc (tam giác có 3 cạnh không
được tô không được gọi là đơn sắc).
Giải
Số cạnh của graph là 2
9 9.4 36C   .
+) Chứng minh tồn tại cách tô 32 cạnh mà không có tam giác đơn sắc.
Xét đỉnh V và tô 4 cạnh xanh nối tới các đỉnh 1 2 3 4, , ,B B B B và 4 cạnh đỏ nối tới
các đỉnh 1 2 3 4, , ,R R R R . Các cạnh 1i iB B tô đỏ, 1i iR R tô xanh với mọi 1,2,3,4i  .
Các cạnh i jB R tô xanh nếu i j lẻ, cạnh i jB R tô đỏ nếu i j chẵn.
Khi đó ta tô màu 32 cạnh và không có tam giác đơn sắc. Suy ra 33n  .
+) Chứng minh 33n  thỏa mãn.
Giả sử phản chứng, có thể tô 33 cạnh mà không có tam giác đơn sắc.
CuuDuongThanCong.com https://fb.com/tailieudientucntt
---------------------------------------------------------------------------------------------------------------------------
16
Xét đỉnh X bất kì, nếu X là đầu mút của 5 cạnh được tô xanh, giả sử được nối
với các đỉnh 1 2 3 4 5, , , ,A A A A A . Nếu có cạnh nối 2 trong 5 đỉnh trên được tô màu
thì đó là màu đỏ.
Chú ý: Nếu có nhiều hơn
2
4
k
cạnh của graph k đỉnh được tô đỏ thì tồn tại tam
giác đơn sắc đỏ.
Suy ra trong các cạnh được tạo từ 5 đỉnh trên thì có nhiều nhất
2
5
6
4
 
 
 
cạnh
được tô đỏ hay có ít nhất 4 cạnh không được tô màu, suy ra có nhiều nhất 32
cạnh được tô màu, mâu thuẫn.
Suy ra X là đầu mút của nhiều nhất 4 cạnh được tô xanh, tương tự suy ra X là
đầu mút của nhiều nhất 4 cạnh được tô đỏ.
Giả sử X là đầu mút của đúng 8 cạnh được tô màu thì có đúng 4 cạnh được tô
xanh nối với 1 2 3 4, , ,B B B B và đúng 4 cạnh được tô đỏ nối với 1 2 3 4, , ,R R R R .
Trong 6 cạnh i jB B nếu được tô màu thì đc tô không nhiều hơn
4
4
4
4
 cạnh đỏ,
nghĩa là có ít nhất 2 cạnh không được tô màu. Tương tự với các cạnh i jR R có ít
nhất 2 cạnh không được tô màu. Suy ra có ít nhất 4 cạnh của G không được tô
màu, nghĩa là tô màu nhiều nhất 32 cạnh, mâu thuẫn.
Suy ra, nếu chỉ xét những cạnh được tô màu thì không có đỉnh nào có bậc 8, suy
ra bậc mỗi đỉnh nhiều nhất là 7, suy ra tổng bậc của 9 đỉnh nhiều nhất là
9.7 63 2.33 69   , mâu thuẫn.
Vậy giả sử sai, ta có điều chứng minh.
Bài 6. Cho graph G đơn, vô hướng, hữu hạn. Chứng minh có thể tô tất cả các
đỉnh của G, mỗi đỉnh bởi 1 trong 2 màu sao cho 2 đỉnh kề nhau được tô màu
khác nhau khi và chỉ khi mọi chu trình đơn trong G đều có độ dài chẵn.
Giải
1) Giả sử có thể tô tất cả các đỉnh của G, mỗi đỉnh bởi 1 trong 2 màu xanh, đỏ
sao cho 2 đỉnh kề nhau được tô màu khác nhau.
Nếu G không có chu trình ta có điều chứng minh.
CuuDuongThanCong.com https://fb.com/tailieudientucntt
---------------------------------------------------------------------------------------------------------------------------
17
Nếu G có chu trình, xét chu trình bất kì  1 2 1... , 1,2,...,n n i iv v v v v v i n   , giả sử
1v được tô xanh, suy ra tất cả các đỉnh iv với i lẻ được tô xanh và iv với i chẵn
được tô đỏ. Do 1v và nv kề nhau nên nv được tô đỏ, hay n chẵn. Ta có điều
chứng minh.
2) Giả sử mọi chu trình đơn trong G đều có độ dài chẵn.
Ta chỉ ra cách tô màu thỏa mãn điều kiện
Chia G thành các thành phần liên thông rời nhau, và tô màu từng thành phần
liên thông.
Xét thành phần liên thông P có tập đỉnh là V.
Chọn đỉnh u trong V và tô xanh cho u. Xét đỉnh v khác u bất kì trong V. Do mọi
chu trình đều có độ dài chẵn nên mọi đường đi từ v đến u có độ dài cùng tính
chẵn lẻ. Nếu độ dài các đường đi là chẵn thì tô v bởi màu xanh, ngược lại tô v
bởi màu đỏ. Ta được cách tô thỏa mãn điều kiện.
Ta có điều chứng minh.
Bài 7 (TOT 1986). Có 20 đội bóng tham gia giải thi đấu. Trong ngày đầu tiên
tất cả các đội thi đấu với 1 đội khác. Ngày thứ 2 tất cả các đội thi đấu với 1 đội
khác ngày hôm trước. Chứng minh sau ngày thi đấu thứ 2 có thể chọn được 10
đội sao cho không có 2 đội trong số chúng đã thi đấu với nhau.
Giải
Xây dựng đồ thị 20 đỉnh ứng với 20 đội bóng, cạnh nối hai đỉnh khi 2 đội thi
đấu với nhau. Tô màu đỏ cho cạnh ứng với trận đấu của ngày thứ nhất và màu
xanh ứng với trận đấu của ngày thứ 2.
Khi đó tại mỗi đỉnh đều có bậc 2 và là đầu mút của 1 canh đỏ, 1 cạnh xanh, suy
ra các thành phần của đồ thị là các chu trình chẵn rời nhau. Trong mỗi chu trình
chọn được 1 nửa đỉnh trong số chúng không kề nhau. Ta có điều phải chứng
minh.
Bài 8.
a. Xét graph  ,G V E , tập D chứa các đỉnh của G được gọi là “trội” nếu mỗi
đỉnh của G hoặc thuộc D, hoặc có 1 đỉnh kề thuộc D.
CuuDuongThanCong.com https://fb.com/tailieudientucntt
---------------------------------------------------------------------------------------------------------------------------
18
Chứng minh nếu G không có điểm cô lập thì tồn tại 1 tập “trội” có không quá
2
V
đỉnh.
b. (Nga 2001) Trong một bữa tiệc có  *
2 1n n  người. Biết rằng nếu chọn
ra n người bất kì thì tồn tại 1 người trong số 1n  người còn lại quen với tất cả
n người này. Chứng minh tồn tại 1 người quen với tất cả những người còn lại
trong bữa tiệc.
Chứng minh bổ đề
Dễ thấy nếu G không có điểm cô lập thì có thể phân hoạch tập đỉnh V thành 2
tập 1 2,V V sao cho mỗi đỉnh trong 1V có đỉnh kề trong 2V và ngược lại.
Hai tập này đều là các tập trội nên có 1 tập có không quá
2
V
phần tử, ta có điều
chứng minh.
Dễ thấy nếu tập D là trội thì các tập hợp thành của D và 1 số đỉnh không thuộc
D cũng là tập trội. Khi đó ta có thể kết luận rằng tồn tại tập trội có đúng
2
V
phần tử.
Chứng minh bài toán
Phát biểu lại bằng ngôn ngữ graph: Cho graph có 2 1n  đỉnh ứng với 2 1n 
người, nếu cặp 2 người không quen nhau thì 2 đỉnh tương ứng sẽ nối với nhau.
Giả sử phản chứng, nghĩa là graph không có đỉnh cô lập, theo bổ đề trên, tồn tại
tập D là tập trội, tập này có n phần tử.
Do điều kiện bài toán nên tồn tại 1 đỉnh trong số 1n  đỉnh còn lại không nối
với bất kì đỉnh nào trong D, mâu thẫu với định nghĩa tập trội.
Vậy giả sử sai, ta có điều chứng minh.
Bài 9 (China TST 2012). Cho n và k là các số nguyên dương sao cho 2n  và
2
n
k n  . Xét graph G có n đỉnh sao cho G không chứa chu trình độ dài 1k 
nhưng nếu thêm bất kì 1 cạnh mới trong G thì sẽ có chu trình độ dài 1k  . Gọi
CuuDuongThanCong.com https://fb.com/tailieudientucntt
---------------------------------------------------------------------------------------------------------------------------
19
đỉnh trong G là tốt nếu nó liên thông đến mọi đỉnh trong G. Hỏi G có ít nhất bao
nhiêu đỉnh tốt.
Giải
Chứng minh nếu 1 2 1k n k    thì có ít nhất 2k n đỉnh tốt.
Do 1k n  nên có 2 đỉnh ,u v G không kề nhau, xét tập  ,T u v , S là tập
các đỉnh mà khi nối u, v thì cùng với các đỉnh này lập thành chu trình độ dài
1k  , tập   ,U G T S .
Chú ý rằng 1 2S k k n U     .
Giả sử có bộ 3 tập rời nhau T, S, U sao cho  ,G T S U T S N S     và
2S k n U   .
Nếu U là tập rỗng thì S chứa ít nhất 2k n đỉnh tốt, có điều cần chứng minh.
Ngược lại, chọn 1 đỉnh tùy ý u U .
Nếu  S N u thì theo trên xét với    '  , ' U U u T T u  và 'S S thì
' 2 'S k n U   . Cố định đỉnh v thuộc S sao cho u, v không kề nhau.
Theo trên, tồn tại 1k  đỉnh cùng với u, v tạo thành chu trình độ dài 1k  lấy
trong cá tập  0 0 ,U U u T T  và  0 S S v . Nhưng G không có chu trình
độ dài 1k  nên 0T S k  .
Suy ra 0 0 0 0 01k U T S U k S S       
Suy ra  0 0 01 2 1S U S k n U U        
Do 0 1U U  nên 0S khác rỗng.
Xét các tập       0 0 0'  , ' 'U U u U T T u U S S      và 0'S S .
Ta có điều chứng minh.
Bài 10 (IMO Shortlist 2004, C8). Cho graph hữu hạn G, kí hiệu  f G là số tam
giác trong G và  g G là số đồ thị con 4K của G. Xác định giá trị nhỏ nhất của
c sao cho    
3 4
,g G cf G G  .
CuuDuongThanCong.com https://fb.com/tailieudientucntt
---------------------------------------------------------------------------------------------------------------------------
20
Giải
Trước hết có bổ đề: Một graph có f tam giác và h cạnh thì 2 32
9
f h .
Chứng minh
Đặt id là bậc của đỉnh i. Do mỗi đỉnh là đỉnh của nhiều nhất h tam giác, nhưng
tính theo bậc thì nó là đỉnh của nhiều nhất
2
2
id
tam giác.
Suy ra
2
3
3 . .2 2
2 2 2
i
i
d h h
f h d h h     , có điều chứng minh.
Quay lại bài toán.
Graph G có f tam giác và g đồ thị con 4K . Xét đỉnh i là đỉnh của if tam giác và
ig đồ thị con 4K .
Xét graph con gồm các đỉnh kề với đỉnh này, graph này có if cạnh và ig tam
giác, theo bổ đề ta có 2 32
9
i ig f .
Tuy nhiên ta có ig f , suy ra
1 1 1
3 3 3
32 2 2
4 . .3
9 9 9
i i i
f f
g g f f f f
     
        
     
  
Suy ra 3 3 4 3 42 3
64 .27 2.27
9 32
f
g f f g f
 
    
 
Vậy giá trị nhỏ nhất của c là
3
32
.
Bài 11. (Croatian TST 2011) Có n người trong 1 bữa tiệc, một số người trong
đó quen biết nhau. Chọn 4 người bất kì thì có 3 người hoặc cùng quen với người
còn lại hoặc cùng không quen với người còn lại. Chứng minh có thể chia những
người này thành 2 nhóm rời nhau A và B sao cho những người trong A đôi một
quen nhau và những người trong B đôi một không quen nhau, (mỗi nhóm A, B
có thể không có người).
CuuDuongThanCong.com https://fb.com/tailieudientucntt
---------------------------------------------------------------------------------------------------------------------------
21
Giải
Phát biểu lại theo ngôn ngữ graph: Xét graph G có n đỉnh ứng với n người, nối 2
đỉnh nếu 2 người tương ứng quen nhau.
Điều kiện bài toán tương đương: Với 4 đỉnh bất kì thì có 3 đỉnh cùng kề với
đỉnh còn lại hoặc cùng không kề với đỉnh còn lại.
Chú ý: Một graph luôn tồn tại graph con đầy đủ. Một graph con đầy đủ được gọi
là lớn nhất nếu nó chứa nhiều đỉnh nhất.
Dễ thấy nếu A là graph con đầy đủ lớn nhất của G thì nó thỏa mãn điều kiện, ta
cần chứng minh B là tập G A .
Nếu G là đầy đủ hoặc tất cả các đỉnh của G là cô lập thì có điều cần chứng
minh.
Ngược lại, ta có 2 1A n   .
Giả sử có 2 đỉnh 1 2,v v trong G A là 2 đỉnh kề nhau.
Do A là tập con đầy đủ lớn nhất của G nên tồn tại đỉnh 1u trong A không kề với
1v .
(Nếu không thì 1v kề với tất cả các đỉnh trong A, khi đó
 1A v là graph đầy đủ lớn hơn A).
Nếu 1u không kề với 2v , xét đỉnh u bất kì trong A.
Với 4 đỉnh 1 2 1, , ,u v v u : u kề với 1u , 1v kề với 2v và 1u
không kề 2v , 1u không kề 1v , suy ra 1 1 2, ,v u v cùng kề với
u.
Suy ra graph     1 1 2,A u v v  là graph đầy đủ lớn hơn A.
Vậy 1u kề với 2v , lập luận tương tự như trên thì u kề với 2v .
Khi đó graph  2A v là graph đầy đủ lớn hơn A.
Vậy giả sử sai, suy ra trong G A tất cả các đỉnh là cô lập, ta có điều cần chứng
minh.
CuuDuongThanCong.com https://fb.com/tailieudientucntt
---------------------------------------------------------------------------------------------------------------------------
22
Bài 12 (USA 1982). Trong nhóm 1982 người, với 4 người bất kì thì có ít nhất 1
người quen với 3 người còn lại. Có ít nhất bao nhiêu người mà người này quen
với tất cả những người còn lại.
Giải
Phát biểu lại theo ngôn ngữ graph: Xét graph G có 1982 đỉnh ứng với 1982
người, nối 2 đỉnh nếu 2 người tương ứng quen nhau.
Xét 4 đỉnh bất kì thì có ít nhất 1 đỉnh kề với cả 3 đỉnh còn lại. Cần xem xét
trong G có ít nhất bao nhiêu đỉnh kề với tất cả những đỉnh còn lại.
Nếu bất kì đỉnh nào trong G cũng kề với các đỉnh còn lại thì giá trị nhỏ nhất là
1981.
Nếu có 2 đỉnh A, B không kề nhau. Xét 2 đỉnh bất kì C, D khác A, B thì 2 đỉnh
này phải kề nhau, ngược lại 4 đỉnh A, B, C, D không có đỉnh nào kề với cả 3
đỉnh còn lại.
Nếu A và B cùng kề với tất cả các đỉnh còn lại thì có 1980 đỉnh mà mỗi đỉnh kề
với tất cả các đỉnh còn lại.
Nếu có C không kề với A thì đỉnh D bất kì sẽ kề với cả A, B, C. Khi đó có 1979
đỉnh mà mỗi đỉnh kề với tất cả những đỉnh còn lại.
Vậy có ít nhất 1979 đỉnh mà mỗi đỉnh kề với tất cả những đỉnh còn lại.
Bài 13 (Taiwan 2001). Cho số nguyên 3n  và 1 2, ,..., nA A A là các tập con phân
biệt của của  1,2,...,S n . Chứng minh tồn tại phần tử x S sao cho các tập
     1 2 ,  ,..., nA x A x A x là phân biệt.
Giải
Xét graph G với các đỉnh 1 2, ,..., nA A A . Với phần tử y S , nếu 2 tập  iA y và
 jA y trùng nhau thì nối iA và jA bởi 1 cạnh, mỗi y ta chỉ nối 1 cạnh (nghĩa
là nếu có nhiều cặp 2 tập  ,i jA A cùng thỏa mãn điều kiện thì ta chọn ra 1 cặp
bất kì nào đó). Dễ thấy không có 2 đỉnh nào được nối 2 lần, ta có graph đơn.
Giả sử phản chứng, không tồn tại phần tử x thỏa mãn yêu cầu bài toán. Khi đó
với mỗi phần tử của S sẽ có đúng 1 cặp đỉnh được nối với nhau. Hơn nữa không
xảy ra trường hợp tồn tại ,i jA A phân biệt mà
CuuDuongThanCong.com https://fb.com/tailieudientucntt
---------------------------------------------------------------------------------------------------------------------------
23
       1 1 2 2  ,  i j i jA x A x A x A x  với 1 2,x x phân biệt (nếu có thì ,i jA A
trùng nhau).
Suy ra graph có ít nhất n cạnh, suy ra tồn tại 1 chu trình, giả sử là
 1 2, ,..., , 3kA A A k  , có nghĩa tồn tại các phần tử phân biệt 1 2, ,..., kx x x S sao
cho    1 1 2 1  ,A x A x        2 2 3 2 1  ,...,  k k kA x A x A x A x  .
Do điều kiện    1 1 2 1 A x A x nên 1x thuộc đúng 1 trong 2 tập 1 2,S S , giả sử
1 2x A
Suy ra 1 3x A (do 1 2x x ), tiếp tục quá trình suy ra 1 kx A và cuối cùng
1 1x A , mâu thuẫn.
Vậy giả sử phản chứng sai, ta có điều chứng minh.
Bài 14 (USA TST 2002). Cho n là số nguyên dương và S là tập có 2 1n
 phần
tử. Xét hàm f đi từ tập các tập con có 2 phần tử của S vào tập  1
0,1,2,...,2 1n
 .
Giả sử với bộ ba phần tử bất kì  , ,x y z của S, một trong các số
        , , , , ,f x y f y z f z x bằng tổng của 2 số còn lại. Chứng minh tồn tại
, ,a b c S sao cho         , , , 0f a b f b c f c a   .
Giải
Xét tập con S’ của S sao cho 1
' 2 1n
S 
  và với mọi , 'x y S thì   ,f x y là
số chẵn.
Đặt   
  ,
, , , '
2
f x y
g x y x y S   , ta có hàm g từ S’ đến  2
0,1,2,...,2 1n

có tính chất giống như hàm f. Vậy ta sẽ xét hàm f từ S’ đến  2
0,1,2,...,2 1n

mà với mọi a, b thuộc S’ thì   ,f a b lẻ hoặc   , 0f a b  .
Xét graph G có 2 1n
 đỉnh tương ứng các phần tử trong S. Hai đỉnh a, b nối với
nhau nếu và chỉ nếu   ,f a b lẻ. Ta chứng minh G là hai phần.
Chú ý rằng với bất kì các đỉnh a, b, c thì có 0 hoặc 2 cạnh tạo ra từ chúng.
CuuDuongThanCong.com https://fb.com/tailieudientucntt
---------------------------------------------------------------------------------------------------------------------------
24
Nếu G không là hai phần từ có 1 chu trình lẻ, giả sử chu trình lẻ có độ dài ngắn
nhất là 1 2 2 1... kv v v  . Xét 3 đỉnh 1 3 4, ,v v v có 0 hoặc 2 cạnh tạo ra từ chúng. Do đã
có cạnh 3 4v v nên sẽ có 1 trong 2 cạnh 1 3v v hoặc 1 4v v , hơn nữa không tồn tại
cạnh 1 3v v (nếu không 3 đỉnh 1 2 3, ,v v v có 3 cạnh) nên có cạnh 1 4v v , suy ra chu
trình 1 4 5 2 1... kv v v v  có độ dài nhỏ hơn hoặc bằng 1 2 2 1... kv v v  , mâu thuẫn.
Vậy G là 2 phần, có 1 phần có ít nhất 3 phần tử, 3 phần tử này không nối với
nhau, ta có điều chứng minh.
Bài 15 (IMO Shortlist 2002, C6). Cho n là số nguyên dương chẵn. Chứng minh
tồn tại 1 hoán vị  1 2, ,..., nx x x của  1,2,...,n sao cho với bất kì  1,2,...,i n , số
1ix  là 1 trong các số 2 ,2 1,2 ,2 1i i i ix x x n x n    , quy ước n i ix x  .
Giải
Đặt 2n m .
Xây dượng graph có hướng với m đỉnh và 2m cạnh. Với mỗi
i m , đỉnh i có 2 cạnh đi ra được đánh số 2 1,2i i và 2 cạnh đi
vào được đánh số là ,i i m . Ta cần chứng minh có 1 chu trình
Ole, bởi vì các cặp cạnh liên tiếp tạo ra có 1 trong các dạng
       ,2 1 , ,2 , ,2 , ,2 1i i i i i m i i m i    .
Ta thấy, bậc vào của mỗi đỉnh bằng bậc ra suy ra tồn tại 1 chu trình Ole.
Thật vậy, ta có thể chứng minh điều trên bằng phương pháp quy nạp.
Có một đường đi từ 1 đến k: Do có 1 đường đi từ 1 đến j khi 2 j k hoặc
2 1j k  và 1 cạnh từ j đến k suy ra có 1 đường đi từ 1 đến k.
Suy ra điều chứng minh.
Bài 16 (St Petersburg 1996, P4). Trong 1 nhóm người, một số người quen biết
nhau và một số người không. Mỗi tối, một người nào đó đãi tiệc mời tất cả
những người mình quen biết và giới thiệu họ cho nhau. Biết rằng, sau khi mỗi
người đều đã tổ chức tiệc vẫn còn hai người trong nhóm vẫn chưa quen nhau.
Chứng minh tại bữa tiệc tiếp theo, họ vẫn không được giới thiệu cho nhau.
Giải
CuuDuongThanCong.com https://fb.com/tailieudientucntt
---------------------------------------------------------------------------------------------------------------------------
25
Xây dựng graph có n đỉnh (giả giả nhóm này có n người), nối 2 đỉnh khi 2
người tương ứng quen nhau.
Mỗi bước thực hiện công việc như sau: Chọn 1 đỉnh và nối đôi một tất cả các
đỉnh kề với đỉnh này. Khi đó ta được 1 graph con đầy đủ. Sau khi thực hiện với
tất cả các đỉnh ta sẽ thu được các graph đầy đủ như trên. Theo giả thiết bài toán
sẽ tồn tại 2 graph rời nhau. Các bước thực hiện tiếp theo sẽ không tạo thêm cạnh
nào, hay 2 graph này luôn rời nhau. Ta có điều chứng minh.
Bài 17 (Czech – Slovak Mathch 1997, P2). Trong một hội đồng có nhiều hơn 6
người, mỗi người trao đổi thư với đúng 3 người khác. Chứng minh có thể chia
hội đồng này thành 2 nhóm khác rỗng mà mỗi người trao đổi thư với ít nhất 2
người trong nhóm của anh ta.
Giải
Xét graph G có n đỉnh (n là số người trong hội đồng), 2 đỉnh được nối với nhau
nếu 2 người tương ứng trao đổi thư với nhau.
Chứng minh có thể chia graph này thành 2 graph con mà trong mỗi graph bậc
của mỗi đỉnh ít nhất là 2.
Theo giả thiết mỗi đỉnh có bậc đúng là 3, suy ra trong graph tồn tại chu trình.
Xét chu trình ngắn nhất, mỗi đỉnh trong chu trình này nối với ít nhất 2 đỉnh
trong chu trình đó, vậy chu trình này là 1 graph con thỏa mãn điều kiện, kí hiệu
graph này là G’.
Xét các đỉnh còn lại, nếu có các đỉnh 1 2, ,..., kv v v , mỗi đỉnh nối với ít nhất 2 đỉnh
của graph con trên thì graph  1 2' , ,..., kG v v v là graph con thỏa mãn.
Ngược lại, các đỉnh còn lại, mỗi đỉnh chỉ nối với nhiều nhất 1 đỉnh trong G’ thì
graph 'G G thỏa mãn điều kiện.
Ta có điều chứng minh.
Bài 18 (BAMO 2005/4). Có 1000 thành phố ở đất nước của Ole, một số cặp 2
thành phố được nối với nhau bởi 1 con đường đất. Biết rằng giữa 2 thành phố
bất kì có 1 đường đi (đi qua 1 số đường đất liên tiếp). Chứng minh chính phủ có
thể lát một số con đường sao cho ở 1 thành phố bất kì luôn có lẻ con đường
được lát xuất phát từ đây.
Giải
CuuDuongThanCong.com https://fb.com/tailieudientucntt
---------------------------------------------------------------------------------------------------------------------------
26
Mấu chốt của bài toán 1000 là số chẵn, ta có thể thay 1000 bằng 1 số chẵn bất kì
thì bài toán vẫn đúng.
Theo kết quả Kết quả 10 ta có thể hạn chế graph về trường hợp graph con là 1
cây.
Ta chứng minh với cây có chẵn đỉnh luôn có thể lát 1 số cạnh mà tại mỗi đỉnh
có lẻ cạnh được lát.
Xét 1 lá và đỉnh v kề với lá này, lát cạnh nối 2 đỉnh và xóa bỏ 2 đỉnh này.
Khi đó ta sẽ thu được các thành phần liên thông rời nhau, mỗi thành phần liên
thông đều có đỉnh nối với v (vì nếu không thì ngay lúc đầu thành phần liên
thông này không liên thông với v, mâu thuẫn).
Có chẵn đỉnh còn lại nên số thành phần liên thông có lẻ đỉnh là chẵn.
Ta xét thành phần liên thông G có lẻ đỉnh, trong thành phần này có đỉnh u nối
với v, ta lát cạnh nối 2 đỉnh này và loại đỉnh u, khi đó  G u được chia thành
các thành phần liên thông có ít đỉnh hơn.
Khi đó đỉnh v có lẻ cạnh được lát (có cạnh nối với lá lúc đầu và các cạnh vừa
xét).
Với thành phần liên thông có chẵn đỉnh thì làm tương tự như graph ban đầu.
Vậy sau mỗi lần lát cạnh thì sẽ thu được các thành phần liên thông mới có ít
đỉnh hơn thành phần liên thông ban đầu. Với mỗi thành phần liên thông, tùy
thuộc số đỉnh chẵn hay lẻ ta sẽ tiếp tục theo 1 trong 2 bước trên. Ta có điều
chứng minh.
Bài 19 (Belarus 2010). Cho 3n  điểm phân biệt trên mặt phẳng sao cho không
có 3 điểm thẳng hàng. Nối tất cả các đoạn thẳng tạo bởi 2 điểm trong n điểm
trên. Mỗi đoạn được tô bởi 1 trong 4 màu sao cho nếu với 1 tam giác có 2 cạnh
được tô cùng màu thì cạnh còn lại cũng được tô màu này. Biết rằng mỗi màu
được dùng ít nhất 1 lần. Tìm giá trị lớn nhất của n.
Giải
Xét mô hình đồ thị như bài toán. Giả sử các màu được dùng là 1, 2, 3, 4.
CuuDuongThanCong.com https://fb.com/tailieudientucntt
---------------------------------------------------------------------------------------------------------------------------
27
Xét các đồ thị con đầy đủ mà các cạnh được tô cùng màu (ta gọi là đồ thị đơn
sắc). Nhận thấy luôn tồn tại những đồ thị như vậy, ít nhất 1 cạnh được tô màu là
1 đồ thị thỏa mãn.
Xét đồ thị G nhiều đỉnh nhất thỏa mãn điều trên và các cạnh của nó được tô màu
1.
Do mỗi màu được dùng ít nhất 1 màu nên có đỉnh v không thuộc G.
Xét các cạnh nối v với các đỉnh trong G, nếu có cạnh được tô màu 1 thì tất cả
các cạnh đều được tô đỏ, khi đó  G v đầy đủ, đơn sắc nhưng nhiều đỉnh hơn
G. Nếu có 2 cạnh cùng được tô màu khác thì cạnh còn lại phải được tô màu này,
mâu thuẫn do nó đang được tô màu 1.
Suy ra tại mỗi đỉnh v không thuộc G thì các cạnh được tô màu đôi một khác
nhau và khác mà 1, suy ra G có nhiều nhất 3 đỉnh. Khi đó tại mỗi đỉnh, chỉ có
nhiều nhất 2 cạnh cùng màu (vì nếu có 3 cạnh cùng màu thì có 3 đỉnh cùng với
đỉnh này tạo thành 1 đồ thị đầy đủ đơn sắc).
Có 4 màu được dùng nên bậc của mỗi đỉnh không vượt quá 8, hay số đỉnh
không vượt quá 9.
Với 9 đỉnh 1 2 3 1 2 3 1 2 3, , , , , , , ,a a a b b b c c c thực hiện việc tô màu như sau: tô màu 1
cho các cạnh của các đồ thị đầy đủ      1 2 3 1 2 3 1 2 3, , , , , , , ,a a a b b b c c c .
Tô màu 2 cho các cạnh của các đồ thị đầy đủ  , , , 1,2,3i i ia b c i 
Tô màu 3 cho các cạnh của các đồ thị đầy đủ  1 2, , , 1,2,3i i ia b c i  
Tô màu 4 cho các cạnh của các đồ thị đầy đủ  2 4, , , 1,2,3i i ia b c i   .
Ta có điều chứng minh.
Bài 20 (IMO 1990). Cho E là tập chứa 2 1n  điểm trên đường tròn với 3n  là
số nguyên. Tô màu đen k điểm trong E. Một cách tô màu được gọi là tốt nếu tồn
tại ít nhất 2 điểm được tô đen mà bên trong 1 cung tròn có đầu mút là 2 điểm
này có đúng n điểm của E. Tìm giá trị nhỏ nhất của k sao cho với mọi cách tô k
điểm đều là 1 cách tô màu tốt.
Giải
CuuDuongThanCong.com https://fb.com/tailieudientucntt
---------------------------------------------------------------------------------------------------------------------------
28
Xét graph G mà các đỉnh là các điểm của E. Hai đỉnh được nối với nhau nếu bên
trong 1 cung tròn có đầu mút là 2 điểm này có đúng n điểm của E. Tìm giá trị
nhỏ nhất của k để với k đỉnh bất kì trong G đều có 2 đỉnh kề.
Nhận thấy bậc của mỗi đỉnh là 2, suy ra G là hợp của các chu trình rời nhau.
Chú ý rằng, trong 1 chu trình độ dài r thì cần chọn ra ít nhất 1
2
r 
   
đỉnh để
đảm bảo luôn có 2 đỉnh kề nhau.
Giả sử các điểm thuộc E theo thứ tự cùng chiều kim đồng hồ trên đường tròn là
1 2 2 1, ,..., nA A A  .
Khi đó chu trình chứa 1A là:  1 2 4 5 7 8 1 1, , , , , ,..., ,n n n nA A A A A A A A    .
Khi đó G là hợp của nhiều nhất 3 chu trình (chứa 1A , 2A hoặc 3A ), số lượng chu
trình phụ thuộc số dư khi chia 2 1n  cho 3.
Nếu 2 1n  không chia hết cho 3 thì G là 1 chu trình có độ dài 2 1n  , các đỉnh
cần chọn ít nhất là
2 1
1
2
n
n
 
   
.
Nếu 2 1n  chia hết cho 3 thì G là hợp của 3 chu trình, mỗi chu trình có độ dài
2 1
3
n 
, các đỉnh cần chọn ít nhất là
2 1
1 3 1
6
n
n
 
    
.
Vậy giá trị nhỏ nhất của k là
2 1
1
2
n
n
 
   
khi 2 1n  không chia hết cho 3 và
là
2 1
1 3 1
6
n
n
 
    
khi 2 1n  chia hết cho 3 .
Bài 21. Một công ty muốn xây một công trình có kích thước 2017 2017 gồm
2
2017 phòng, mỗi phòng có kích thước 1 1 , một số phòng kề nhau (chung
cạnh) được nối với nhau bằng 1 cửa giữa 2 phòng. Hỏi có thể xây dựng mà mỗi
phòng có đúng 2 cửa hay không?
Giải
Giả sử có thể xây dựng mà mỗi phòng có đúng 2 cửa.
CuuDuongThanCong.com https://fb.com/tailieudientucntt
---------------------------------------------------------------------------------------------------------------------------
29
Xét graph G có 2
2017 đỉnh, 2 đỉnh được nối với nhau nếu có cửa nối 2 phòng
tương ứng.
Khi đó mỗi đỉnh có bậc đúng bằng 2 suy ra G là hợp của các chu trình rời nhau.
Trên thực tế, để đi từ 1 phòng qua các phòng khác rồi quay lại đúng phòng đó
thì phải qua chẵn cửa (các bước đi qua cửa theo hướng thẳng đứng với 2 chiều
khác nhau là bằng nhau nên tổng bước đi theo hướng thẳng đứng là chẵn, tương
tự tổng bước đi theo hướng nằm ngang đều chẵn).
Suy ra mỗi chu trình có độ dài chẵn nên mỗi chu trình có chẵn đỉnh suy ra G có
chẵn đỉnh, mâu thuẫn khi 2
2017 lẻ.
Vậy giả sử sai, suy ra không thể xây dựng mà mỗi phòng có đúng 2 cửa.
Bài 22 (Iran TST 2006). Một đất nước có 2 hãng hàng không, giữa 2 thành phố
bất kì có đường bay của đúng 1 hãng hàng không. Chứng minh tồn tại 1 thành
phố mà từ một thành phố bất kì có thể chọn đường bay đến đó mà không cần
thay đổi hãng hàng không.
Giải
Xét graph đầy đủ G có số đỉnh bằng số thành phố, mỗi cạnh được tô bởi 1 trong
2 màu xanh, đỏ tương ứng với đường bay của hai hãng hàng không.
Cần chứng minh tồn tại đỉnh v sao cho với đỉnh u bất kì thì tồn tại đường đi đơn
sắc nối u và v.
Ta sẽ chứng minh quy nạp theo số đỉnh của G.
Khi G có 1, 2, 3, 4 đỉnh là hiển nhiên.
Bài 23 (BMO 1987). Một cuộc hội thảo có 1985 người, nếu chọn ra 3 người bất
kì thì luôn có ít nhất 2 người nói cùng ngôn ngữ. Giả sử mỗi người có thể nói
được nhiều nhất 5 ngôn ngữ. Chứng minh có ít nhất 200 người nói được cùng
ngôn ngữ.
Giải
Giả sử tồn tại một người nói được cùng ngôn ngữ với tất cả những người còn
lại, do
1984
200
5
 nên có 200 người có thể nói cùng ngôn ngữ.
CuuDuongThanCong.com https://fb.com/tailieudientucntt
---------------------------------------------------------------------------------------------------------------------------
30
Ngược lại thì có 2 người 1 2,P P không có cùng ngôn ngữ. Xét 1983 người còn
lại, có ít nhất 992 người có thể nói chuyện với 1 trong 2 người, giả sử là 1P .
Xét 992 người này, có ít nhất
992
1 199
5
 
   
người nói cùng ngôn ngữ.
Suy ra có ít nhất 200 người (199 người trên và 1P ) nói cùng ngôn ngữ, có điều
chứng minh.
Bài 24. Trong một hội thảo khoa học tất cả các đại biểu tham dự biết tổng cộng
2n ngôn ngữ 2n  . Mỗi người biết đúng 2 ngôn ngữ và bất cứ hai người nào
cũng biết chung nhiều nhất một ngôn ngữ. Biết rằng với một số nguyên k thỏa
mãn 1 1k n   đều có không quá 1k  ngôn ngữ mà mỗi ngôn ngữ này có
không quá k người biết. Chứng minh rằng ta có thể chọn ra một nhóm 2n đại
biểu biết tổng cộng 2n ngôn ngữ và mỗi ngôn ngữ có đúng 2 đại biểu trong
nhóm biết.
Giải
Xét graph G đỉnh biểu diễn cho “ngôn ngữ”, cạnh nối hai đỉnh biểu diễn “người
biết hai ngôn ngữ đó”. Vậy G là đồ thị 2n đỉnh. Điều kiện “hai người biết chung
nhiều nhất một ngôn ngữ” suy ra G là đơn. Điều kiện còn lại cho biết: với mỗi k
nguyên 1 1k n   có không quá 1k  ngôn ngữ mà mỗi ngôn ngữ này có
không quá k người biết suy ra mỗi đỉnh có bậc nhỏ hơn hoặc bằng k (*).
Ta cần chứng minh trong G tồn tại chu trình Hamilton.
Ta chứng minh điều này bằng phản chứng. Giả sử trong G không có chung trình
H. Khi đó tập các đỉnh không kề nhau của G là không rỗng và hữu hạn. Bằng
cách thêm dần hai cạnh nối hai đỉnh không kề nhau ta sẽ xây dựng đồ thị 2n
đỉnh G thỏa mãn 1) (*), 2) trong G không có chu trình H 3) Khi thêm cạnh nối
hai đỉnh bất kì không kề nhau của G ta sẽ nhận được đồ thị có chu trình H.
Xét G với v là đỉnh của G kí hiệu  f v là bậc của v.
a) Từ 2) và 3) suy ra giữa hai đỉnh bất kì không kề nhau của G đều tồn tại
một đường đi nhận hai đỉnh ấy làm hai đầu mút, đi qua tất cả các đỉnh của G và
có độ dài 2 1n 
CuuDuongThanCong.com https://fb.com/tailieudientucntt
---------------------------------------------------------------------------------------------------------------------------
31
b) Nếu hai đỉnh v và v’ của G có    , 'f v n f v n  thì v và v’ phải kề
nhau.
Thật vậy, giả sử v và v’ không kề nhau thì có đường đi
1 2 2 1 2, ,..., ( , 'n nv v v v v v v  đi qua tất cả các đỉnh của G và có độ dài 2 1n  . Giả
sử  f v s n  . Kí hiệu 1 2 1 2, ,..., (2 ... 2 )si i i sv v v i i i n     là các đỉnh kề với
1v v . Khi đó với mỗi 1,2,...,j s các đỉnh ( ) 1jiv  không kề với 2 'nv v vì nếu
ngược lại thì chu trình H trong G là 1 2 ( ) 1 2 2 1.... ...j ji n n iv v v v v v  mâu thuẫn với 2). Từ
đó suy ra  ' 2 ( 1) 1f v n s n     (do s n ), mâu thuẫn với  'f v n . Vậy
v, v’ phải kều nhau.
c) Từ b) suy ra tập v gồm các đỉnh v của G mà   1f v n  là không rỗng,
vậy có  max 1v V f v m n    . Lấy 1v mà  1f v m . Điều kiện (*) với
1k n  nói rằng có ít nhất  2 1 1 2n n n     đỉnh có bậc n , do với
1k n  nói rằng có ít nhất một trong các đỉnh này, chẳng hạn 2nv , không kề
với 1v . Suy ra có đường đi 1 2 2, ,..., nv v v đi qua tất cả các đỉnh của G và có độ dài
2 1n  . Kí hiệu 1 2 1 2, ,..., (2 ... 2 )mi i i mv v v i i i n     là các đỉnh không kề với 1v
thì lập luận như ở b) chứng tỏ với mọi 1j n  ta có ( ) 1jiv  không kề với
2nv (chú ý rằng điều kiện (*) với k=1) chứng tỏ mọi đỉnh của G có bậc 2 . Áp
dụng điều kiện (*) với k=m  2 1m n   suy ra       1 21 1 1
, ,..., mi i i
v v v  
phải có
ít nhất một đỉnh qv có   1qf v m  . Từ định nghĩa của m suy ra  qf v n
như vậy 2,q nv v có  qf v n ,  2nf v n mà không kề nhau, mâu thuẫn với b).
Mâu thuẫn này cho ta điều phải chứng minh.
Bài 25. Cho số nguyên dương 2n  và 1 cây có n đỉnh. Tại mỗi đỉnh của cây đã
cho ghi 1 số thực và trên mỗi cạnh ghi số thực bằng tích của 2 số đã ghi ở 2 đầu
mút của cạnh đó. Kí hiệu S là tổng của các số ghi trên các cạnh và 1 2, ,..., nx x x là
các số ghi trên các đỉnh. Chứng minh rằng  2
1
1. 2 *
n
i
i
n x S

  .
Giải
Chú ý rằng 1 cây n đỉnh luôn có 1n  cạnh và có ít nhất 1 lá (điểm treo).
CuuDuongThanCong.com https://fb.com/tailieudientucntt
---------------------------------------------------------------------------------------------------------------------------
32
Ta đi tìm giá trị lớn nhất có thể của S và chứng minh giá trị này thỏa mãn bất
đẳng thức  * .
Giả sử 0, 1,ix i n   (nếu có số âm thì làm S giảm mà vế trái của (*) không
thay đổi.
Đặt  1 max : 1,2,...,ix x i n  .
Giả sử cây ban đầu là 0C , ta xây dựng một cây mà S lớn nhất.
Ta coi đỉnh tx là đỉnh được viết số tx .
Xét đỉnh 1x và jx là 1 lá của cây suy ra tồn tại ix kề với jx .
Nếu ix khác 1x và không kề với 1x thì bỏ cạnh i jx x và nối
cạnh 1 jx x
Khi đó ta được cây mới 1C mà    0 1S C S C , vậy sau mỗi quá trình biến đổi
thì giá trị của S không giảm và nó sẽ đạt giá trị lớn nhất khi không thể biến đổi
cây.
Tiếp tục quá trình với các lá còn lại của cây đến khi tất cả lá đều kề với 1x , ta
nhận thấy tất cả các đỉnh 2 3, ,..., nx x x đều kề với 1x .
Thật vậy, nếu có đỉnh kx không kề với 1x , xét đường đi từ 1x đến kx và tiếp tục
đường đi cho đến khi gặp 1 là mx , khi đó ta có chu trình (do mx đang kề với 1x ),
mâu thuẫn.
Khi đó giá trị lớn nhất của S là max 1 2 1 3 1... nS x x x x x x    .
Ta có 2 2 2
1
1 2
1. 1 1
n n
i i
i i
n x n x n x
 
     
2
2 1
1 max
2 2
1 2 2 2
1
n n
i i
i i
x
n x x x S S
n 
 
      
 
 
Ta có điều chứng minh.
xi
xj
x1
CuuDuongThanCong.com https://fb.com/tailieudientucntt
---------------------------------------------------------------------------------------------------------------------------
33
Bài 26. Trong không gian cho n điểm ( 2n  ) mà không có bốn điểm nào đồng
phẳng và cho  21
3 4
2
n n  đoạn thẳng mà tất cả các đầu mút của chúng nằm
trong số n điểm đã cho. Biết rằng có ít nhất một đoạn thẳng mà sau khi bỏ nó đi
(giữ nguyên các đầu mút) thì sẽ tồn tại hai điểm phân biệt mà không phải là hai
đầu mút của một đường gấp khúc nào. Hãy tìm số k lớn nhất sao cho có k đoạn
thẳng tạo thành đường gấp khúc khép kín mà mỗi đỉnh của nó là mút của đúng
hai đoạn thẳng thuộc đường gấp khúc đó.
Giải
Xét graph G có tập đỉnh là tập gồm n điểm đã cho và tập cạnh là tập gồm
 21
3 4
2
n n  đoạn thẳng đã cho. Từ giả thiết của bài toán ta thấy trong G tồn
tại một cạnh mà sau khi bỏ nó đi thì được G’ không liên thông. Giả sử a và b là
hai đỉnh không liên thông với nhau trong G’.
Gọi aV và bV lần lượt là tập gồm tất cả các đỉnh của G’ mà liên thông với a và b.
Giả sử 1aV n và 2bV n .
Dễ thấy G’ có  21
3 4
2
n n  cạnh; 1 2 1 21, 1,n n n n n    và
        2
1 1 2 2 1 2 1 2
1 1 1 1
3 4 1 1 1
2 2 2 2
n n n n n n n n n n n n            hay
     1 2 1 2 1 21 1 1 0n n n n n n n        .
Do đó
  
  
1 2
1 2 1 2
1 1 0
1 0
n n
n n n n n
  

    
Vậy 1 21, 1n n n   hoặc 2 1n n  , 1 1n  .
Từ đó suy ra G’ có một đỉnh cô lập và  1n  đỉnh mà bậc của mỗi đỉnh bằng
2n  . Do đó G có một đỉnh bậc 1,  2n  đỉnh mà bậc của mỗi đỉnh bằng
2n  và một đỉnh có bậc bằng 1n  . Bởi thế chu trình đơn có độ dài lớn nhất
trong G là chu trình đơn độ dài 1n  nếu 4n  , 0 nếu 2n  hoặc 3n 
CuuDuongThanCong.com https://fb.com/tailieudientucntt
---------------------------------------------------------------------------------------------------------------------------
34
Vậy
 max
1 ( 4)
0 2, 3
n n
k
n n
 
 
 
.
Bài 27. Ở một nước có 25 thành phố. Hãy xác định số k bé nhất sao cho có thể
thiết lập các đường bay (dùng cho cả đi lẫn về) giữa các thành phố để hai điều
kiện sau được đồng thời thỏa mãn
1. Từ mỗi thành phố có đường bay trực tiếp đến đúng k thành phố khác
2. Nếu giữa hai thành phố không có đường bay trực tiếp thì tồn tại ít nhất
một thành phố có đường bay trực tiếp đến hai thành phố đó.
Giải
Giả sử k là số sao cho có thể thiết lập được hệ thống đường bay thỏa mãn các
điều kiện của đề bài. Khi đó, tổng số đường bay trực tiếp giữa hai thành phố sẽ
là
25
2
k
. Suy ra  0 mod2k 
Xét một thành phố A bất kỳ. Theo giả thiết, từ A có đường bay trực tiếp đến k
thành phố khác, gọi là 1 2, ,..., kA A A . Mỗi thành phố , 1,iA i k , lại có đường bay
trực tiếp đến k – 1 thành phố khác, (không kể A). Hơn nữa, ta lại có: Nếu từ B
đến A không có đường bay trực tiếp thì B phải có đường bay trực tiếp đến ít
nhất một thành phố iA . Từ những lập luận trên suy ra, số thành phố chỉ có thể
tối đa là   2
1 k k k –1 1k    . Như vậy 2
25 1k  . Kết hợp với
 0 mod2k  , suy ra 6k  .
Với 6k  ta sẽ chỉ ra cách thiếp lập hệ thống đường bay thỏa mãn các điều kiện
cỉa đề bài. Chia 25 thành phố thành năm nhóm, mỗi nhóm gồm năm thành phố.
Các thành phố của nhóm thứ , 1,5i i  , ta kí hiệu bởi          
1 2 3 4 5, , , ,
i i i i i
A A A A A . Với
các thành phố trong cùng nhóm i , ta thiết lập các đường bay
                   
1 2 2 3 3 4 4 5 5 1, , , ,
i i i i i i i i i i
A A A A A A A A A A . Giữa các thành phố thuộc hai nhóm ,i j
bất kỳ,  1,2,3,4,5i j  , xây dựng các đường bay sau
                   
1 1 2 4 3 2 4 5 5 3, , , ,
i j i j i j i j i j
A A A A A A A A A A .
Bằng cách xây dựng các đường bay như trên, ta có: Từ thành phố A bất
kỳ sẽ có đường bay trực tiếp đến đúng 2 thành phố, trong cùng nhóm với A và
có đường bay trực tiếp đến đúng 4 thành phố khác nhóm với A. Do vậy từ mỗi
thành phố sẽ có đường bay trực tiếp đến đúng 6 thành phố khác.
CuuDuongThanCong.com https://fb.com/tailieudientucntt
---------------------------------------------------------------------------------------------------------------------------
35
Hơn nữa, với A, B là hai thành phố bất kỳ mà giữa chúng không có
đường bay trực tiếp ta thấy:
- Nếu A, B cùng thuộc nhóm thì dễ thấy luôn tồn tại 1 thành phố trong
nhóm đó mà từ C có đường bay trực tiếp đến cả A và B.
- Nếu A, B không cùng nhóm thì qua hình vẽ trên dễ dàng kiểm tra
được sự tồn tại của thành phố C mà từ C có đường bay trực tiếp đến cả
A và B.
Vậy min 6k  .
Bài 28. Trong một cuộc hội thảo có  10n n  người tham dự. Biết rằng:
1. Mỗi người quen với ít nhất
2
3
n  
  
người tham dự;
2. Hai người bất kỳ A và B nếu không quen nhau thì quen nhau gián tiếp
nghĩa là có  1k k  người 1 2, ,..., kA A A sao cho A quen 1, iA A quen
 1, 1,2,..., 1iA i k   và kA quen B;
3. Không thể xếp n người thành một hàng ngang sao cho hai người cạnh
nhau bất kỳ đều quen nhau.
Chứng minh rằng có thể chia n người thành hai nhóm: nhóm thứ nhất xếp được
quanh một bàn tròn sao cho hai người cạnh nhau bất kỳ đều quen nhau, còn
nhóm thứ hai gồm người đôi một không quen nhau.
Giải
Chuyển bài toán sang ngôn ngữ Graph, trong đó mỗi người coi là một điểm
trên mặt phẳng, còn quan hệ quen nhau coi là một cạnh (1 đoạn thẳng với giả
thiết rằng các đoạn thẳng này không cắt nhau trừ hai điểm đầu mút), ta có graph
G đơn, vô hướng với tập đỉnh gồm n điểm  1 2, ,..., np A A A và bậc của đỉnh A
bất kỳ là  
2
3
n
d A
 
   
Điều kiện “Hai người bất kỳ quen nhau hoặc quen nhau gián tiếp chứng tỏ
Graph G là liên thông.
Trong G (hữu hạn) xét đường gấp khúc nhiều cạnh nhất Po, giả sử Po có k đỉnh
là  0 1 2, ,..., kP A A A với 1( 1,2,..., 1)i iA A i k   là các cạnh ( iA kề với 1iA )
CuuDuongThanCong.com https://fb.com/tailieudientucntt
---------------------------------------------------------------------------------------------------------------------------
36
Do điều kiện (3) thì 1k n 
Gọi N(A) là tập các đỉnh kề với đỉnh A. Ta có    1 2,..., kN A A A và
   2 1,..., kN Ak A A 
Vì trái lại thì tồn tại đường gấp khúc khác có nhiều cạnh hơn Po.
Giả sử      1 2
, ,..., , 1,2,...,si i i iN A A A A i n  ký hiệu
       1 2 1 21 1 1 1 1 1, ,..., , , ,...,s si i i i i i i iN A A A A N A A A A
 
      
Do 1k n  nên tồn tại đỉnh 0B P . Ta có    kN B N A

  
Thật vậy nếu    j kA N B N A

   thì tồn tại đường gấp khúc
 1 1 1 1,..., , , ,..., , ,j k k j jA A A A A A B   có k+1 cạnh, trái giả thiết đối với 0P . Lập luận
tương tự có    1N B N A

   .
Ta cũng có    1 kN A N A

   vì nếu trái lại thì
           1 1
2 2
3 3 1 1
3 3
k kN B N A N A N B B A N A
n n
n
  
    
    
         
Suy ra số đỉnh của tập hợp này lớn hơn hoặc bằng n mà tập hợp đó không chứa
đỉnh B. Mâu thuẫn.
Vậy    1i kA N A N A

  
Khi đó tồn tại đường gấp khúc khép kín có k – 1 đỉnh thuộc tập  c iP A là
 1 2 1 1 1, ,..., , , ,...,i k k iA A A A A A  
Tập còn lại chứa các đỉnh đôi một không kề nhau (không có đoạn thẳng nối
chung) vì nếu trái lại, chẳng hạn có  1 2 0,  iB B P A mà 1B kề với 2B do tính
liên thông tồn tại đường gấp khúc chứa 1B , 2B và  0  iP A có nhiều cạnh hơn
0P mâu thuẫn.
CuuDuongThanCong.com https://fb.com/tailieudientucntt
---------------------------------------------------------------------------------------------------------------------------
37
Bài 29. Cho n là số nguyên dương sao cho tồn tại đồ thị đơn G, vô hướng có n
đỉnh, bậc mỗi đỉnh nhỏ hơn 1n  và thỏa mãn: 2 đỉnh bất kì là 2 đầu mút của
duy nhất 1 đường đi đơn có độ dài không vượt quá 2. Chứng minh 1n  là số
chính phương.
Giải
Xét đồ thị  ,G V E với  1 2, ,..., nV A A A , đặt  i id A d , suy ra
1 2
n
i
i
d
E

  .
Số đường đi có độ dài 1 bằng số cạnh.
Số đường đi có độ dài 2 bằng
 2
1 1
1
.
2i
n n
i i
d
i i
d d
C
 

 
Do chọn 2 điểm bất kì thì có đúng 1 đường đi có độ dài không quá 2 nên tổng số
đường đi độ dài 1 và đường đi độ dài 2 bằng số cặp 2 điểm, ta có:
   
 2 2
1 1 1
1 1
1
2 2 2
n n n
i ii
n i
i i i
d d n nd
C d n n
  
 
       
Ta chứng minh bậc của 2 đỉnh bất kì a, b bằng nhau.
Xét đỉnh x có bậc nhỏ nhất, giả sử là t.
Tập  1 2, ,...,x tS x x x chứa các đỉnh kề với x.
Với mỗi 1,2,...,i t , đặt iA là tập các đỉnh khác x và kề với ix .
Nhận thấy các tập iA là rời nhau (nếu có y thuộc ,i jA A thì có 2 đường đi
 , ,ix x y và  , ,jx x y cùng có độ dài 2 nối x và y,
Hợp của tất cả các tập này cùng với x, xS là tập tất cả các đỉnh,
Hơn nữa, mỗi phần tử trong tập iA kề với nhiều nhất 1 đỉnh trong tập jA ,
Hai phần tử trong cùng tập jA thì không kề nhau.
Suy ra bậc của mỗi phần tử trong  1,2,...,iA i t không vượt quá t, do tính nhỏ
nhất nên bậc của tất cả các đỉnh này là t.
CuuDuongThanCong.com https://fb.com/tailieudientucntt
---------------------------------------------------------------------------------------------------------------------------
38
Suy ra 1 2 ... nd d d   hay   2 2
1 11 . 1n n n d n d     là số chính phương.
Ta có điều chứng minh.
Bài 30. Giả sử có 2017 thành phố mà từ mỗi thành phố đều có đường bay trực
tiếp đến ít nhất 93 thành phố khác và từ 1 thành phố có thể đi đến thành phố bất
kì qua một số đường bay liên tiếp (một đường bay là đường nối trực tiếp 2 thành
phố). Chứng minh có thể đi từ 1 thành phố tùy ý đến 1 thành phố khác với số
đường bay không vượt qua 63.
Giải
Xây dựng mô hình graph: Xét graph G có 2017 đỉnh tương ứng với 2017 thành
phố, 2 đỉnh được nối với nhau nếu 2 thành phố tương ứng có đường bay trực
tiếp. Suy ra bậc của mỗi đỉnh không nhỏ hơn 93 và G là liên thông. Ta cần
chứng minh giữa 2 đỉnh bất kì có đường đi mà độ dài không vượt quá 63.
Giả sử phản chứng, tồn tại 2 đỉnh X, Y mà mọi đường đi nối chúng đều có độ dài
lớn hơn 63. Xét đường đi  0 1 0... , , 64n nA A A X A Y A n   ngắn nhất nối X và
Y.
Xét các đỉnh dạng  3 , 0,1,2,...,21iA i  với 3iV là tập các đỉnh kề với 3iA , suy ra
3 93, 0,21iV i   .
Ta chứng minh 3 3 ,i jV V i j     .
Thật vậy, giả sử tồn tại i j mà 3 3i jV V  , suy ra tồn tại đỉnh T kề với cả
3iA và 3 jA , khi đó có đường đi 0 1 3 3... ...i j nA A A TA A nối X và Y, nhưng đường đi
này có độ dài ngắn hơn đường đi 0 1... nA A A , suy ra mâu thuẫn.
Suy ra G phải có ít nhất 22.93 2046 đỉnh, mâu thuẫn.
Vậy giả sử sai, ta có điều chứng minh.
Bài 31. Trong một hội thảo khoa học, mỗi người quen với ít nhất 3 người khác.
Chứng minh ta có thể chọn ra từ những người này một nhóm người thỏa mãn
đồng thời 2 điều kiện:
1. Số người của nhóm không chia hết cho 3;
CuuDuongThanCong.com https://fb.com/tailieudientucntt
---------------------------------------------------------------------------------------------------------------------------
39
2. Có thể xếp tất cả những người này quanh bàn tròn mà mỗi người ngồi
giữa 2 người quen.
Giải
Xét graph G với đỉnh tương ứng với những người trong cuộc hội thảo, 2 đỉnh
được nối nếu 2 người tương ứng quen nhau. Khi đó mỗi đỉnh có bậc không nhỏ
hơn 3. Cần chứng minh G có chu trình độ dài không chia hết cho 3.
Xây dựng đường đi từ đỉnh 1A bất kì của G: Tồn tại 2A kề 1A , 3 1A A kề với
2A , 4 2A A kề với 3A .
+ Nếu 4A kề với cả 1 2,A A thì dừng lại;
+ Nếu 4A không kề với 1 trong 2 đỉnh 1 2,A A thì tiếp tục mở rộng tới các đỉnh
tiếp theo đến khi gặp đỉnh nA kề với 3 đỉnh trên đường đi đang xét, giả sử nA
kề với  1, , 1 2n i jA A A i j n     .
Khi đó ta có đường đi dạng như sau
Với cách xây dựng đường đi này, ta có 3
chu trình 1...i i j n iA A A A A ; 1 1... ...i i j n n iA A A A A A  và 1 1...j j n n jA A A A A 
Giả sử cả 3 chu trình trên đều có độ dài chia hết cho 3 thì ta có hệ điều kiện
 
   
 
 
 
 
2 3 2 3
1 2 3 3 2 3
1 2 3 1 2 3
j i j i
j i n j j i
n j n j
    
 
        
 
      
 
  
 
, vô lí.
Vậy giả sử sai, ta có điều chứng minh.
... ...
...
A1
A2 Ai
Aj
An-1
An
CuuDuongThanCong.com https://fb.com/tailieudientucntt
---------------------------------------------------------------------------------------------------------------------------
40
Bài tập tham khảo
Bài 1. Giả sử có 2014 thành phố mà từ mỗi thành phố đều có đường bay trực
tiếp đến đúng 3 thành phố khác đồng thời có thể đi từ 1 thành phố đến thành
phố bất kì bằng các đường bay liên tiếp. Chứng minh tồn tại 202 thành phố đôi
một không có đường bay trực tiếp đến nhau sao cho nếu các sân bay tại các
thành phố này ngừng hoạt động thì người ta vẫn có thể di chuyển giữa các thành
phố còn lại bằng máy bay.
Bài 2. Ở một quốc gia có 5 hãng hàng không hoạt động. Biết rằng từ mỗi thành
phố có đúng 5 đường bay trực tiếp đến 5 thành phố khác và mỗi 2 đường bay
khác nhau thuộc 2 hãng hàng không khác nhau. Người ta có thể đi từ 1 thành
phố đến thành phố khác bằng máy bay. Chứng minh khi hủy 4 đường bay tùy ý
mà không có 2 đường bay thuộc cùng một hãng hàng không thì người ta vẫn có
thể đi lại giữa các thành phố bằng máy bay.
Bài 3. Trong 1 cuộc thi có 25 bạn nam và 1 số bạn nữ. Biết rằng với mỗi nhóm
m bạn nam  10m  đều có đúng 1m  bạn nữ mà mỗi bạn nữ quen ít nhất 1
bạn nam trong nhóm này. Chứng minh tồn tại 1 bạn nữ quen với ít nhất 16 bạn
nam trong cuộc thi.
Bài 4. Cho số nguyên dương 3n  và đồ thị đơn vô hướng có n đỉnh thỏa mãn
điều kiện: Với mỗi 2 k n  , số cạnh của 1 đồ thị con bất kì k đỉnh của G
không vượt quá 2 2k  . Chứng minh có thể tô màu tất cả các cạnh, mỗi cạnh
bởi 1 trong 2 màu xanh đỏ mà không tồn tại chu trình mà các cạnh được tô cùng
màu.
Bài 5 (IMO Shortlist 2002, C7). Trong nhóm có 120 người, một số người là bạn
của nhau. Một nhóm được gọi là bộ bốn yếu là nhóm có 4 người chứa đúng 1
cặp là bạn của nhau. Tìm giá trị lớn nhất số bộ bốn yếu.
Bài 6. Cho số nguyên 3n  . Trong không gian cho n điểm mà không có 3 điểm
nào thẳng hàng và không có 4 điểm nào đồng phẳng. Tại mỗi điểm ghi 1 số
nguyên dương sao cho không có 2 điểm nào được ghi bởi cùng 1 số và n số
được ghi là n số nguyên dương đầu tiên. Gọi điểm được ghi số i là điểm i. Nối
tất cả các cặp điểm  ,p q với p q bởi 1 mũi tên từ p đến q. Tô mỗi mũi tên
bởi 1 trong 2 màu xanh, đỏ. Một cách tô màu được gọi là “tốt” nếu tồn tại 2
điểm j, k sao cho có thể đi từ j đến k qua các mũi tên xanh và cũng có thể đi từ j
đến k qua các mũi tên đỏ. Hỏi có tất cả bao nhiêu cách tô màu “tốt”.
CuuDuongThanCong.com https://fb.com/tailieudientucntt
---------------------------------------------------------------------------------------------------------------------------
41
Bài 7 (Poland 2000). Cho số nguyên dương 2n  . Tìm số k nhỏ nhất có tính
chất: với bất kì tập k ô đơn vị của bảng n n luôn tồn tại 1 tập con khác rỗng A
sao cho trên mỗi dòng, trên mỗi cột có chẵn ô đơn vị trong A.
Bài 8 (Thụy Điển 2010). Một thành phố có 3n công dân. Hai người bất kì trong
thành phố có ít nhất 1 người bạn chung trong thành phố này. Chứng minh có thể
chọn một nhóm có n người sao cho bất kì người nào trong số 2n người còn lại
có người quen trong nhóm n người.
Bài 9 (Generalization of USAMO 2007). Xét graph liên thông G với V đỉnh,
mỗi đỉnh có bậc nhiều nhất là d. Chứng minh G có thể phân hoạch thành 2
graph con liên thông, mỗi graph có ít nhất
1V
d

đỉnh.
Bài 10 (Hefetz, Krivelevich, Stojakovic, Szabo). Xét số nguyên dương d và
graph G mà mỗi đỉnh đều có bậc d. Maker và Breaker chơi 1 trò chơi với cạnh
của G. Hai người lần lượt, mỗi bước chọn đúng 1 cạnh của graph và dời cạnh
này, Breaker là người đi đầu tiên. Trò chơi kết thúc khi tất cả các cạnh được
chọn. Maker thắng nếu sau khi kết thúc trò chơi, với mỗi đỉnh anh ấy chọn được
ít nhất
4
d 
  
cạnh chứa đỉnh này, ngược lại anh ấy thua cuộc. Chứng minh
Maker có chiến thuật thắng.
Bài 11. Người ta muốn mời một số em học sinh tới dự một buổi gặp mặt, mà
trong số đó mỗi em chưa quen với ít nhất là 56 em khác và với mỗi cặp hai em
chưa quen nhau thì đều có ít nhất một em quen với cả hai em đó. Hỏi số học
sinh được mời dự buổi gặp mặt nói trên có thể là 65 em được hay không?
Bài 12 (IMOSL 2015, C7). Trong một công ty có một số người là đối thủ của
nhau. Một nhóm người được gọi là khó gần nếu số người trong nhóm này là lẻ
và nhóm có ít nhất 3 người, hơn nữa có thể xếp những người trong nhóm quanh
bàn tròn sao cho hai người kề nhau là đối thủ của nhau. Biết rằng có nhiều nhất
2015 nhóm khó gần, chứng minh có thể chia công ty này thành 11 nhóm sao
cho không có 2 đối thủ trong cùng 1 nhóm.
Bài 13 (China TST 2011). Xét graph G có  2
3 1n n  đỉnh và không có đỉnh
nào có bậc lớn hơn 4n. Giả sử tồn tại 1 đỉnh bậc 1 và với 2 đỉnh bất kì luôn tồn
CuuDuongThanCong.com https://fb.com/tailieudientucntt
---------------------------------------------------------------------------------------------------------------------------
42
tại đường đi có độ dài không vượt quá 3 nối chúng. Chứng minh G có ít nhất
2
7 3
2
n n
cạnh.
Bài 14 (IMO Shortlist 2013, C6). Trong graph G, mỗi đỉnh v có nhiều nhất 2k
đỉnh có khoảng cách 3 đến nó. Chứng minh với bất kì đỉnh u, tồn tại nhiều nhất
 1k k  đỉnh mà khoảng cách đến nó là 4.
Bài 15 (IMO Shortlist 2004, C3). Xét số nguyên 4n  và 1 graph đầy đủ có n
đỉnh. Thực hiện công việc như sau: mỗi bước chọn bất kì chu trình có độ dài 4
(nếu tồn tại), chọn cạnh tùy ý trong chu trình này và xóa cạnh này. Quá trình
dừng lại khi không còn chu trình 4. Tìm số cạnh nhỏ nhất có thể bị xóa.
CuuDuongThanCong.com https://fb.com/tailieudientucntt
---------------------------------------------------------------------------------------------------------------------------
43
Graph hai phần
Định nghĩa : Graph  ,G V E được gọi là 2 phần nếu tập đỉnh có thể phân
hoạch thành 2 tập con khác rỗng A, B sao cho không có cạnh nối 2 đỉnh trong
cùng 1 tập.
Định lý : Một graph là hai phần nếu và chỉ nếu nó không có chu trình lẻ.
Chứng minh
Giả sử  , ,G V E V A B  là 1 graph 2 phần. Nếu có chu trình thì độ dài của
chu trình phải là chẵn. Thật vậy, giả sử chu trình 1 2 1... kv v v v thì nếu có 1 cạnh nối
1 đỉnh trong A với 1 đỉnh trong B thì ngay sau đó có 1 cạnh nối 1 đỉnh trong B
với 1 đỉnh trong A.
Giả sử G không có chu trình độ dài lẻ.
Ta có thể giả sử G là liên thông, vì nếu G không liên thông thì nó là hợp của các
thành phần liên thông rời nhau, khi đó ta thực hiện với từng thành phần liên
thông và ghép chúng lại.
Gọi A là 1 đỉnh bất kì của G. Xét 2 tập X, Y tương ứng chứa các đỉnh mà độ dài
tới A là chẵn, lẻ tương ứng. Suy ra X, Y chính là 1 phân hoạch của tập đỉnh V.
Ta chứng minh mỗi tập X, Y các đỉnh là độc lập.
Giả sử có 2 đỉnh B và C trong X được nối với nhau, ta xét đường đi ngắn nhất
nối A, B sau đó nối B, C, cuối cùng là đường đi ngắn nhất nối C, A (có thể các
đường đi có đỉnh lặp hoặc cạnh trùng thì ta sẽ loại bỏ đỉnh, cạnh đó). Khi đó có
chu trình có độ dài lẻ, mâu thuẫn. Tương tự với tập Y.
Vậy ta có điều chứng minh.
Bài 1 (ShortlistIMO 1983 C1). Một đất nước có 1983 thành phố, mỗi cặp 2
thành phố có một đường đi nối chúng. Mỗi đường đi thuộc quản lí của đúng 1
trong 10 công ty. Chứng minh tồn tại một tour du lịch theo vòng khép kín qua lẻ
con đường mà các con đường này thuộc quyền quản lí của một công ty.
Giải
Xét graph G có 1983 đỉnh ứng với 1983 thành phố, 2 đỉnh được nối với nhau và
tô cạnh đó bởi 1 trong 10 màu 1, 2, …, 10 ứng với 1 trong 10 hãng hàng không.
CuuDuongThanCong.com https://fb.com/tailieudientucntt
---------------------------------------------------------------------------------------------------------------------------
44
Giả sử phản chứng không có một tour du lịch theo vòng khép kín qua lẻ con
đường mà các con đường này thuộc quyền quản lí của một công ty, nghĩa là
graph G không có chu trình lẻ khi xét các đỉnh và các cạnh cùng màu. Suy ra
khi xét các đỉnh và các cạnh cùng màu thì G là 2 phần.
Xét các đỉnh và các cạnh được tô màu 1 thì G là graph 2 phần, có 1 phần có ít
nhất
1983
1 992
2
 
   
đỉnh, đặt đó là 1G .
Trong 1G chỉ xét các cạnh được tô màu 2, đây là graph 2 phần, có 1 phần có ít
nhất
992
496
2
 
  
đỉnh, đặt là 2G .
Tiếp tục quá trình có các graph 3G có ít nhất
496
248
2
 
  
đỉnh, graph 4G có ít
nhất
248
124
2
 
  
đỉnh, graph 5G có ít nhất
124
62
2
 
  
đỉnh, graph 6G có ít
nhất
62
31
2
 
  
đỉnh, graph 7G có ít nhất
31
1 16
2
 
   
đỉnh, graph 8G có ít nhất
16
8
2
 
  
đỉnh, graph 9G có ít nhất 4 đỉnh, graph 10G có ít nhất 2 đỉnh.
Cần chú ý rằng, các đỉnh trong 10G không được nối với nhau bởi bất kì cạnh nào
vì nếu nối bởi 1 cạnh thì cạnh này được tô màu 1 màu i, khi đó trái với 1iG  là 2
phần khi ta xét các đỉnh trong 1iG  và các cạnh được tô màu i.
Bài 2 (IMC 1999). Trong mặt phẳng tọa độ cho n đường thẳng song song với
Ox (ta gọi là các dòng) và n đường thẳng song song với Oy (ta gọi là các cột).
Xét 2n giao điểm được tạo ra bởi các đường thẳng trên. Chứng minh tồn tại
 2 2k k n  điểm 1 2 2, ,..., ka a a trong 2n điểm trên thỏa mãn với mỗi
1,2,..., 1i k  : 2 1ia  và 2ia thuộc cùng 1 dòng nhưng 2ia và 2 1ia  thuộc cùng 1
cột.
Giải
Xét graph 2 phần G A B  , với các đỉnh trong A tương ứng các dòng và các
đỉnh trong B tương ứng các cột. Cặp điểm được nối với nhau nếu có 1 điểm
được xét là giao của dòng và cột tương ứng. Nếu 2 cạnh có cùng 1 đầu mút thì 2
CuuDuongThanCong.com https://fb.com/tailieudientucntt
Ly thuyet-do-thi-va-ung-dung - [cuuduongthancong.com]
Ly thuyet-do-thi-va-ung-dung - [cuuduongthancong.com]
Ly thuyet-do-thi-va-ung-dung - [cuuduongthancong.com]
Ly thuyet-do-thi-va-ung-dung - [cuuduongthancong.com]
Ly thuyet-do-thi-va-ung-dung - [cuuduongthancong.com]
Ly thuyet-do-thi-va-ung-dung - [cuuduongthancong.com]
Ly thuyet-do-thi-va-ung-dung - [cuuduongthancong.com]

More Related Content

What's hot

Dùng cây đồ thị trong dạy học toán tiểu học
Dùng cây đồ thị trong dạy học toán tiểu học Dùng cây đồ thị trong dạy học toán tiểu học
Dùng cây đồ thị trong dạy học toán tiểu học nataliej4
 
CHƯƠNG 3 MÔ HÌNH HÓA CÁC PHẦN TỬ TRONGHỆ THỐNGĐIỆN
CHƯƠNG 3  MÔ HÌNH HÓA CÁC PHẦN TỬ TRONGHỆ THỐNGĐIỆNCHƯƠNG 3  MÔ HÌNH HÓA CÁC PHẦN TỬ TRONGHỆ THỐNGĐIỆN
CHƯƠNG 3 MÔ HÌNH HÓA CÁC PHẦN TỬ TRONGHỆ THỐNGĐIỆNĐinh Công Thiện Taydo University
 
[Vnmath.com] phan-dang-va-100-bai-tap-toa-do-trong-matphang
[Vnmath.com] phan-dang-va-100-bai-tap-toa-do-trong-matphang[Vnmath.com] phan-dang-va-100-bai-tap-toa-do-trong-matphang
[Vnmath.com] phan-dang-va-100-bai-tap-toa-do-trong-matphangHuynh ICT
 
Giaitichmang
GiaitichmangGiaitichmang
GiaitichmangGara Mít
 
Chuong6 pdf-luoi khongche trac dia
Chuong6 pdf-luoi khongche trac diaChuong6 pdf-luoi khongche trac dia
Chuong6 pdf-luoi khongche trac diathai lehong
 
Chuyên đề 1. tính đơn điệu của hàm số câu hỏi
Chuyên đề 1. tính đơn điệu của hàm số   câu hỏiChuyên đề 1. tính đơn điệu của hàm số   câu hỏi
Chuyên đề 1. tính đơn điệu của hàm số câu hỏiLongV86
 
báo cáo môn dạy học lập trình
báo cáo môn dạy học lập trìnhbáo cáo môn dạy học lập trình
báo cáo môn dạy học lập trìnhkimngan2203
 
Bài 5 khảo sát sự biến thiên & vẽ đồ thị hàm số (1)
Bài 5   khảo sát sự biến thiên & vẽ đồ thị hàm số (1)Bài 5   khảo sát sự biến thiên & vẽ đồ thị hàm số (1)
Bài 5 khảo sát sự biến thiên & vẽ đồ thị hàm số (1)LongV86
 
Tập 3 chuyên đề Toán học: Hình học phẳng Oxy - Megabook.vn
Tập 3 chuyên đề Toán học: Hình học phẳng Oxy - Megabook.vnTập 3 chuyên đề Toán học: Hình học phẳng Oxy - Megabook.vn
Tập 3 chuyên đề Toán học: Hình học phẳng Oxy - Megabook.vnMegabook
 
Phieu so 02 khao sat ham so
Phieu so 02 khao sat ham soPhieu so 02 khao sat ham so
Phieu so 02 khao sat ham soLongV86
 
Cac chuyen de on thi tot nghiep 2013 [giasuductri.edu.vn]]
Cac chuyen de on thi tot nghiep 2013 [giasuductri.edu.vn]]Cac chuyen de on thi tot nghiep 2013 [giasuductri.edu.vn]]
Cac chuyen de on thi tot nghiep 2013 [giasuductri.edu.vn]]Gia sư Đức Trí
 
Chuyen de ltdh toan tuyen tap 2013
Chuyen de ltdh toan  tuyen tap  2013Chuyen de ltdh toan  tuyen tap  2013
Chuyen de ltdh toan tuyen tap 2013Lê Hà
 
08 bai toan lap pt mat phang p1
08 bai toan lap pt mat phang p108 bai toan lap pt mat phang p1
08 bai toan lap pt mat phang p1Huynh ICT
 
chuyen de giai tich trong mat phang on luyen thi dai hoc cho hoc sinh va gia ...
chuyen de giai tich trong mat phang on luyen thi dai hoc cho hoc sinh va gia ...chuyen de giai tich trong mat phang on luyen thi dai hoc cho hoc sinh va gia ...
chuyen de giai tich trong mat phang on luyen thi dai hoc cho hoc sinh va gia ...Hoàng Thái Việt
 

What's hot (17)

Dùng cây đồ thị trong dạy học toán tiểu học
Dùng cây đồ thị trong dạy học toán tiểu học Dùng cây đồ thị trong dạy học toán tiểu học
Dùng cây đồ thị trong dạy học toán tiểu học
 
Ltdt chuong 3
Ltdt chuong 3Ltdt chuong 3
Ltdt chuong 3
 
CHƯƠNG 3 MÔ HÌNH HÓA CÁC PHẦN TỬ TRONGHỆ THỐNGĐIỆN
CHƯƠNG 3  MÔ HÌNH HÓA CÁC PHẦN TỬ TRONGHỆ THỐNGĐIỆNCHƯƠNG 3  MÔ HÌNH HÓA CÁC PHẦN TỬ TRONGHỆ THỐNGĐIỆN
CHƯƠNG 3 MÔ HÌNH HÓA CÁC PHẦN TỬ TRONGHỆ THỐNGĐIỆN
 
[Vnmath.com] phan-dang-va-100-bai-tap-toa-do-trong-matphang
[Vnmath.com] phan-dang-va-100-bai-tap-toa-do-trong-matphang[Vnmath.com] phan-dang-va-100-bai-tap-toa-do-trong-matphang
[Vnmath.com] phan-dang-va-100-bai-tap-toa-do-trong-matphang
 
Ltdt chuong 3 (2)
Ltdt chuong 3 (2)Ltdt chuong 3 (2)
Ltdt chuong 3 (2)
 
Giaitichmang
GiaitichmangGiaitichmang
Giaitichmang
 
Chuong6 pdf-luoi khongche trac dia
Chuong6 pdf-luoi khongche trac diaChuong6 pdf-luoi khongche trac dia
Chuong6 pdf-luoi khongche trac dia
 
Chuyên đề 1. tính đơn điệu của hàm số câu hỏi
Chuyên đề 1. tính đơn điệu của hàm số   câu hỏiChuyên đề 1. tính đơn điệu của hàm số   câu hỏi
Chuyên đề 1. tính đơn điệu của hàm số câu hỏi
 
báo cáo môn dạy học lập trình
báo cáo môn dạy học lập trìnhbáo cáo môn dạy học lập trình
báo cáo môn dạy học lập trình
 
Bài 5 khảo sát sự biến thiên & vẽ đồ thị hàm số (1)
Bài 5   khảo sát sự biến thiên & vẽ đồ thị hàm số (1)Bài 5   khảo sát sự biến thiên & vẽ đồ thị hàm số (1)
Bài 5 khảo sát sự biến thiên & vẽ đồ thị hàm số (1)
 
Tập 3 chuyên đề Toán học: Hình học phẳng Oxy - Megabook.vn
Tập 3 chuyên đề Toán học: Hình học phẳng Oxy - Megabook.vnTập 3 chuyên đề Toán học: Hình học phẳng Oxy - Megabook.vn
Tập 3 chuyên đề Toán học: Hình học phẳng Oxy - Megabook.vn
 
Tổng Hợp Các Công Thức Toán 10-11-12
Tổng Hợp Các Công Thức Toán 10-11-12Tổng Hợp Các Công Thức Toán 10-11-12
Tổng Hợp Các Công Thức Toán 10-11-12
 
Phieu so 02 khao sat ham so
Phieu so 02 khao sat ham soPhieu so 02 khao sat ham so
Phieu so 02 khao sat ham so
 
Cac chuyen de on thi tot nghiep 2013 [giasuductri.edu.vn]]
Cac chuyen de on thi tot nghiep 2013 [giasuductri.edu.vn]]Cac chuyen de on thi tot nghiep 2013 [giasuductri.edu.vn]]
Cac chuyen de on thi tot nghiep 2013 [giasuductri.edu.vn]]
 
Chuyen de ltdh toan tuyen tap 2013
Chuyen de ltdh toan  tuyen tap  2013Chuyen de ltdh toan  tuyen tap  2013
Chuyen de ltdh toan tuyen tap 2013
 
08 bai toan lap pt mat phang p1
08 bai toan lap pt mat phang p108 bai toan lap pt mat phang p1
08 bai toan lap pt mat phang p1
 
chuyen de giai tich trong mat phang on luyen thi dai hoc cho hoc sinh va gia ...
chuyen de giai tich trong mat phang on luyen thi dai hoc cho hoc sinh va gia ...chuyen de giai tich trong mat phang on luyen thi dai hoc cho hoc sinh va gia ...
chuyen de giai tich trong mat phang on luyen thi dai hoc cho hoc sinh va gia ...
 

Similar to Ly thuyet-do-thi-va-ung-dung - [cuuduongthancong.com]

Tom tat bai giang ly thuyet do thi - nguyen ngoc trung
Tom tat bai giang   ly thuyet do thi - nguyen ngoc trungTom tat bai giang   ly thuyet do thi - nguyen ngoc trung
Tom tat bai giang ly thuyet do thi - nguyen ngoc trungPhi Phi
 
Luận văn: Lý thuyết đồ thị với các bài toán phổ thông, HOT - Gửi miễn phí qua...
Luận văn: Lý thuyết đồ thị với các bài toán phổ thông, HOT - Gửi miễn phí qua...Luận văn: Lý thuyết đồ thị với các bài toán phổ thông, HOT - Gửi miễn phí qua...
Luận văn: Lý thuyết đồ thị với các bài toán phổ thông, HOT - Gửi miễn phí qua...Dịch vụ viết bài trọn gói ZALO: 0909232620
 
Chde ltdh-mon-toan-mathvn.com-2013
Chde ltdh-mon-toan-mathvn.com-2013Chde ltdh-mon-toan-mathvn.com-2013
Chde ltdh-mon-toan-mathvn.com-2013Huynh ICT
 
Chuyên Đề LTĐH Toán 2013 - Biên Soạn VNMath
Chuyên Đề LTĐH Toán 2013 - Biên Soạn VNMathChuyên Đề LTĐH Toán 2013 - Biên Soạn VNMath
Chuyên Đề LTĐH Toán 2013 - Biên Soạn VNMathHải Finiks Huỳnh
 
[Vnmath.com] chuyen de on thi dai hoccao dang mon toan nam2013
[Vnmath.com] chuyen de on thi dai hoccao dang mon toan nam2013[Vnmath.com] chuyen de on thi dai hoccao dang mon toan nam2013
[Vnmath.com] chuyen de on thi dai hoccao dang mon toan nam2013Huynh ICT
 
[Vnmath.com] chuyen de on thi dai hoccao dang mon toan nam2013
[Vnmath.com] chuyen de on thi dai hoccao dang mon toan nam2013[Vnmath.com] chuyen de on thi dai hoccao dang mon toan nam2013
[Vnmath.com] chuyen de on thi dai hoccao dang mon toan nam2013Huynh ICT
 
Huong dan 8 o so
Huong dan 8 o soHuong dan 8 o so
Huong dan 8 o soshjdunglv
 
Chde ltdh-mon-toan-2013
Chde ltdh-mon-toan-2013Chde ltdh-mon-toan-2013
Chde ltdh-mon-toan-2013phanhungvinh
 
Chuyên đề oxy thầy đặng thành nam
Chuyên đề oxy thầy đặng thành namChuyên đề oxy thầy đặng thành nam
Chuyên đề oxy thầy đặng thành namOn thi
 
GIÁO ÁN CHUYÊN ĐỀ TOÁN 11 CẢ NĂM (CÁNH DIỀU) SOẠN THEO CÔNG VĂN 5512 (2 CỘT) ...
GIÁO ÁN CHUYÊN ĐỀ TOÁN 11 CẢ NĂM (CÁNH DIỀU) SOẠN THEO CÔNG VĂN 5512 (2 CỘT) ...GIÁO ÁN CHUYÊN ĐỀ TOÁN 11 CẢ NĂM (CÁNH DIỀU) SOẠN THEO CÔNG VĂN 5512 (2 CỘT) ...
GIÁO ÁN CHUYÊN ĐỀ TOÁN 11 CẢ NĂM (CÁNH DIỀU) SOẠN THEO CÔNG VĂN 5512 (2 CỘT) ...Nguyen Thanh Tu Collection
 
05 mat102-bai 2-v1.0
05 mat102-bai 2-v1.005 mat102-bai 2-v1.0
05 mat102-bai 2-v1.0Yen Dang
 
báo-cáo-mô-hình-hóa-nhóm-15.pdf
báo-cáo-mô-hình-hóa-nhóm-15.pdfbáo-cáo-mô-hình-hóa-nhóm-15.pdf
báo-cáo-mô-hình-hóa-nhóm-15.pdfngTrnh17
 

Similar to Ly thuyet-do-thi-va-ung-dung - [cuuduongthancong.com] (20)

Luận văn: Mô hình đồ thị luồng và mối quan hệ với đồ thị, HAY
Luận văn: Mô hình đồ thị luồng và mối quan hệ với đồ thị, HAYLuận văn: Mô hình đồ thị luồng và mối quan hệ với đồ thị, HAY
Luận văn: Mô hình đồ thị luồng và mối quan hệ với đồ thị, HAY
 
CHƯƠNG 6.pdf
CHƯƠNG 6.pdfCHƯƠNG 6.pdf
CHƯƠNG 6.pdf
 
Tom tat bai giang ly thuyet do thi - nguyen ngoc trung
Tom tat bai giang   ly thuyet do thi - nguyen ngoc trungTom tat bai giang   ly thuyet do thi - nguyen ngoc trung
Tom tat bai giang ly thuyet do thi - nguyen ngoc trung
 
Ctdlgt
CtdlgtCtdlgt
Ctdlgt
 
Ctdlgt
CtdlgtCtdlgt
Ctdlgt
 
Luận văn: Lý thuyết đồ thị với các bài toán phổ thông, HOT - Gửi miễn phí qua...
Luận văn: Lý thuyết đồ thị với các bài toán phổ thông, HOT - Gửi miễn phí qua...Luận văn: Lý thuyết đồ thị với các bài toán phổ thông, HOT - Gửi miễn phí qua...
Luận văn: Lý thuyết đồ thị với các bài toán phổ thông, HOT - Gửi miễn phí qua...
 
Chde ltdh-mon-toan-mathvn.com-2013
Chde ltdh-mon-toan-mathvn.com-2013Chde ltdh-mon-toan-mathvn.com-2013
Chde ltdh-mon-toan-mathvn.com-2013
 
Chuyên đề ôn thi đại học môn toán học
Chuyên đề ôn thi đại học môn toán họcChuyên đề ôn thi đại học môn toán học
Chuyên đề ôn thi đại học môn toán học
 
Bài Toán Phân Luồng Giao Thông Và Ứng Dụng.doc
Bài Toán Phân Luồng Giao Thông Và Ứng Dụng.docBài Toán Phân Luồng Giao Thông Và Ứng Dụng.doc
Bài Toán Phân Luồng Giao Thông Và Ứng Dụng.doc
 
Luận văn: Chu kỳ của Chip-firing game song song trên đồ thị, 9đ
Luận văn: Chu kỳ của Chip-firing game song song trên đồ thị, 9đLuận văn: Chu kỳ của Chip-firing game song song trên đồ thị, 9đ
Luận văn: Chu kỳ của Chip-firing game song song trên đồ thị, 9đ
 
Chuyên Đề LTĐH Toán 2013 - Biên Soạn VNMath
Chuyên Đề LTĐH Toán 2013 - Biên Soạn VNMathChuyên Đề LTĐH Toán 2013 - Biên Soạn VNMath
Chuyên Đề LTĐH Toán 2013 - Biên Soạn VNMath
 
[Vnmath.com] chuyen de on thi dai hoccao dang mon toan nam2013
[Vnmath.com] chuyen de on thi dai hoccao dang mon toan nam2013[Vnmath.com] chuyen de on thi dai hoccao dang mon toan nam2013
[Vnmath.com] chuyen de on thi dai hoccao dang mon toan nam2013
 
[Vnmath.com] chuyen de on thi dai hoccao dang mon toan nam2013
[Vnmath.com] chuyen de on thi dai hoccao dang mon toan nam2013[Vnmath.com] chuyen de on thi dai hoccao dang mon toan nam2013
[Vnmath.com] chuyen de on thi dai hoccao dang mon toan nam2013
 
Huong dan 8 o so
Huong dan 8 o soHuong dan 8 o so
Huong dan 8 o so
 
Chde ltdh-mon-toan-2013
Chde ltdh-mon-toan-2013Chde ltdh-mon-toan-2013
Chde ltdh-mon-toan-2013
 
Chuyên đề oxy thầy đặng thành nam
Chuyên đề oxy thầy đặng thành namChuyên đề oxy thầy đặng thành nam
Chuyên đề oxy thầy đặng thành nam
 
GIÁO ÁN CHUYÊN ĐỀ TOÁN 11 CẢ NĂM (CÁNH DIỀU) SOẠN THEO CÔNG VĂN 5512 (2 CỘT) ...
GIÁO ÁN CHUYÊN ĐỀ TOÁN 11 CẢ NĂM (CÁNH DIỀU) SOẠN THEO CÔNG VĂN 5512 (2 CỘT) ...GIÁO ÁN CHUYÊN ĐỀ TOÁN 11 CẢ NĂM (CÁNH DIỀU) SOẠN THEO CÔNG VĂN 5512 (2 CỘT) ...
GIÁO ÁN CHUYÊN ĐỀ TOÁN 11 CẢ NĂM (CÁNH DIỀU) SOẠN THEO CÔNG VĂN 5512 (2 CỘT) ...
 
05 mat102-bai 2-v1.0
05 mat102-bai 2-v1.005 mat102-bai 2-v1.0
05 mat102-bai 2-v1.0
 
Ctdlgt
CtdlgtCtdlgt
Ctdlgt
 
báo-cáo-mô-hình-hóa-nhóm-15.pdf
báo-cáo-mô-hình-hóa-nhóm-15.pdfbáo-cáo-mô-hình-hóa-nhóm-15.pdf
báo-cáo-mô-hình-hóa-nhóm-15.pdf
 

Ly thuyet-do-thi-va-ung-dung - [cuuduongthancong.com]

  • 1. --------------------------------------------------------------------------------------------------------------------------- 1 Lý thuyết Graph và ứng dụng Một trong những kết quả đầu tiên trong lý thuyết đồ thị (graph) xuất hiện trong bài báo của Leonhard Euler về Bảy cây cầu ở Königsberg, xuất bản năm 1736. Bài báo này cũng được xem như một trong những kết quả topo đầu tiên trong hình học, tức là, nó không hề phụ thuộc vào bất cứ độ đo nào. Nó diễn tả mối liên hệ sâu sắc giữa lý thuyết đồ thị và tôpô học. Năm 1845, Gustav Kirchhoff đưa ra Định luật Kirchhoff cho mạch điện để tính điện thế và cường độ dòng điện trong mạch điện. Năm 1852 Francis Guthrie đưa ra bài toán bốn màu về vấn đề liệu chỉ với bốn màu có thể tô màu một bản đồ bất kì sao cho không có hai nước nào cùng biên giới được tô cùng màu. Bài toán này được xem như đã khai sinh ra lý thuyết đồ thị, và chỉ được giải sau một thế kỉ vào năm 1976 bởi Kenneth Appel và Wolfgang Haken. Trong khi cố gắng giải quyết bài toán này, các nhà toán học đã phát minh ra nhiều thuật ngữ và khái niệm nền tảng cho lý thuyết đồ thị. Năm 1933 nhà toán học Dénes Kőnig đã xuất bản cuốn sách giáo khoa đầu tiên về đồ thị và phát biểu định lí Kőnig rất nổi tiếng. Năm 1959 xuất bản CuuDuongThanCong.com https://fb.com/tailieudientucntt
  • 2. --------------------------------------------------------------------------------------------------------------------------- 2 cuốn sách The Theory of Graphs and its Applications được coi như cuốn sách giáo khoa thứ 2 về vấn đề graph. Mặc dù Lý thuyết đồ thị đã được khoa học phát triển từ rất lâu nhưng lại có nhiều ứng dụng hiện đại. Đặc biệt trong khoảng vài mươi năm trở lại đây, cùng với sự ra đời của máy tính điện tử và sự phát triển nhanh chóng của Tin học, Lý thuyết đồ thị càng được quan tâm đến nhiều hơn. Đặc biệt là các thuật toán trên đồ thị đã có nhiều ứng dụng trong nhiều lĩnh vực khác nhau như: Mạng máy tính, Lý thuyết mã, Tối ưu hoá, Kinh tế học v.v... Chẳng hạn như trả lời câu hỏi: Hai máy tính trong mạng có thể liên hệ được với nhau hay không ?; hay vấn đề phân biệt hai hợp chất hoá học có cùng công thức phân tử nhưng lại khác nhau về công thức cấu tạo cũng được giải quyết nhờ mô hình đồ thị. Hiện nay, môn học này là một trong những kiến thức cơ sở của bộ môn khoa học máy tính. Trong toán học và tin học, lý thuyết đồ thị nghiên cứu các tính chất của đồ thị. Một cách không chính thức, đồ thị là một tập các đối tượng được gọi là các đỉnh (hoặc nút) nối với nhau bởi các cạnh (hoặc cung). Cạnh có thể có hướng hoặc vô hướng. Đồ thị thường được vẽ dưới dạng một tập các điểm (các đỉnh nối với nhau bằng các đoạn thẳng (các cạnh). Đồ thị biểu diễn được rất nhiều cấu trúc, nhiều bài toán thực tế có thể được biểu diễn bằng đồ thị. Ví dụ, cấu trúc liên kết của một website có thể được biểu diễn bằng một đồ thị có hướng như sau: các đỉnh là các trang web hiện có tại website, tồn tại một cạnh có hướng nối từ trang A tới trang B khi và chỉ khi A có chứa 1 liên kết tới B. Do vậy, sự phát triển của các thuật toán xử lý đồ thị là một trong các mối quan tâm chính của khoa học máy tính. Cấu trúc đồ thị có thể được mở rộng bằng cách gán trọng số cho mỗi cạnh. Có thể sử dụng đồ thị có trọng số để biểu diễn nhiều khái niệm khác nhau. Ví dụ, nếu đồ thị biểu diễn một mạng đường giao thông, các trọng số có thể là độ dài của mỗi con đường. Một cách khác để mở rộng đồ thị cơ bản là quy định hướng cho các cạnh của đồ thị (như đối với các trang web, A liên kết tới B, nhưng B không nhất thiết cũng liên kết tới A). Loại đồ thị này được gọi là đồ thị có hướng. Một đồ thị có hướng với các cạnh có trọng số được gọi là một lưới. Trong những năm gần đây, vấn đề đồ thị được đưa vào áp dụng trong nhiều bài toán tổ hợp thi học sinh giỏi (Quốc gia và quốc tế). Theo xu hướng của quốc tế, vấn đề tổ hợp nói chung và vấn đề đồ thị nói riêng cần được quan tâm phát triển mạnh hơn nữa, tạo tiền đề tốt cho học sinh khi tham gia các kì thi học sinh giỏi. Trong chuyên đề này chúng tôi trình bày lý thuyết từ đầu của phương pháp đồ thị (các khái niệm và kết quả) cùng một số phương pháp áp dụng vào giải các bài toán thi học sinh giỏi. CuuDuongThanCong.com https://fb.com/tailieudientucntt
  • 3. --------------------------------------------------------------------------------------------------------------------------- 3 1. Định nghĩa mở đầu Định nghĩa 1. Một graph (hay một đồ thị) là tập các đỉnh và các cạnh nối một số đỉnh với nhau. Kí hiệu  ,G V E với V là tập đỉnh và E V V  là tập cạnh. Ví dụ: Có 11 graph khác nhau với tập đỉnh có 4 phần tử. Biểu diễn của các graph như hình vẽ bên. Định nghĩa 2. Hai đỉnh được gọi là kề nhau nếu có 1 cạnh nối 2 đỉnh này. Kí hiệu cạnh nối 2 đỉnh A, B là  AB , nói chung  AB khác  BA , nếu ta coi 2 cạnh này là 1 thì ta có graph vô hướng, nếu coi chúng khác nhau thì ta có graph có hướng. Có thể tồn tại cạnh nối 1 điểm với chính nó, cạnh này gọi là khuyên. Có thể tồn tại nhiều cạnh nối 2 điểm phân biệt, các cạnh này gọi là cạnh kép. Một graph đơn nếu nó không có khuyên và không có cạnh kép. Các bài toán ta gặp chủ yếu là graph đơn. Định nghĩa 3. Một graph đầy đủ với n đỉnh, kí hiệu là nK , là graph mà 2 đỉnh bất kì đều có cạnh nối giữa chúng, khi đó có tất cả 2 nC cạnh. Ví dụ : 4K và 5K Định nghĩa 4. Với U là tập con của tập các đỉnh, kí hiệu  G U là graph con của G, thu được khi ta xóa tất cả các đỉnh nằm ngoài U, chỉ giữ lại các cạnh mà cả 2 đầu mút thuộc U. Các định nghĩa và kết quả dưới đây có giả thiết là graph đơn và vô hướng. CuuDuongThanCong.com https://fb.com/tailieudientucntt
  • 4. --------------------------------------------------------------------------------------------------------------------------- 4 2. Bậc của đỉnh Định nghĩa 5. Kí hiệu  d v hoặc  deg v cho bậc của đỉnh v, là số cạnh mà v là đầu mút. Một khuyên được tính 2 lần cho đỉnh. Một điểm gọi là chẵn nếu nó có bậc chẵn và được gọi là lẻ nếu nó có bậc lẻ. Ví dụ: Graph bên có      1 2 34, 6, 1,d v d v d v      4 53, 4d v d v  . Kết quả 1. Trong một graph có nhiều hơn 1 đỉnh luôn có 2 đỉnh có cùng bậc. Chứng minh Xét  ,G V E có n đỉnh, khi đó bậc của mỗi đỉnh sẽ là số tự nhiên nhỏ hơn n, hơn nữa không tồn tại 2 đỉnh mà bậc của chúng tương ứng là 0 và 1n  (Có đỉnh bậc 1n  có nghĩa nó nối với tất cả các đỉnh khác nên không còn đỉnh bậc 0). Nếu không tồn tại 2 đỉnh cùng bậc thì bậc của các đỉnh này nhận tất cả các giá trị 0,1,2,..., 1n  , mâu thuẫn với nhận xét trên. Ta có điều chứng minh. Kết quả 2. Trong một Graph vô hướng G tùy ý tổng bậc của tất cả các đỉnh gấp đôi số cạnh của Graph. Chứng minh: Trong mỗi graph tổng bậc các đỉnh của một graph thì mỗi cạnh được tính đúng hai lần bởi hai đỉnh của nó. Do đó tổng này gấp đôi số cạnh của graph Hệ quả 1: Trong một Graph vô hướng G tùy ý số đỉnh bậc lẻ luôn là một số chẵn. Chứng minh: Theo định lý trên thì tổng các bậc của các đỉnh luôn là một số chẵn do vậy số các đỉnh bậc lẻ luôn là một số chẵn. Hệ quả 2: Trong một graph vô hướng G có số lẻ đỉnh luôn có một số lẻ các đỉnh có bậc chẵn. Chứng minh: Theo hệ quả trên thì số đỉnh bậc lẻ trong graph G là một số chẵn. Do trong graph G có số lẻ đỉnh, nên số các đỉnh bậc chẵn phải là số lẻ. CuuDuongThanCong.com https://fb.com/tailieudientucntt
  • 5. --------------------------------------------------------------------------------------------------------------------------- 5 Ví dụ: Trong bữa tiệc có 51 người. Khi đó: 1. Có 1 người quen với chẵn người khác trong bữa tiệc; 2. Có 2 người có cùng số người quen; 3. Nếu mỗi người tính số người quen của mình trong bữa tiệc thì tổng của các số này là số chẵn. 3. Đường đi, chu trình trong graph, graph liên thông. Định nghĩa 6. Đường đi trong graph là một dãy các cạnh liên tiếp (hai cạnh liên tiếp nếu chúng có chung đỉnh). Với đồ thị đơn, nếu đường đi đi qua các đỉnh 1 2, ,..., nv v v theo thứ tự thì ta kí hiệu  1 2, ,..., nv v v , nếu đường đi đi qua các cạnh 1 2, ,.., ne e e theo thứ tự thì ta kí hiệu  1 2, ,.., ne e e . Ví dụ với graph bên, ta có đường đi  1 2 6 5 3, , , ,v v v v v , đường đi này có thể kí hiệu là  1 6 5 4, , ,e e e e . Độ dài của đường đi là số cạnh của đường đi này. Ví dụ: Đường đi ở ví dụ trên có độ dài là 4. Khoảng cách giữa 2 đỉnh a và b là độ dài của đường đi ngắn nhất nối 2 đỉnh này, kí hiệu  ,d a b . Quy ước  , 0d a a  . Nếu không có đường đi nối a, b thì quy ước  ,d a b   . Ví dụ: Ở graph trên ta có    1 3 3 4, 4, , 1d v v d v v  . Đường kính của graph G là khoảng cách lớn nhất giữa 2 đỉnh của graph, kí hiệu  d G . Nếu graph có 2 điểm a, b mà  ,d a b   thì quy ước  d G  . Ví dụ: Graph trên có đường kính là 4. Định nghĩa 7. Graph gọi là liên thông nếu với 2 đỉnh bất kì luôn tìm được 1 đường đi nối chúng. Kết quả 3. Với  ,G V E là liên thông thì 1E V  . Chứng minh e2 e3 e5 e6 e1 e4 v3 v4 v5 v6v2 v1 CuuDuongThanCong.com https://fb.com/tailieudientucntt
  • 6. --------------------------------------------------------------------------------------------------------------------------- 6 Ta chứng minh quy nạp theo số đỉnh n của đồ thị. Với 1,2n  ta thấy điều cần chứng minh. Giả sử bài toán đúng với graph N đỉnh, nghĩa là có ít nhất 1N  cạnh. Xét  ,G V E có 1N  đỉnh và liên thông, suy ra bậc của mỗi đỉnh đều lớn hơn 0. Ta xét 2 trường hợp Trường hợp 1: Bậc của mỗi đỉnh đều lớn hơn 1. Ta có  2 2 A V E d A N    , suy ra E N , có điều chứng minh. Trường hợp 2 : Giả sử có đỉnh A của đồ thị có bậc 1, xét graph con  'G G A  (bỏ A và cạnh mà A là đầu mút). Dễ thấy 'G là liên thông và có N đỉnh và 1E  cạnh. Theo giả thiết quy nạp với 'G ta có  1 1 1 1E N E N       , có điều chứng minh. Kết quả 4. Các đỉnh có thể phân hoạch thành các tập 1 2, ,..., rV V V mà các graph con  iG V là liên thông và không có cạnh nào nối cặp điểm ở 2 tập khác nhau. Chúng được gọi là các thành phần liên thông của G. Định nghĩa 8. Chu trình của đồ thị là 1 đường đi đóng (điểm đầu và điểm cuối trùng nhau). Độ dài của chu trình là số cạnh trong chu trình này. Định nghĩa 9. Đường đi Ole là 1 đường đi qua tất cả các cạnh, mỗi cạnh đúng 1 lần. Ví dụ trong graph bên, đường đi 1, 2, 3, 4, 5, 6, 7 là 1 đường đi Ole. Đường đi này có thể đi qua 1 đỉnh nhiều lần, cần chú ý điều này để so sánh với đường đi Hamilton ở phần sau. Định nghĩa 10. Đường đi Ole được gọi là chu trình Ole nếu điểm đầu và điểm cuối trùng nhau. Ví dụ graph bên có đường đi 1, 2, 3, 4, 5, 6, 7, 8 là chu trình Ole. Kết quả 5. Một graph có chu trình Ole thì tất cả các đỉnh là chẵn. Chứng minh CuuDuongThanCong.com https://fb.com/tailieudientucntt
  • 7. --------------------------------------------------------------------------------------------------------------------------- 7 Chú ý rằng, tại mỗi đỉnh nếu có cạnh vào thì sẽ có cạnh ra nên bậc luôn là chẵn. Kết quả 6. Graph liên thông bất kì với tất cả các đỉnh là chẵn có 1 chu trình Ole. Chứng minh Bắt đầu từ đỉnh 1v không lặp lại bất kì cạnh nào và chú ý rằng do điều kiện bậc mỗi đỉnh đều chẵn, đường đi này sẽ kết thúc ở 1v , đây là 1 chu trình. Nếu vẫn còn cạnh chưa đi qua, xét đỉnh 2v thuộc đường đi trên và là đầu mút của 1 cạnh chưa đi qua, xét đường đi bắt đầu từ 2v và qua cạnh chưa sử dụng này, đường đi này cũng sẽ kết thúc ở 2v . Liên hết 2 đường đi này tại đỉnh 2v , nếu chưa được chu trình thì tiếp tục quá trình. Ta có điều cần chứng minh. Kết quả 7. Nếu 1 graph có đường đi Ole thì có nhiều nhất 2 đỉnh lẻ. Chứng minh Tương tự như kết quả 5. Kết quả 8. Một graph liên thông có đúng 2 đỉnh lẻ chứa 1 đường đi Ole. Chứng minh Nối 1 cạnh mới giữa 2 đỉnh lẻ, khi đó tất cả các đỉnh là chẵn. Theo kết quả trên, graph này có chu trình Ole. Xét chu trình bắt đầu từ 1 đỉnh lẻ lúc đầu, cạnh mới thêm đi qua sau cùng. Khi đó nếu xóa cạnh mới thêm này thì chu trình này trở thành đường đi Ole. Ta có điều chứng minh. 4. Cây Định nghĩa 11. Một rừng là 1 graph không nhất thiết liên thông và không có chu trình. Định nghĩa 12. Một cây là 1 graph liên thông và không có chu trình. Kết quả 9. Một cây bất kì luôn chứa đỉnh có bậc 1, đỉnh này được gọi là lá. Chứng minh Giả sử tất cả các đỉnh đều có bậc không nhỏ hơn 2. Xét đường đi bất kì   1 2, ,..., 2nv v v n  . Nếu nv nối với 1 trong các đỉnh 1 2,..., nv v  thì ta có 1 chu trình, mâu thuẫn. CuuDuongThanCong.com https://fb.com/tailieudientucntt
  • 8. --------------------------------------------------------------------------------------------------------------------------- 8 Nếu không thì nv phải nối với 1 đỉnh khác 1nv  . Tiếp tục quá trình thì graph sẽ có vô hạn đỉnh, mâu thuẫn. Hệ quả. Từ kết quả trên suy ra nếu graph có bậc mỗi đỉnh đều lớn hơn hoặc bằng 2 thì có chu trình. Kết quả 10. Một cây có thể tạo thành từ 1 graph liên thông bằng cách bỏ đi 1 số cạnh. Thật vậy, ta có thể tạo ra bằng cách bỏ đi 1 cạnh trong 1 chu trình, làm với tất cả các chu trình ta sẽ thu được cây. Kết quả 11. Một graph liên thông là 1 cây nếu và chỉ nếu nó chứa đúng 1V  cạnh. Chứng minh Giả sử G là liên thông, có n đỉnh và có 1n  cạnh và G có chu trình, chẳng hạn 1 2 1... kA A A A . Nhận thấy nếu loại bỏ cạnh 1 2A A đồ thị mới vẫn liên thông, tuy nhiên lúc này nó chỉ có 2n  cạnh, mâu thuẫn với 1E V  , do G liên thông. Ngược lại, do mọi cây đều có đỉnh bậc 1, bỏ đỉnh này khỏi đồ thị, khi đó ta có 1 cây mới mà số đỉnh và số cạnh cùng giảm đi 1. Bằng quy nạp theo số đỉnh ta có điều chứng minh. Kết quả 12. Nếu bỏ đi bất kì 1 cạnh của cây thì nó không liên thông. Chứng minh Giả sử cây có n đỉnh, theo kết quả trên thì nó có 1n  cạnh. Nếu bỏ đi 1 cạnh thì nó còn 2n  cạnh. Cũng theo kết quả trên thì nó không còn là cây. Việc bỏ đi 1 cạnh sẽ không làm xuất hiện chu trình nào, suy ra nó không liên thông. Kết quả 13. Nếu graph G không có chu trình, có n đỉnh và 1n  cạnh thì nó là 1 cây. Chứng minh Cần chứng minh G liên thông. Phân hoạch G thành các thành phần liên thông, giả sử có k thành phần liên thông. Ta tạo ra 1k  cạnh mới bằng cách nối thành phần liên thông thứ 1 với thứ 2 (lấy 1 đỉnh ở 1 nối với 1 đỉnh ở 2), làm đến thành phần liên thông thứ k. CuuDuongThanCong.com https://fb.com/tailieudientucntt
  • 9. --------------------------------------------------------------------------------------------------------------------------- 9 Khi đó ta sẽ được graph mới liên thông và không có chu trình (mỗi thành phần liên thông không có chu trình và cách nối không tạo ra chu trình), suy ra graph mới này là 1 cây. Theo kết quả trên, số cạnh của cây này là 1n  , suy ra 1k  , nghĩa là graph ban đầu liên thông, ta có điều chứng minh. Kết quả 14. Giữa 2 đỉnh A, B bất kì trong 1 cây có đúng 1 đường đi. Chứng minh Do 1 cây là liên thông nên giữa A, B có ít nhất 1 đường đi. Giả sử còn 1 đường đi nữa nối A, B. Ta thấy chỉ xảy ra 1 trong 3 trường hợp như hình dưới đây, khi đó cây này có chu trình, mâu thuẫn. Vậy giả sử sai, có điều chứng minh. Nhận xét: 1. Nếu ta nối 2 đỉnh không kề nhau trong cây thì sẽ thu được 1 chu trình. Khi đó nếu graph G có n đỉnh và ít nhất n cạnh thì nó có chu trình. 2. Kết quả mạnh hơn được đưa ra bởi Erdos: Một graph có n đỉnh và số cạnh ít nhất là  1 2 n k thì tồn tại 1 chu trình có độ dài ít nhất là 1k  . 3. Một graph có n đỉnh và 1n  cạnh thì có ít nhất 2 chu trình. Thật vậy, trước hết khẳng định graph có 1 chu trình, ta xóa 1 cạnh của chu trình này thì graph còn lại có n cạnh, khi đó có 1 chu trình khác. Vậy nó có ít nhất 2 chu trình. 4. Nếu tất cả các đỉnh có bậc ít nhất là d thì có 1 đường đi có độ dài ít nhất là 1d  . Định nghĩa 13. Đường đi Hamilton là đường đi đi qua tất cả các đỉnh, mỗi đỉnh đúng 1 lần. Một đường đi Hamilton có điểm đầu và cuối trùng nhau được gọi là chu trình Hamiltonian. CuuDuongThanCong.com https://fb.com/tailieudientucntt
  • 10. --------------------------------------------------------------------------------------------------------------------------- 10 Ví dụ 1: Graph bên có đường đi 1, 2, 3, 4, 5 là 1 đường đi Hamilton. Đường đi 1, 2, 3, 4, 5, 6 là 1 chu trình Hamilton. Ví dụ 2: Trong các graph dưới đây, graph nào có chu trình Ole, chu trình Hamilton? Hình 1: Có cả chu trình Ole và Hamilton. Hình 2: Có chu trình Hamilton và không có chu trình Ole. Hình 3: Có chu trình Ole và không có chu trình Hamilton. Hình 4: Không có cả chu trình Ole và chu trình Hamilton. 5. Tô màu Số màu (sắc số) của graph là số nhỏ nhất của số màu cần thiết để tô các đỉnh sao cho không có 2 đỉnh kề nhau được tô cùng màu. Số màu của graph luôn nhỏ hơn hoặc bằng đỉnh lớn nhất cộng 1. Hiển nhiên theo định nghĩa trên. 2 5 4 1 3 6 CuuDuongThanCong.com https://fb.com/tailieudientucntt
  • 11. --------------------------------------------------------------------------------------------------------------------------- 11 Một số bài toán áp dụng Bài 1. Cho số nguyên dương 2n  và n thành phố, có 1 đường đi trực tiếp nối 2 thành phố bất kì, đường đi này thuộc sở hữu của 1 hãng hàng không. Biết rằng không có hãng hàng không nào có quá 2n  đường đi. Chứng minh tồn tại 3 thành phố mà đường đi nối chúng đôi một thuộc 3 hãng hàng không khác nhau. Giải Đưa bài toán về ngôn ngữ đồ thị như sau: Cho đồ thị đầy đủ có n đỉnh, tô màu mỗi cạnh bởi 1 màu sao cho không có màu nào được dùng quá 2n  lần. Chứng minh tồn tại một tam giác mà 3 cạnh được tô 3 màu khác nhau. Chứng minh bài toán bằng nguyên lí cựu hạn Giả sử phản chứng, nghĩa là không tồn tại tam giác mà 3 cạnh được tô 3 màu khác nhau. Giả sử ta sử dụng các màu 1, 2, 3, …, k. Với mỗi i, kí hiệu iC là tập tất cả các đỉnh mà với 2 đỉnh bất kì trong đó có đường đi được tô cùng màu i (Đường đi trong đồ thị là dãy các cạnh liên tiếp). Đặt 1,..., max i i k C C   , giả sử các đỉnh trong C được tô màu 1. Do điều kiện mỗi màu được dùng không quá 2n  lần nên tồn tại đỉnh v không thuộc C. Xét 2 đỉnh bất kì u, w trong C, khi đó cả 2 cạnh vu, vw được tô màu khác màu 1 (vì nếu có cạnh được tô màu 1 thì v thuộc C). Nếu 2 cạnh này được tô 2 màu khác nhau thì tam giác uvw thỏa mãn điều kiện, mâu thẫu giả sử. Nếu 2 cạnh này được tô cùng màu, giả sử màu 2. Khi đó tất cả các cạnh nối v và đỉnh trong C cùng được tô màu 2 (do u, w được lấy bất kì). Suy ra tập  C v chứa các đỉnh mà với 2 đỉnh bất kì trong đó có đường đi được tô cùng màu 2, tập này có nhiều phần tử hơn tập C, mâu thuẫn cách chọn C. Vậy điều giả sử là sai, ta có điều phải chứng minh. CuuDuongThanCong.com https://fb.com/tailieudientucntt
  • 12. --------------------------------------------------------------------------------------------------------------------------- 12 Bài 2 (IMO Shortlist 1990). Có 10 thành phố và 2 hãng hàng không cung cấp tất cả các đường bay trực tiếp từ 2 thành phố bất kì, giữa 2 thành phố có đúng 1 đường bay. Một hãng hàng không chỉ tổ chức được chuyến bay trên đường bay mà mình quản lí. Chứng minh có 1 hãng hàng không có thể tổ chức 2 chuyến du lịch, mỗi chuyến là đường đi vòng tròn qua lẻ thành phố và không có thành phố chung của 2 chuyến du lịch này. Giải Đưa bài toán về ngôn ngữ graph: Xét graph đầy đủ 10K . Mỗi cạnh của graph được tô bởi màu xanh hoặc đỏ. Chứng minh tồn tại 2 chu trình rời nhau, cùng màu và có độ dài lẻ. Trước hết cần có 2 bổ đề: Bổ đề 1. Mỗi cạnh của graph 6K được tô bởi 1 trong 2 màu, khi đó tồn tại tam giác đơn sắc. Chứng minh Dễ chứng minh bằng nguyên lí Dirichle. Bổ đề 2. Mỗi cạnh của graph 5K được tô bởi 1 trong 2 màu, giả sử không tồn tại tam giác đơn sắc. Khi đó tồn tại 2 chu trình đơn sắc có độ dài 5. Chứng minh Xét các đỉnh của 5K là 1 2 3 4 5, , , ,v v v v v và các cạnh 1 2 1 3 1 4 1 5, , ,v v v v v v v v . Nếu cả 4 cạnh này cùng màu thì sẽ tồn tại tam giác đơn sắc, mâu thuẫn giả thiết. Nếu có 3 cạnh, giả sử là 1 2 1 3 1 4, ,v v v v v v cùng màu, giả sử là màu đỏ. Xét 3 cạnh 2 3 3 4 4 2, ,v v v v v v , nếu có cạnh màu đỏ thì có tam giác đỏ, nếu không thì có tam giác xanh, mâu thuẫn giả sử. Vậy mỗi đỉnh là đầu mút của đúng 2 cạnh xanh và 2 cạnh đỏ. Xét graph sinh ra từ graph trên mà chỉ giữ lại cạnh được tô đỏ, graph này có 5 đỉnh, mỗi đỉnh đều có bậc 2, khi đó nó là hợp của các chu trình rời nhau, do chỉ có 5 đỉnh nên nó chỉ có 1 chu trình, chu trình này có độ dài là 5. Tương tự với graph khi ta chỉ giữ lại các cạnh được tô xanh. CuuDuongThanCong.com https://fb.com/tailieudientucntt
  • 13. --------------------------------------------------------------------------------------------------------------------------- 13 Ta có điều phải chứng minh. Quay lại bài toán Xét 10 đỉnh của graph là 1 2 10, ,...,v v v . Theo bổ đề 1, tồn tại tam giác đơn sắc, giả sử là 1 2 3v v v . Tiếp tục bổ đề 1 với graph con  10 1 2 3, ,K v v v , ta có tam giác đơn sắc, giả sử là 4 5 6v v v . Nếu 2 tam giác này cùng màu ta có điều phải chứng minh. Ngược lại, giả sử 1 2 3v v v màu xanh và 4 5 6v v v màu đỏ. Xét 9 cạnh ,1 3,4 6i jv v i j    , theo nguyên lí Dirichle có 5 cạnh trong số chúng cùng màu, giả sử là màu xanh. Khi đó tồn tại 0 0,4 6j j  sao cho 2 trong 3 cạnh 0 0 01 2 3, ,j j jv v v v v v cùng màu xanh, ta có 1 tam giác xanh và 1 tam giác đỏ chung nhau đúng 1 đỉnh 0jv , giả sử ta có 1 2 3v v v màu đỏ và 3 4 5v v v màu xanh. Xét graph con  10 1 5,...,K v v có 5 đỉnh. Nếu graph này có tam giác đơn sắc thì có điều chứng minh. Ngược lại, theo bổ đề 2 thì tồn tại 1 chu trình đơn sắc có độ dài 5. Chu trình này cùng với 1 trong 2 tam giác ở trên thỏa mãn điều kiện bài toán. Vậy bài toán được chứng minh. Bài 3. Một viện hàn lâm có 999 viện sĩ, mỗi đề tài khoa học có đúng 3 viện sĩ nghiên cứu và 2 viện sĩ bất kì có chung nhau đúng 1 đề tài cùng nghiên cứu. Chứng minh trong số các đề tài nghiên cứu, tồn tại 250 đề tài mà mỗi viện sĩ chỉ nghiên cứu tối đa 1 đề tài trong số 250 đề tài đó. Giải Giả sử các đề tài là 1, 2, …, n tương ứng với các màu 1, 2, …, n. Xây dựng đồ thị G có 999 đỉnh, 2 đỉnh nối với nhau nếu 2 viện sĩ tương ứng nghiên cứu cùng 1 đề tài, đồng thời tô màu cạnh này tương ứng với đề tài đó. CuuDuongThanCong.com https://fb.com/tailieudientucntt
  • 14. --------------------------------------------------------------------------------------------------------------------------- 14 Khi đó ta được G là đầy đủ, hơn nữa với mỗi màu có đúng 3 cạnh được tô màu này và 3 cạnh này tạo thành 1 tam giác đơn sắc. Cần chứng minh tồn tại 250 tam giác đơn sắc đôi một rời nhau. Gọi k là số lớn nhất các tam giác đơn sắc đôi một không có điểm chung. Nếu 250k  ta chọn k đề tài ứng với k màu của k tam giác trên thì bài toán được chứng minh. Nếu 250k  , trong  999 3k đỉnh còn lại không thể tìm được 1 tam giác đơn sắc nào nữa (nếu có thì k không lớn nhất). Gọi S là tập chứa  999 3k đỉnh này, T là tập các đỉnh không thuộc S. Suy ra với 2 điểm X, Y bất kì trong S, tồn tại duy nhất điểm Z trong T sao cho XYZ đơn sắc. Ta có 999 3.249 252, 249.3 747.S T     Có tất cả 2 S C cặp điểm trong S. Theo nguyên lí Dirichle, tồn tại 2 2 1 1 999 3 S S C C Tk                     cặp điểm trong S mà mỗi cặp điểm này đều kề với 1 đỉnh của tam giác ABC trong T. Dễ dàng chứng minh được 2 1 ,252 999 999 2 3 S C S S T               nên trong số các cặp điểm kề với các đỉnh của tam giác ABC sẽ có 2 cặp kề với 2 đỉnh khác nhau (vì nếu không thì sẽ có 2 cặp điểm chung nhau 1 đỉnh cùng nối với 1 đỉnh của tam giác, suy ra có 4 đỉnh tương ứng với 4 người có cùng đề tài, mâu thuẫn). Suy ra tồn tại 2 cặp điểm  1 1,X Y và  2 2,X Y trong S tương ứng kề với 2 điểm ,A B là 2 đỉnh của tam giác ABC đơn sắc trong T. CuuDuongThanCong.com https://fb.com/tailieudientucntt
  • 15. --------------------------------------------------------------------------------------------------------------------------- 15 Khi đó nếu bỏ đi tam giác ABC và thay bởi 2 tam giác đơn sắc 1 1X Y A và 2 2X Y B thì số tam giác đơn sắc nhiều hơn k, trái với tính lớn nhất của k. Vậy 250k  không thỏa mãn. Ta có điều chứng minh. Bài 4. Cho 1 nhóm người mà mỗi người quen không quá m người khác trong nhóm. Chứng minh có thể chia nhóm người này thành 1m  nhóm sao cho 2 người trong cùng 1 nhóm thì không quen nhau. Giải Xây dựng graph G có số đỉnh ứng với số người trong nhóm, 2 đỉnh được nối với nhau nếu 2 người tương ứng quen nhau. Tô màu 1m  đỉnh bất kì bởi 1m  màu khác nhau. Thực hiện tô màu các đỉnh còn lại sao cho 2 đỉnh kề nhau được tô màu khác nhau. Do mỗi đỉnh kề với không quá m đỉnh nên việc tô màu thực hiện được. Khi đó tạo được 1m  nhóm mà trong mỗi nhóm các đỉnh được tô cùng màu, hay chúng không kề với nhau. Ta có điều chứng minh. Bài 5 (IMO 1992, P3). Cho G là graph đầy đủ với 9 đỉnh. Mỗi cạnh được tô bởi 1 trong 2 màu xanh, đỏ hoặc không tô màu. Tìm n nhỏ nhất sao cho nếu tô màu n cạnh bất kì thì tồn tại ít nhất 1 tam giác đơn sắc (tam giác có 3 cạnh không được tô không được gọi là đơn sắc). Giải Số cạnh của graph là 2 9 9.4 36C   . +) Chứng minh tồn tại cách tô 32 cạnh mà không có tam giác đơn sắc. Xét đỉnh V và tô 4 cạnh xanh nối tới các đỉnh 1 2 3 4, , ,B B B B và 4 cạnh đỏ nối tới các đỉnh 1 2 3 4, , ,R R R R . Các cạnh 1i iB B tô đỏ, 1i iR R tô xanh với mọi 1,2,3,4i  . Các cạnh i jB R tô xanh nếu i j lẻ, cạnh i jB R tô đỏ nếu i j chẵn. Khi đó ta tô màu 32 cạnh và không có tam giác đơn sắc. Suy ra 33n  . +) Chứng minh 33n  thỏa mãn. Giả sử phản chứng, có thể tô 33 cạnh mà không có tam giác đơn sắc. CuuDuongThanCong.com https://fb.com/tailieudientucntt
  • 16. --------------------------------------------------------------------------------------------------------------------------- 16 Xét đỉnh X bất kì, nếu X là đầu mút của 5 cạnh được tô xanh, giả sử được nối với các đỉnh 1 2 3 4 5, , , ,A A A A A . Nếu có cạnh nối 2 trong 5 đỉnh trên được tô màu thì đó là màu đỏ. Chú ý: Nếu có nhiều hơn 2 4 k cạnh của graph k đỉnh được tô đỏ thì tồn tại tam giác đơn sắc đỏ. Suy ra trong các cạnh được tạo từ 5 đỉnh trên thì có nhiều nhất 2 5 6 4       cạnh được tô đỏ hay có ít nhất 4 cạnh không được tô màu, suy ra có nhiều nhất 32 cạnh được tô màu, mâu thuẫn. Suy ra X là đầu mút của nhiều nhất 4 cạnh được tô xanh, tương tự suy ra X là đầu mút của nhiều nhất 4 cạnh được tô đỏ. Giả sử X là đầu mút của đúng 8 cạnh được tô màu thì có đúng 4 cạnh được tô xanh nối với 1 2 3 4, , ,B B B B và đúng 4 cạnh được tô đỏ nối với 1 2 3 4, , ,R R R R . Trong 6 cạnh i jB B nếu được tô màu thì đc tô không nhiều hơn 4 4 4 4  cạnh đỏ, nghĩa là có ít nhất 2 cạnh không được tô màu. Tương tự với các cạnh i jR R có ít nhất 2 cạnh không được tô màu. Suy ra có ít nhất 4 cạnh của G không được tô màu, nghĩa là tô màu nhiều nhất 32 cạnh, mâu thuẫn. Suy ra, nếu chỉ xét những cạnh được tô màu thì không có đỉnh nào có bậc 8, suy ra bậc mỗi đỉnh nhiều nhất là 7, suy ra tổng bậc của 9 đỉnh nhiều nhất là 9.7 63 2.33 69   , mâu thuẫn. Vậy giả sử sai, ta có điều chứng minh. Bài 6. Cho graph G đơn, vô hướng, hữu hạn. Chứng minh có thể tô tất cả các đỉnh của G, mỗi đỉnh bởi 1 trong 2 màu sao cho 2 đỉnh kề nhau được tô màu khác nhau khi và chỉ khi mọi chu trình đơn trong G đều có độ dài chẵn. Giải 1) Giả sử có thể tô tất cả các đỉnh của G, mỗi đỉnh bởi 1 trong 2 màu xanh, đỏ sao cho 2 đỉnh kề nhau được tô màu khác nhau. Nếu G không có chu trình ta có điều chứng minh. CuuDuongThanCong.com https://fb.com/tailieudientucntt
  • 17. --------------------------------------------------------------------------------------------------------------------------- 17 Nếu G có chu trình, xét chu trình bất kì  1 2 1... , 1,2,...,n n i iv v v v v v i n   , giả sử 1v được tô xanh, suy ra tất cả các đỉnh iv với i lẻ được tô xanh và iv với i chẵn được tô đỏ. Do 1v và nv kề nhau nên nv được tô đỏ, hay n chẵn. Ta có điều chứng minh. 2) Giả sử mọi chu trình đơn trong G đều có độ dài chẵn. Ta chỉ ra cách tô màu thỏa mãn điều kiện Chia G thành các thành phần liên thông rời nhau, và tô màu từng thành phần liên thông. Xét thành phần liên thông P có tập đỉnh là V. Chọn đỉnh u trong V và tô xanh cho u. Xét đỉnh v khác u bất kì trong V. Do mọi chu trình đều có độ dài chẵn nên mọi đường đi từ v đến u có độ dài cùng tính chẵn lẻ. Nếu độ dài các đường đi là chẵn thì tô v bởi màu xanh, ngược lại tô v bởi màu đỏ. Ta được cách tô thỏa mãn điều kiện. Ta có điều chứng minh. Bài 7 (TOT 1986). Có 20 đội bóng tham gia giải thi đấu. Trong ngày đầu tiên tất cả các đội thi đấu với 1 đội khác. Ngày thứ 2 tất cả các đội thi đấu với 1 đội khác ngày hôm trước. Chứng minh sau ngày thi đấu thứ 2 có thể chọn được 10 đội sao cho không có 2 đội trong số chúng đã thi đấu với nhau. Giải Xây dựng đồ thị 20 đỉnh ứng với 20 đội bóng, cạnh nối hai đỉnh khi 2 đội thi đấu với nhau. Tô màu đỏ cho cạnh ứng với trận đấu của ngày thứ nhất và màu xanh ứng với trận đấu của ngày thứ 2. Khi đó tại mỗi đỉnh đều có bậc 2 và là đầu mút của 1 canh đỏ, 1 cạnh xanh, suy ra các thành phần của đồ thị là các chu trình chẵn rời nhau. Trong mỗi chu trình chọn được 1 nửa đỉnh trong số chúng không kề nhau. Ta có điều phải chứng minh. Bài 8. a. Xét graph  ,G V E , tập D chứa các đỉnh của G được gọi là “trội” nếu mỗi đỉnh của G hoặc thuộc D, hoặc có 1 đỉnh kề thuộc D. CuuDuongThanCong.com https://fb.com/tailieudientucntt
  • 18. --------------------------------------------------------------------------------------------------------------------------- 18 Chứng minh nếu G không có điểm cô lập thì tồn tại 1 tập “trội” có không quá 2 V đỉnh. b. (Nga 2001) Trong một bữa tiệc có  * 2 1n n  người. Biết rằng nếu chọn ra n người bất kì thì tồn tại 1 người trong số 1n  người còn lại quen với tất cả n người này. Chứng minh tồn tại 1 người quen với tất cả những người còn lại trong bữa tiệc. Chứng minh bổ đề Dễ thấy nếu G không có điểm cô lập thì có thể phân hoạch tập đỉnh V thành 2 tập 1 2,V V sao cho mỗi đỉnh trong 1V có đỉnh kề trong 2V và ngược lại. Hai tập này đều là các tập trội nên có 1 tập có không quá 2 V phần tử, ta có điều chứng minh. Dễ thấy nếu tập D là trội thì các tập hợp thành của D và 1 số đỉnh không thuộc D cũng là tập trội. Khi đó ta có thể kết luận rằng tồn tại tập trội có đúng 2 V phần tử. Chứng minh bài toán Phát biểu lại bằng ngôn ngữ graph: Cho graph có 2 1n  đỉnh ứng với 2 1n  người, nếu cặp 2 người không quen nhau thì 2 đỉnh tương ứng sẽ nối với nhau. Giả sử phản chứng, nghĩa là graph không có đỉnh cô lập, theo bổ đề trên, tồn tại tập D là tập trội, tập này có n phần tử. Do điều kiện bài toán nên tồn tại 1 đỉnh trong số 1n  đỉnh còn lại không nối với bất kì đỉnh nào trong D, mâu thẫu với định nghĩa tập trội. Vậy giả sử sai, ta có điều chứng minh. Bài 9 (China TST 2012). Cho n và k là các số nguyên dương sao cho 2n  và 2 n k n  . Xét graph G có n đỉnh sao cho G không chứa chu trình độ dài 1k  nhưng nếu thêm bất kì 1 cạnh mới trong G thì sẽ có chu trình độ dài 1k  . Gọi CuuDuongThanCong.com https://fb.com/tailieudientucntt
  • 19. --------------------------------------------------------------------------------------------------------------------------- 19 đỉnh trong G là tốt nếu nó liên thông đến mọi đỉnh trong G. Hỏi G có ít nhất bao nhiêu đỉnh tốt. Giải Chứng minh nếu 1 2 1k n k    thì có ít nhất 2k n đỉnh tốt. Do 1k n  nên có 2 đỉnh ,u v G không kề nhau, xét tập  ,T u v , S là tập các đỉnh mà khi nối u, v thì cùng với các đỉnh này lập thành chu trình độ dài 1k  , tập   ,U G T S . Chú ý rằng 1 2S k k n U     . Giả sử có bộ 3 tập rời nhau T, S, U sao cho  ,G T S U T S N S     và 2S k n U   . Nếu U là tập rỗng thì S chứa ít nhất 2k n đỉnh tốt, có điều cần chứng minh. Ngược lại, chọn 1 đỉnh tùy ý u U . Nếu  S N u thì theo trên xét với    ' , ' U U u T T u  và 'S S thì ' 2 'S k n U   . Cố định đỉnh v thuộc S sao cho u, v không kề nhau. Theo trên, tồn tại 1k  đỉnh cùng với u, v tạo thành chu trình độ dài 1k  lấy trong cá tập  0 0 ,U U u T T  và  0 S S v . Nhưng G không có chu trình độ dài 1k  nên 0T S k  . Suy ra 0 0 0 0 01k U T S U k S S        Suy ra  0 0 01 2 1S U S k n U U         Do 0 1U U  nên 0S khác rỗng. Xét các tập       0 0 0' , ' 'U U u U T T u U S S      và 0'S S . Ta có điều chứng minh. Bài 10 (IMO Shortlist 2004, C8). Cho graph hữu hạn G, kí hiệu  f G là số tam giác trong G và  g G là số đồ thị con 4K của G. Xác định giá trị nhỏ nhất của c sao cho     3 4 ,g G cf G G  . CuuDuongThanCong.com https://fb.com/tailieudientucntt
  • 20. --------------------------------------------------------------------------------------------------------------------------- 20 Giải Trước hết có bổ đề: Một graph có f tam giác và h cạnh thì 2 32 9 f h . Chứng minh Đặt id là bậc của đỉnh i. Do mỗi đỉnh là đỉnh của nhiều nhất h tam giác, nhưng tính theo bậc thì nó là đỉnh của nhiều nhất 2 2 id tam giác. Suy ra 2 3 3 . .2 2 2 2 2 i i d h h f h d h h     , có điều chứng minh. Quay lại bài toán. Graph G có f tam giác và g đồ thị con 4K . Xét đỉnh i là đỉnh của if tam giác và ig đồ thị con 4K . Xét graph con gồm các đỉnh kề với đỉnh này, graph này có if cạnh và ig tam giác, theo bổ đề ta có 2 32 9 i ig f . Tuy nhiên ta có ig f , suy ra 1 1 1 3 3 3 32 2 2 4 . .3 9 9 9 i i i f f g g f f f f                         Suy ra 3 3 4 3 42 3 64 .27 2.27 9 32 f g f f g f          Vậy giá trị nhỏ nhất của c là 3 32 . Bài 11. (Croatian TST 2011) Có n người trong 1 bữa tiệc, một số người trong đó quen biết nhau. Chọn 4 người bất kì thì có 3 người hoặc cùng quen với người còn lại hoặc cùng không quen với người còn lại. Chứng minh có thể chia những người này thành 2 nhóm rời nhau A và B sao cho những người trong A đôi một quen nhau và những người trong B đôi một không quen nhau, (mỗi nhóm A, B có thể không có người). CuuDuongThanCong.com https://fb.com/tailieudientucntt
  • 21. --------------------------------------------------------------------------------------------------------------------------- 21 Giải Phát biểu lại theo ngôn ngữ graph: Xét graph G có n đỉnh ứng với n người, nối 2 đỉnh nếu 2 người tương ứng quen nhau. Điều kiện bài toán tương đương: Với 4 đỉnh bất kì thì có 3 đỉnh cùng kề với đỉnh còn lại hoặc cùng không kề với đỉnh còn lại. Chú ý: Một graph luôn tồn tại graph con đầy đủ. Một graph con đầy đủ được gọi là lớn nhất nếu nó chứa nhiều đỉnh nhất. Dễ thấy nếu A là graph con đầy đủ lớn nhất của G thì nó thỏa mãn điều kiện, ta cần chứng minh B là tập G A . Nếu G là đầy đủ hoặc tất cả các đỉnh của G là cô lập thì có điều cần chứng minh. Ngược lại, ta có 2 1A n   . Giả sử có 2 đỉnh 1 2,v v trong G A là 2 đỉnh kề nhau. Do A là tập con đầy đủ lớn nhất của G nên tồn tại đỉnh 1u trong A không kề với 1v . (Nếu không thì 1v kề với tất cả các đỉnh trong A, khi đó  1A v là graph đầy đủ lớn hơn A). Nếu 1u không kề với 2v , xét đỉnh u bất kì trong A. Với 4 đỉnh 1 2 1, , ,u v v u : u kề với 1u , 1v kề với 2v và 1u không kề 2v , 1u không kề 1v , suy ra 1 1 2, ,v u v cùng kề với u. Suy ra graph     1 1 2,A u v v  là graph đầy đủ lớn hơn A. Vậy 1u kề với 2v , lập luận tương tự như trên thì u kề với 2v . Khi đó graph  2A v là graph đầy đủ lớn hơn A. Vậy giả sử sai, suy ra trong G A tất cả các đỉnh là cô lập, ta có điều cần chứng minh. CuuDuongThanCong.com https://fb.com/tailieudientucntt
  • 22. --------------------------------------------------------------------------------------------------------------------------- 22 Bài 12 (USA 1982). Trong nhóm 1982 người, với 4 người bất kì thì có ít nhất 1 người quen với 3 người còn lại. Có ít nhất bao nhiêu người mà người này quen với tất cả những người còn lại. Giải Phát biểu lại theo ngôn ngữ graph: Xét graph G có 1982 đỉnh ứng với 1982 người, nối 2 đỉnh nếu 2 người tương ứng quen nhau. Xét 4 đỉnh bất kì thì có ít nhất 1 đỉnh kề với cả 3 đỉnh còn lại. Cần xem xét trong G có ít nhất bao nhiêu đỉnh kề với tất cả những đỉnh còn lại. Nếu bất kì đỉnh nào trong G cũng kề với các đỉnh còn lại thì giá trị nhỏ nhất là 1981. Nếu có 2 đỉnh A, B không kề nhau. Xét 2 đỉnh bất kì C, D khác A, B thì 2 đỉnh này phải kề nhau, ngược lại 4 đỉnh A, B, C, D không có đỉnh nào kề với cả 3 đỉnh còn lại. Nếu A và B cùng kề với tất cả các đỉnh còn lại thì có 1980 đỉnh mà mỗi đỉnh kề với tất cả các đỉnh còn lại. Nếu có C không kề với A thì đỉnh D bất kì sẽ kề với cả A, B, C. Khi đó có 1979 đỉnh mà mỗi đỉnh kề với tất cả những đỉnh còn lại. Vậy có ít nhất 1979 đỉnh mà mỗi đỉnh kề với tất cả những đỉnh còn lại. Bài 13 (Taiwan 2001). Cho số nguyên 3n  và 1 2, ,..., nA A A là các tập con phân biệt của của  1,2,...,S n . Chứng minh tồn tại phần tử x S sao cho các tập      1 2 , ,..., nA x A x A x là phân biệt. Giải Xét graph G với các đỉnh 1 2, ,..., nA A A . Với phần tử y S , nếu 2 tập  iA y và  jA y trùng nhau thì nối iA và jA bởi 1 cạnh, mỗi y ta chỉ nối 1 cạnh (nghĩa là nếu có nhiều cặp 2 tập  ,i jA A cùng thỏa mãn điều kiện thì ta chọn ra 1 cặp bất kì nào đó). Dễ thấy không có 2 đỉnh nào được nối 2 lần, ta có graph đơn. Giả sử phản chứng, không tồn tại phần tử x thỏa mãn yêu cầu bài toán. Khi đó với mỗi phần tử của S sẽ có đúng 1 cặp đỉnh được nối với nhau. Hơn nữa không xảy ra trường hợp tồn tại ,i jA A phân biệt mà CuuDuongThanCong.com https://fb.com/tailieudientucntt
  • 23. --------------------------------------------------------------------------------------------------------------------------- 23        1 1 2 2 , i j i jA x A x A x A x  với 1 2,x x phân biệt (nếu có thì ,i jA A trùng nhau). Suy ra graph có ít nhất n cạnh, suy ra tồn tại 1 chu trình, giả sử là  1 2, ,..., , 3kA A A k  , có nghĩa tồn tại các phần tử phân biệt 1 2, ,..., kx x x S sao cho    1 1 2 1 ,A x A x        2 2 3 2 1 ,..., k k kA x A x A x A x  . Do điều kiện    1 1 2 1 A x A x nên 1x thuộc đúng 1 trong 2 tập 1 2,S S , giả sử 1 2x A Suy ra 1 3x A (do 1 2x x ), tiếp tục quá trình suy ra 1 kx A và cuối cùng 1 1x A , mâu thuẫn. Vậy giả sử phản chứng sai, ta có điều chứng minh. Bài 14 (USA TST 2002). Cho n là số nguyên dương và S là tập có 2 1n  phần tử. Xét hàm f đi từ tập các tập con có 2 phần tử của S vào tập  1 0,1,2,...,2 1n  . Giả sử với bộ ba phần tử bất kì  , ,x y z của S, một trong các số         , , , , ,f x y f y z f z x bằng tổng của 2 số còn lại. Chứng minh tồn tại , ,a b c S sao cho         , , , 0f a b f b c f c a   . Giải Xét tập con S’ của S sao cho 1 ' 2 1n S    và với mọi , 'x y S thì   ,f x y là số chẵn. Đặt      , , , , ' 2 f x y g x y x y S   , ta có hàm g từ S’ đến  2 0,1,2,...,2 1n  có tính chất giống như hàm f. Vậy ta sẽ xét hàm f từ S’ đến  2 0,1,2,...,2 1n  mà với mọi a, b thuộc S’ thì   ,f a b lẻ hoặc   , 0f a b  . Xét graph G có 2 1n  đỉnh tương ứng các phần tử trong S. Hai đỉnh a, b nối với nhau nếu và chỉ nếu   ,f a b lẻ. Ta chứng minh G là hai phần. Chú ý rằng với bất kì các đỉnh a, b, c thì có 0 hoặc 2 cạnh tạo ra từ chúng. CuuDuongThanCong.com https://fb.com/tailieudientucntt
  • 24. --------------------------------------------------------------------------------------------------------------------------- 24 Nếu G không là hai phần từ có 1 chu trình lẻ, giả sử chu trình lẻ có độ dài ngắn nhất là 1 2 2 1... kv v v  . Xét 3 đỉnh 1 3 4, ,v v v có 0 hoặc 2 cạnh tạo ra từ chúng. Do đã có cạnh 3 4v v nên sẽ có 1 trong 2 cạnh 1 3v v hoặc 1 4v v , hơn nữa không tồn tại cạnh 1 3v v (nếu không 3 đỉnh 1 2 3, ,v v v có 3 cạnh) nên có cạnh 1 4v v , suy ra chu trình 1 4 5 2 1... kv v v v  có độ dài nhỏ hơn hoặc bằng 1 2 2 1... kv v v  , mâu thuẫn. Vậy G là 2 phần, có 1 phần có ít nhất 3 phần tử, 3 phần tử này không nối với nhau, ta có điều chứng minh. Bài 15 (IMO Shortlist 2002, C6). Cho n là số nguyên dương chẵn. Chứng minh tồn tại 1 hoán vị  1 2, ,..., nx x x của  1,2,...,n sao cho với bất kì  1,2,...,i n , số 1ix  là 1 trong các số 2 ,2 1,2 ,2 1i i i ix x x n x n    , quy ước n i ix x  . Giải Đặt 2n m . Xây dượng graph có hướng với m đỉnh và 2m cạnh. Với mỗi i m , đỉnh i có 2 cạnh đi ra được đánh số 2 1,2i i và 2 cạnh đi vào được đánh số là ,i i m . Ta cần chứng minh có 1 chu trình Ole, bởi vì các cặp cạnh liên tiếp tạo ra có 1 trong các dạng        ,2 1 , ,2 , ,2 , ,2 1i i i i i m i i m i    . Ta thấy, bậc vào của mỗi đỉnh bằng bậc ra suy ra tồn tại 1 chu trình Ole. Thật vậy, ta có thể chứng minh điều trên bằng phương pháp quy nạp. Có một đường đi từ 1 đến k: Do có 1 đường đi từ 1 đến j khi 2 j k hoặc 2 1j k  và 1 cạnh từ j đến k suy ra có 1 đường đi từ 1 đến k. Suy ra điều chứng minh. Bài 16 (St Petersburg 1996, P4). Trong 1 nhóm người, một số người quen biết nhau và một số người không. Mỗi tối, một người nào đó đãi tiệc mời tất cả những người mình quen biết và giới thiệu họ cho nhau. Biết rằng, sau khi mỗi người đều đã tổ chức tiệc vẫn còn hai người trong nhóm vẫn chưa quen nhau. Chứng minh tại bữa tiệc tiếp theo, họ vẫn không được giới thiệu cho nhau. Giải CuuDuongThanCong.com https://fb.com/tailieudientucntt
  • 25. --------------------------------------------------------------------------------------------------------------------------- 25 Xây dựng graph có n đỉnh (giả giả nhóm này có n người), nối 2 đỉnh khi 2 người tương ứng quen nhau. Mỗi bước thực hiện công việc như sau: Chọn 1 đỉnh và nối đôi một tất cả các đỉnh kề với đỉnh này. Khi đó ta được 1 graph con đầy đủ. Sau khi thực hiện với tất cả các đỉnh ta sẽ thu được các graph đầy đủ như trên. Theo giả thiết bài toán sẽ tồn tại 2 graph rời nhau. Các bước thực hiện tiếp theo sẽ không tạo thêm cạnh nào, hay 2 graph này luôn rời nhau. Ta có điều chứng minh. Bài 17 (Czech – Slovak Mathch 1997, P2). Trong một hội đồng có nhiều hơn 6 người, mỗi người trao đổi thư với đúng 3 người khác. Chứng minh có thể chia hội đồng này thành 2 nhóm khác rỗng mà mỗi người trao đổi thư với ít nhất 2 người trong nhóm của anh ta. Giải Xét graph G có n đỉnh (n là số người trong hội đồng), 2 đỉnh được nối với nhau nếu 2 người tương ứng trao đổi thư với nhau. Chứng minh có thể chia graph này thành 2 graph con mà trong mỗi graph bậc của mỗi đỉnh ít nhất là 2. Theo giả thiết mỗi đỉnh có bậc đúng là 3, suy ra trong graph tồn tại chu trình. Xét chu trình ngắn nhất, mỗi đỉnh trong chu trình này nối với ít nhất 2 đỉnh trong chu trình đó, vậy chu trình này là 1 graph con thỏa mãn điều kiện, kí hiệu graph này là G’. Xét các đỉnh còn lại, nếu có các đỉnh 1 2, ,..., kv v v , mỗi đỉnh nối với ít nhất 2 đỉnh của graph con trên thì graph  1 2' , ,..., kG v v v là graph con thỏa mãn. Ngược lại, các đỉnh còn lại, mỗi đỉnh chỉ nối với nhiều nhất 1 đỉnh trong G’ thì graph 'G G thỏa mãn điều kiện. Ta có điều chứng minh. Bài 18 (BAMO 2005/4). Có 1000 thành phố ở đất nước của Ole, một số cặp 2 thành phố được nối với nhau bởi 1 con đường đất. Biết rằng giữa 2 thành phố bất kì có 1 đường đi (đi qua 1 số đường đất liên tiếp). Chứng minh chính phủ có thể lát một số con đường sao cho ở 1 thành phố bất kì luôn có lẻ con đường được lát xuất phát từ đây. Giải CuuDuongThanCong.com https://fb.com/tailieudientucntt
  • 26. --------------------------------------------------------------------------------------------------------------------------- 26 Mấu chốt của bài toán 1000 là số chẵn, ta có thể thay 1000 bằng 1 số chẵn bất kì thì bài toán vẫn đúng. Theo kết quả Kết quả 10 ta có thể hạn chế graph về trường hợp graph con là 1 cây. Ta chứng minh với cây có chẵn đỉnh luôn có thể lát 1 số cạnh mà tại mỗi đỉnh có lẻ cạnh được lát. Xét 1 lá và đỉnh v kề với lá này, lát cạnh nối 2 đỉnh và xóa bỏ 2 đỉnh này. Khi đó ta sẽ thu được các thành phần liên thông rời nhau, mỗi thành phần liên thông đều có đỉnh nối với v (vì nếu không thì ngay lúc đầu thành phần liên thông này không liên thông với v, mâu thuẫn). Có chẵn đỉnh còn lại nên số thành phần liên thông có lẻ đỉnh là chẵn. Ta xét thành phần liên thông G có lẻ đỉnh, trong thành phần này có đỉnh u nối với v, ta lát cạnh nối 2 đỉnh này và loại đỉnh u, khi đó  G u được chia thành các thành phần liên thông có ít đỉnh hơn. Khi đó đỉnh v có lẻ cạnh được lát (có cạnh nối với lá lúc đầu và các cạnh vừa xét). Với thành phần liên thông có chẵn đỉnh thì làm tương tự như graph ban đầu. Vậy sau mỗi lần lát cạnh thì sẽ thu được các thành phần liên thông mới có ít đỉnh hơn thành phần liên thông ban đầu. Với mỗi thành phần liên thông, tùy thuộc số đỉnh chẵn hay lẻ ta sẽ tiếp tục theo 1 trong 2 bước trên. Ta có điều chứng minh. Bài 19 (Belarus 2010). Cho 3n  điểm phân biệt trên mặt phẳng sao cho không có 3 điểm thẳng hàng. Nối tất cả các đoạn thẳng tạo bởi 2 điểm trong n điểm trên. Mỗi đoạn được tô bởi 1 trong 4 màu sao cho nếu với 1 tam giác có 2 cạnh được tô cùng màu thì cạnh còn lại cũng được tô màu này. Biết rằng mỗi màu được dùng ít nhất 1 lần. Tìm giá trị lớn nhất của n. Giải Xét mô hình đồ thị như bài toán. Giả sử các màu được dùng là 1, 2, 3, 4. CuuDuongThanCong.com https://fb.com/tailieudientucntt
  • 27. --------------------------------------------------------------------------------------------------------------------------- 27 Xét các đồ thị con đầy đủ mà các cạnh được tô cùng màu (ta gọi là đồ thị đơn sắc). Nhận thấy luôn tồn tại những đồ thị như vậy, ít nhất 1 cạnh được tô màu là 1 đồ thị thỏa mãn. Xét đồ thị G nhiều đỉnh nhất thỏa mãn điều trên và các cạnh của nó được tô màu 1. Do mỗi màu được dùng ít nhất 1 màu nên có đỉnh v không thuộc G. Xét các cạnh nối v với các đỉnh trong G, nếu có cạnh được tô màu 1 thì tất cả các cạnh đều được tô đỏ, khi đó  G v đầy đủ, đơn sắc nhưng nhiều đỉnh hơn G. Nếu có 2 cạnh cùng được tô màu khác thì cạnh còn lại phải được tô màu này, mâu thuẫn do nó đang được tô màu 1. Suy ra tại mỗi đỉnh v không thuộc G thì các cạnh được tô màu đôi một khác nhau và khác mà 1, suy ra G có nhiều nhất 3 đỉnh. Khi đó tại mỗi đỉnh, chỉ có nhiều nhất 2 cạnh cùng màu (vì nếu có 3 cạnh cùng màu thì có 3 đỉnh cùng với đỉnh này tạo thành 1 đồ thị đầy đủ đơn sắc). Có 4 màu được dùng nên bậc của mỗi đỉnh không vượt quá 8, hay số đỉnh không vượt quá 9. Với 9 đỉnh 1 2 3 1 2 3 1 2 3, , , , , , , ,a a a b b b c c c thực hiện việc tô màu như sau: tô màu 1 cho các cạnh của các đồ thị đầy đủ      1 2 3 1 2 3 1 2 3, , , , , , , ,a a a b b b c c c . Tô màu 2 cho các cạnh của các đồ thị đầy đủ  , , , 1,2,3i i ia b c i  Tô màu 3 cho các cạnh của các đồ thị đầy đủ  1 2, , , 1,2,3i i ia b c i   Tô màu 4 cho các cạnh của các đồ thị đầy đủ  2 4, , , 1,2,3i i ia b c i   . Ta có điều chứng minh. Bài 20 (IMO 1990). Cho E là tập chứa 2 1n  điểm trên đường tròn với 3n  là số nguyên. Tô màu đen k điểm trong E. Một cách tô màu được gọi là tốt nếu tồn tại ít nhất 2 điểm được tô đen mà bên trong 1 cung tròn có đầu mút là 2 điểm này có đúng n điểm của E. Tìm giá trị nhỏ nhất của k sao cho với mọi cách tô k điểm đều là 1 cách tô màu tốt. Giải CuuDuongThanCong.com https://fb.com/tailieudientucntt
  • 28. --------------------------------------------------------------------------------------------------------------------------- 28 Xét graph G mà các đỉnh là các điểm của E. Hai đỉnh được nối với nhau nếu bên trong 1 cung tròn có đầu mút là 2 điểm này có đúng n điểm của E. Tìm giá trị nhỏ nhất của k để với k đỉnh bất kì trong G đều có 2 đỉnh kề. Nhận thấy bậc của mỗi đỉnh là 2, suy ra G là hợp của các chu trình rời nhau. Chú ý rằng, trong 1 chu trình độ dài r thì cần chọn ra ít nhất 1 2 r      đỉnh để đảm bảo luôn có 2 đỉnh kề nhau. Giả sử các điểm thuộc E theo thứ tự cùng chiều kim đồng hồ trên đường tròn là 1 2 2 1, ,..., nA A A  . Khi đó chu trình chứa 1A là:  1 2 4 5 7 8 1 1, , , , , ,..., ,n n n nA A A A A A A A    . Khi đó G là hợp của nhiều nhất 3 chu trình (chứa 1A , 2A hoặc 3A ), số lượng chu trình phụ thuộc số dư khi chia 2 1n  cho 3. Nếu 2 1n  không chia hết cho 3 thì G là 1 chu trình có độ dài 2 1n  , các đỉnh cần chọn ít nhất là 2 1 1 2 n n       . Nếu 2 1n  chia hết cho 3 thì G là hợp của 3 chu trình, mỗi chu trình có độ dài 2 1 3 n  , các đỉnh cần chọn ít nhất là 2 1 1 3 1 6 n n        . Vậy giá trị nhỏ nhất của k là 2 1 1 2 n n       khi 2 1n  không chia hết cho 3 và là 2 1 1 3 1 6 n n        khi 2 1n  chia hết cho 3 . Bài 21. Một công ty muốn xây một công trình có kích thước 2017 2017 gồm 2 2017 phòng, mỗi phòng có kích thước 1 1 , một số phòng kề nhau (chung cạnh) được nối với nhau bằng 1 cửa giữa 2 phòng. Hỏi có thể xây dựng mà mỗi phòng có đúng 2 cửa hay không? Giải Giả sử có thể xây dựng mà mỗi phòng có đúng 2 cửa. CuuDuongThanCong.com https://fb.com/tailieudientucntt
  • 29. --------------------------------------------------------------------------------------------------------------------------- 29 Xét graph G có 2 2017 đỉnh, 2 đỉnh được nối với nhau nếu có cửa nối 2 phòng tương ứng. Khi đó mỗi đỉnh có bậc đúng bằng 2 suy ra G là hợp của các chu trình rời nhau. Trên thực tế, để đi từ 1 phòng qua các phòng khác rồi quay lại đúng phòng đó thì phải qua chẵn cửa (các bước đi qua cửa theo hướng thẳng đứng với 2 chiều khác nhau là bằng nhau nên tổng bước đi theo hướng thẳng đứng là chẵn, tương tự tổng bước đi theo hướng nằm ngang đều chẵn). Suy ra mỗi chu trình có độ dài chẵn nên mỗi chu trình có chẵn đỉnh suy ra G có chẵn đỉnh, mâu thuẫn khi 2 2017 lẻ. Vậy giả sử sai, suy ra không thể xây dựng mà mỗi phòng có đúng 2 cửa. Bài 22 (Iran TST 2006). Một đất nước có 2 hãng hàng không, giữa 2 thành phố bất kì có đường bay của đúng 1 hãng hàng không. Chứng minh tồn tại 1 thành phố mà từ một thành phố bất kì có thể chọn đường bay đến đó mà không cần thay đổi hãng hàng không. Giải Xét graph đầy đủ G có số đỉnh bằng số thành phố, mỗi cạnh được tô bởi 1 trong 2 màu xanh, đỏ tương ứng với đường bay của hai hãng hàng không. Cần chứng minh tồn tại đỉnh v sao cho với đỉnh u bất kì thì tồn tại đường đi đơn sắc nối u và v. Ta sẽ chứng minh quy nạp theo số đỉnh của G. Khi G có 1, 2, 3, 4 đỉnh là hiển nhiên. Bài 23 (BMO 1987). Một cuộc hội thảo có 1985 người, nếu chọn ra 3 người bất kì thì luôn có ít nhất 2 người nói cùng ngôn ngữ. Giả sử mỗi người có thể nói được nhiều nhất 5 ngôn ngữ. Chứng minh có ít nhất 200 người nói được cùng ngôn ngữ. Giải Giả sử tồn tại một người nói được cùng ngôn ngữ với tất cả những người còn lại, do 1984 200 5  nên có 200 người có thể nói cùng ngôn ngữ. CuuDuongThanCong.com https://fb.com/tailieudientucntt
  • 30. --------------------------------------------------------------------------------------------------------------------------- 30 Ngược lại thì có 2 người 1 2,P P không có cùng ngôn ngữ. Xét 1983 người còn lại, có ít nhất 992 người có thể nói chuyện với 1 trong 2 người, giả sử là 1P . Xét 992 người này, có ít nhất 992 1 199 5       người nói cùng ngôn ngữ. Suy ra có ít nhất 200 người (199 người trên và 1P ) nói cùng ngôn ngữ, có điều chứng minh. Bài 24. Trong một hội thảo khoa học tất cả các đại biểu tham dự biết tổng cộng 2n ngôn ngữ 2n  . Mỗi người biết đúng 2 ngôn ngữ và bất cứ hai người nào cũng biết chung nhiều nhất một ngôn ngữ. Biết rằng với một số nguyên k thỏa mãn 1 1k n   đều có không quá 1k  ngôn ngữ mà mỗi ngôn ngữ này có không quá k người biết. Chứng minh rằng ta có thể chọn ra một nhóm 2n đại biểu biết tổng cộng 2n ngôn ngữ và mỗi ngôn ngữ có đúng 2 đại biểu trong nhóm biết. Giải Xét graph G đỉnh biểu diễn cho “ngôn ngữ”, cạnh nối hai đỉnh biểu diễn “người biết hai ngôn ngữ đó”. Vậy G là đồ thị 2n đỉnh. Điều kiện “hai người biết chung nhiều nhất một ngôn ngữ” suy ra G là đơn. Điều kiện còn lại cho biết: với mỗi k nguyên 1 1k n   có không quá 1k  ngôn ngữ mà mỗi ngôn ngữ này có không quá k người biết suy ra mỗi đỉnh có bậc nhỏ hơn hoặc bằng k (*). Ta cần chứng minh trong G tồn tại chu trình Hamilton. Ta chứng minh điều này bằng phản chứng. Giả sử trong G không có chung trình H. Khi đó tập các đỉnh không kề nhau của G là không rỗng và hữu hạn. Bằng cách thêm dần hai cạnh nối hai đỉnh không kề nhau ta sẽ xây dựng đồ thị 2n đỉnh G thỏa mãn 1) (*), 2) trong G không có chu trình H 3) Khi thêm cạnh nối hai đỉnh bất kì không kề nhau của G ta sẽ nhận được đồ thị có chu trình H. Xét G với v là đỉnh của G kí hiệu  f v là bậc của v. a) Từ 2) và 3) suy ra giữa hai đỉnh bất kì không kề nhau của G đều tồn tại một đường đi nhận hai đỉnh ấy làm hai đầu mút, đi qua tất cả các đỉnh của G và có độ dài 2 1n  CuuDuongThanCong.com https://fb.com/tailieudientucntt
  • 31. --------------------------------------------------------------------------------------------------------------------------- 31 b) Nếu hai đỉnh v và v’ của G có    , 'f v n f v n  thì v và v’ phải kề nhau. Thật vậy, giả sử v và v’ không kề nhau thì có đường đi 1 2 2 1 2, ,..., ( , 'n nv v v v v v v  đi qua tất cả các đỉnh của G và có độ dài 2 1n  . Giả sử  f v s n  . Kí hiệu 1 2 1 2, ,..., (2 ... 2 )si i i sv v v i i i n     là các đỉnh kề với 1v v . Khi đó với mỗi 1,2,...,j s các đỉnh ( ) 1jiv  không kề với 2 'nv v vì nếu ngược lại thì chu trình H trong G là 1 2 ( ) 1 2 2 1.... ...j ji n n iv v v v v v  mâu thuẫn với 2). Từ đó suy ra  ' 2 ( 1) 1f v n s n     (do s n ), mâu thuẫn với  'f v n . Vậy v, v’ phải kều nhau. c) Từ b) suy ra tập v gồm các đỉnh v của G mà   1f v n  là không rỗng, vậy có  max 1v V f v m n    . Lấy 1v mà  1f v m . Điều kiện (*) với 1k n  nói rằng có ít nhất  2 1 1 2n n n     đỉnh có bậc n , do với 1k n  nói rằng có ít nhất một trong các đỉnh này, chẳng hạn 2nv , không kề với 1v . Suy ra có đường đi 1 2 2, ,..., nv v v đi qua tất cả các đỉnh của G và có độ dài 2 1n  . Kí hiệu 1 2 1 2, ,..., (2 ... 2 )mi i i mv v v i i i n     là các đỉnh không kề với 1v thì lập luận như ở b) chứng tỏ với mọi 1j n  ta có ( ) 1jiv  không kề với 2nv (chú ý rằng điều kiện (*) với k=1) chứng tỏ mọi đỉnh của G có bậc 2 . Áp dụng điều kiện (*) với k=m  2 1m n   suy ra       1 21 1 1 , ,..., mi i i v v v   phải có ít nhất một đỉnh qv có   1qf v m  . Từ định nghĩa của m suy ra  qf v n như vậy 2,q nv v có  qf v n ,  2nf v n mà không kề nhau, mâu thuẫn với b). Mâu thuẫn này cho ta điều phải chứng minh. Bài 25. Cho số nguyên dương 2n  và 1 cây có n đỉnh. Tại mỗi đỉnh của cây đã cho ghi 1 số thực và trên mỗi cạnh ghi số thực bằng tích của 2 số đã ghi ở 2 đầu mút của cạnh đó. Kí hiệu S là tổng của các số ghi trên các cạnh và 1 2, ,..., nx x x là các số ghi trên các đỉnh. Chứng minh rằng  2 1 1. 2 * n i i n x S    . Giải Chú ý rằng 1 cây n đỉnh luôn có 1n  cạnh và có ít nhất 1 lá (điểm treo). CuuDuongThanCong.com https://fb.com/tailieudientucntt
  • 32. --------------------------------------------------------------------------------------------------------------------------- 32 Ta đi tìm giá trị lớn nhất có thể của S và chứng minh giá trị này thỏa mãn bất đẳng thức  * . Giả sử 0, 1,ix i n   (nếu có số âm thì làm S giảm mà vế trái của (*) không thay đổi. Đặt  1 max : 1,2,...,ix x i n  . Giả sử cây ban đầu là 0C , ta xây dựng một cây mà S lớn nhất. Ta coi đỉnh tx là đỉnh được viết số tx . Xét đỉnh 1x và jx là 1 lá của cây suy ra tồn tại ix kề với jx . Nếu ix khác 1x và không kề với 1x thì bỏ cạnh i jx x và nối cạnh 1 jx x Khi đó ta được cây mới 1C mà    0 1S C S C , vậy sau mỗi quá trình biến đổi thì giá trị của S không giảm và nó sẽ đạt giá trị lớn nhất khi không thể biến đổi cây. Tiếp tục quá trình với các lá còn lại của cây đến khi tất cả lá đều kề với 1x , ta nhận thấy tất cả các đỉnh 2 3, ,..., nx x x đều kề với 1x . Thật vậy, nếu có đỉnh kx không kề với 1x , xét đường đi từ 1x đến kx và tiếp tục đường đi cho đến khi gặp 1 là mx , khi đó ta có chu trình (do mx đang kề với 1x ), mâu thuẫn. Khi đó giá trị lớn nhất của S là max 1 2 1 3 1... nS x x x x x x    . Ta có 2 2 2 1 1 2 1. 1 1 n n i i i i n x n x n x         2 2 1 1 max 2 2 1 2 2 2 1 n n i i i i x n x x x S S n               Ta có điều chứng minh. xi xj x1 CuuDuongThanCong.com https://fb.com/tailieudientucntt
  • 33. --------------------------------------------------------------------------------------------------------------------------- 33 Bài 26. Trong không gian cho n điểm ( 2n  ) mà không có bốn điểm nào đồng phẳng và cho  21 3 4 2 n n  đoạn thẳng mà tất cả các đầu mút của chúng nằm trong số n điểm đã cho. Biết rằng có ít nhất một đoạn thẳng mà sau khi bỏ nó đi (giữ nguyên các đầu mút) thì sẽ tồn tại hai điểm phân biệt mà không phải là hai đầu mút của một đường gấp khúc nào. Hãy tìm số k lớn nhất sao cho có k đoạn thẳng tạo thành đường gấp khúc khép kín mà mỗi đỉnh của nó là mút của đúng hai đoạn thẳng thuộc đường gấp khúc đó. Giải Xét graph G có tập đỉnh là tập gồm n điểm đã cho và tập cạnh là tập gồm  21 3 4 2 n n  đoạn thẳng đã cho. Từ giả thiết của bài toán ta thấy trong G tồn tại một cạnh mà sau khi bỏ nó đi thì được G’ không liên thông. Giả sử a và b là hai đỉnh không liên thông với nhau trong G’. Gọi aV và bV lần lượt là tập gồm tất cả các đỉnh của G’ mà liên thông với a và b. Giả sử 1aV n và 2bV n . Dễ thấy G’ có  21 3 4 2 n n  cạnh; 1 2 1 21, 1,n n n n n    và         2 1 1 2 2 1 2 1 2 1 1 1 1 3 4 1 1 1 2 2 2 2 n n n n n n n n n n n n            hay      1 2 1 2 1 21 1 1 0n n n n n n n        . Do đó       1 2 1 2 1 2 1 1 0 1 0 n n n n n n n          Vậy 1 21, 1n n n   hoặc 2 1n n  , 1 1n  . Từ đó suy ra G’ có một đỉnh cô lập và  1n  đỉnh mà bậc của mỗi đỉnh bằng 2n  . Do đó G có một đỉnh bậc 1,  2n  đỉnh mà bậc của mỗi đỉnh bằng 2n  và một đỉnh có bậc bằng 1n  . Bởi thế chu trình đơn có độ dài lớn nhất trong G là chu trình đơn độ dài 1n  nếu 4n  , 0 nếu 2n  hoặc 3n  CuuDuongThanCong.com https://fb.com/tailieudientucntt
  • 34. --------------------------------------------------------------------------------------------------------------------------- 34 Vậy  max 1 ( 4) 0 2, 3 n n k n n       . Bài 27. Ở một nước có 25 thành phố. Hãy xác định số k bé nhất sao cho có thể thiết lập các đường bay (dùng cho cả đi lẫn về) giữa các thành phố để hai điều kiện sau được đồng thời thỏa mãn 1. Từ mỗi thành phố có đường bay trực tiếp đến đúng k thành phố khác 2. Nếu giữa hai thành phố không có đường bay trực tiếp thì tồn tại ít nhất một thành phố có đường bay trực tiếp đến hai thành phố đó. Giải Giả sử k là số sao cho có thể thiết lập được hệ thống đường bay thỏa mãn các điều kiện của đề bài. Khi đó, tổng số đường bay trực tiếp giữa hai thành phố sẽ là 25 2 k . Suy ra  0 mod2k  Xét một thành phố A bất kỳ. Theo giả thiết, từ A có đường bay trực tiếp đến k thành phố khác, gọi là 1 2, ,..., kA A A . Mỗi thành phố , 1,iA i k , lại có đường bay trực tiếp đến k – 1 thành phố khác, (không kể A). Hơn nữa, ta lại có: Nếu từ B đến A không có đường bay trực tiếp thì B phải có đường bay trực tiếp đến ít nhất một thành phố iA . Từ những lập luận trên suy ra, số thành phố chỉ có thể tối đa là   2 1 k k k –1 1k    . Như vậy 2 25 1k  . Kết hợp với  0 mod2k  , suy ra 6k  . Với 6k  ta sẽ chỉ ra cách thiếp lập hệ thống đường bay thỏa mãn các điều kiện cỉa đề bài. Chia 25 thành phố thành năm nhóm, mỗi nhóm gồm năm thành phố. Các thành phố của nhóm thứ , 1,5i i  , ta kí hiệu bởi           1 2 3 4 5, , , , i i i i i A A A A A . Với các thành phố trong cùng nhóm i , ta thiết lập các đường bay                     1 2 2 3 3 4 4 5 5 1, , , , i i i i i i i i i i A A A A A A A A A A . Giữa các thành phố thuộc hai nhóm ,i j bất kỳ,  1,2,3,4,5i j  , xây dựng các đường bay sau                     1 1 2 4 3 2 4 5 5 3, , , , i j i j i j i j i j A A A A A A A A A A . Bằng cách xây dựng các đường bay như trên, ta có: Từ thành phố A bất kỳ sẽ có đường bay trực tiếp đến đúng 2 thành phố, trong cùng nhóm với A và có đường bay trực tiếp đến đúng 4 thành phố khác nhóm với A. Do vậy từ mỗi thành phố sẽ có đường bay trực tiếp đến đúng 6 thành phố khác. CuuDuongThanCong.com https://fb.com/tailieudientucntt
  • 35. --------------------------------------------------------------------------------------------------------------------------- 35 Hơn nữa, với A, B là hai thành phố bất kỳ mà giữa chúng không có đường bay trực tiếp ta thấy: - Nếu A, B cùng thuộc nhóm thì dễ thấy luôn tồn tại 1 thành phố trong nhóm đó mà từ C có đường bay trực tiếp đến cả A và B. - Nếu A, B không cùng nhóm thì qua hình vẽ trên dễ dàng kiểm tra được sự tồn tại của thành phố C mà từ C có đường bay trực tiếp đến cả A và B. Vậy min 6k  . Bài 28. Trong một cuộc hội thảo có  10n n  người tham dự. Biết rằng: 1. Mỗi người quen với ít nhất 2 3 n      người tham dự; 2. Hai người bất kỳ A và B nếu không quen nhau thì quen nhau gián tiếp nghĩa là có  1k k  người 1 2, ,..., kA A A sao cho A quen 1, iA A quen  1, 1,2,..., 1iA i k   và kA quen B; 3. Không thể xếp n người thành một hàng ngang sao cho hai người cạnh nhau bất kỳ đều quen nhau. Chứng minh rằng có thể chia n người thành hai nhóm: nhóm thứ nhất xếp được quanh một bàn tròn sao cho hai người cạnh nhau bất kỳ đều quen nhau, còn nhóm thứ hai gồm người đôi một không quen nhau. Giải Chuyển bài toán sang ngôn ngữ Graph, trong đó mỗi người coi là một điểm trên mặt phẳng, còn quan hệ quen nhau coi là một cạnh (1 đoạn thẳng với giả thiết rằng các đoạn thẳng này không cắt nhau trừ hai điểm đầu mút), ta có graph G đơn, vô hướng với tập đỉnh gồm n điểm  1 2, ,..., np A A A và bậc của đỉnh A bất kỳ là   2 3 n d A       Điều kiện “Hai người bất kỳ quen nhau hoặc quen nhau gián tiếp chứng tỏ Graph G là liên thông. Trong G (hữu hạn) xét đường gấp khúc nhiều cạnh nhất Po, giả sử Po có k đỉnh là  0 1 2, ,..., kP A A A với 1( 1,2,..., 1)i iA A i k   là các cạnh ( iA kề với 1iA ) CuuDuongThanCong.com https://fb.com/tailieudientucntt
  • 36. --------------------------------------------------------------------------------------------------------------------------- 36 Do điều kiện (3) thì 1k n  Gọi N(A) là tập các đỉnh kề với đỉnh A. Ta có    1 2,..., kN A A A và    2 1,..., kN Ak A A  Vì trái lại thì tồn tại đường gấp khúc khác có nhiều cạnh hơn Po. Giả sử      1 2 , ,..., , 1,2,...,si i i iN A A A A i n  ký hiệu        1 2 1 21 1 1 1 1 1, ,..., , , ,...,s si i i i i i i iN A A A A N A A A A          Do 1k n  nên tồn tại đỉnh 0B P . Ta có    kN B N A     Thật vậy nếu    j kA N B N A     thì tồn tại đường gấp khúc  1 1 1 1,..., , , ,..., , ,j k k j jA A A A A A B   có k+1 cạnh, trái giả thiết đối với 0P . Lập luận tương tự có    1N B N A     . Ta cũng có    1 kN A N A     vì nếu trái lại thì            1 1 2 2 3 3 1 1 3 3 k kN B N A N A N B B A N A n n n                        Suy ra số đỉnh của tập hợp này lớn hơn hoặc bằng n mà tập hợp đó không chứa đỉnh B. Mâu thuẫn. Vậy    1i kA N A N A     Khi đó tồn tại đường gấp khúc khép kín có k – 1 đỉnh thuộc tập  c iP A là  1 2 1 1 1, ,..., , , ,...,i k k iA A A A A A   Tập còn lại chứa các đỉnh đôi một không kề nhau (không có đoạn thẳng nối chung) vì nếu trái lại, chẳng hạn có  1 2 0, iB B P A mà 1B kề với 2B do tính liên thông tồn tại đường gấp khúc chứa 1B , 2B và  0 iP A có nhiều cạnh hơn 0P mâu thuẫn. CuuDuongThanCong.com https://fb.com/tailieudientucntt
  • 37. --------------------------------------------------------------------------------------------------------------------------- 37 Bài 29. Cho n là số nguyên dương sao cho tồn tại đồ thị đơn G, vô hướng có n đỉnh, bậc mỗi đỉnh nhỏ hơn 1n  và thỏa mãn: 2 đỉnh bất kì là 2 đầu mút của duy nhất 1 đường đi đơn có độ dài không vượt quá 2. Chứng minh 1n  là số chính phương. Giải Xét đồ thị  ,G V E với  1 2, ,..., nV A A A , đặt  i id A d , suy ra 1 2 n i i d E    . Số đường đi có độ dài 1 bằng số cạnh. Số đường đi có độ dài 2 bằng  2 1 1 1 . 2i n n i i d i i d d C      Do chọn 2 điểm bất kì thì có đúng 1 đường đi có độ dài không quá 2 nên tổng số đường đi độ dài 1 và đường đi độ dài 2 bằng số cặp 2 điểm, ta có:      2 2 1 1 1 1 1 1 2 2 2 n n n i ii n i i i i d d n nd C d n n              Ta chứng minh bậc của 2 đỉnh bất kì a, b bằng nhau. Xét đỉnh x có bậc nhỏ nhất, giả sử là t. Tập  1 2, ,...,x tS x x x chứa các đỉnh kề với x. Với mỗi 1,2,...,i t , đặt iA là tập các đỉnh khác x và kề với ix . Nhận thấy các tập iA là rời nhau (nếu có y thuộc ,i jA A thì có 2 đường đi  , ,ix x y và  , ,jx x y cùng có độ dài 2 nối x và y, Hợp của tất cả các tập này cùng với x, xS là tập tất cả các đỉnh, Hơn nữa, mỗi phần tử trong tập iA kề với nhiều nhất 1 đỉnh trong tập jA , Hai phần tử trong cùng tập jA thì không kề nhau. Suy ra bậc của mỗi phần tử trong  1,2,...,iA i t không vượt quá t, do tính nhỏ nhất nên bậc của tất cả các đỉnh này là t. CuuDuongThanCong.com https://fb.com/tailieudientucntt
  • 38. --------------------------------------------------------------------------------------------------------------------------- 38 Suy ra 1 2 ... nd d d   hay   2 2 1 11 . 1n n n d n d     là số chính phương. Ta có điều chứng minh. Bài 30. Giả sử có 2017 thành phố mà từ mỗi thành phố đều có đường bay trực tiếp đến ít nhất 93 thành phố khác và từ 1 thành phố có thể đi đến thành phố bất kì qua một số đường bay liên tiếp (một đường bay là đường nối trực tiếp 2 thành phố). Chứng minh có thể đi từ 1 thành phố tùy ý đến 1 thành phố khác với số đường bay không vượt qua 63. Giải Xây dựng mô hình graph: Xét graph G có 2017 đỉnh tương ứng với 2017 thành phố, 2 đỉnh được nối với nhau nếu 2 thành phố tương ứng có đường bay trực tiếp. Suy ra bậc của mỗi đỉnh không nhỏ hơn 93 và G là liên thông. Ta cần chứng minh giữa 2 đỉnh bất kì có đường đi mà độ dài không vượt quá 63. Giả sử phản chứng, tồn tại 2 đỉnh X, Y mà mọi đường đi nối chúng đều có độ dài lớn hơn 63. Xét đường đi  0 1 0... , , 64n nA A A X A Y A n   ngắn nhất nối X và Y. Xét các đỉnh dạng  3 , 0,1,2,...,21iA i  với 3iV là tập các đỉnh kề với 3iA , suy ra 3 93, 0,21iV i   . Ta chứng minh 3 3 ,i jV V i j     . Thật vậy, giả sử tồn tại i j mà 3 3i jV V  , suy ra tồn tại đỉnh T kề với cả 3iA và 3 jA , khi đó có đường đi 0 1 3 3... ...i j nA A A TA A nối X và Y, nhưng đường đi này có độ dài ngắn hơn đường đi 0 1... nA A A , suy ra mâu thuẫn. Suy ra G phải có ít nhất 22.93 2046 đỉnh, mâu thuẫn. Vậy giả sử sai, ta có điều chứng minh. Bài 31. Trong một hội thảo khoa học, mỗi người quen với ít nhất 3 người khác. Chứng minh ta có thể chọn ra từ những người này một nhóm người thỏa mãn đồng thời 2 điều kiện: 1. Số người của nhóm không chia hết cho 3; CuuDuongThanCong.com https://fb.com/tailieudientucntt
  • 39. --------------------------------------------------------------------------------------------------------------------------- 39 2. Có thể xếp tất cả những người này quanh bàn tròn mà mỗi người ngồi giữa 2 người quen. Giải Xét graph G với đỉnh tương ứng với những người trong cuộc hội thảo, 2 đỉnh được nối nếu 2 người tương ứng quen nhau. Khi đó mỗi đỉnh có bậc không nhỏ hơn 3. Cần chứng minh G có chu trình độ dài không chia hết cho 3. Xây dựng đường đi từ đỉnh 1A bất kì của G: Tồn tại 2A kề 1A , 3 1A A kề với 2A , 4 2A A kề với 3A . + Nếu 4A kề với cả 1 2,A A thì dừng lại; + Nếu 4A không kề với 1 trong 2 đỉnh 1 2,A A thì tiếp tục mở rộng tới các đỉnh tiếp theo đến khi gặp đỉnh nA kề với 3 đỉnh trên đường đi đang xét, giả sử nA kề với  1, , 1 2n i jA A A i j n     . Khi đó ta có đường đi dạng như sau Với cách xây dựng đường đi này, ta có 3 chu trình 1...i i j n iA A A A A ; 1 1... ...i i j n n iA A A A A A  và 1 1...j j n n jA A A A A  Giả sử cả 3 chu trình trên đều có độ dài chia hết cho 3 thì ta có hệ điều kiện               2 3 2 3 1 2 3 3 2 3 1 2 3 1 2 3 j i j i j i n j j i n j n j                                 , vô lí. Vậy giả sử sai, ta có điều chứng minh. ... ... ... A1 A2 Ai Aj An-1 An CuuDuongThanCong.com https://fb.com/tailieudientucntt
  • 40. --------------------------------------------------------------------------------------------------------------------------- 40 Bài tập tham khảo Bài 1. Giả sử có 2014 thành phố mà từ mỗi thành phố đều có đường bay trực tiếp đến đúng 3 thành phố khác đồng thời có thể đi từ 1 thành phố đến thành phố bất kì bằng các đường bay liên tiếp. Chứng minh tồn tại 202 thành phố đôi một không có đường bay trực tiếp đến nhau sao cho nếu các sân bay tại các thành phố này ngừng hoạt động thì người ta vẫn có thể di chuyển giữa các thành phố còn lại bằng máy bay. Bài 2. Ở một quốc gia có 5 hãng hàng không hoạt động. Biết rằng từ mỗi thành phố có đúng 5 đường bay trực tiếp đến 5 thành phố khác và mỗi 2 đường bay khác nhau thuộc 2 hãng hàng không khác nhau. Người ta có thể đi từ 1 thành phố đến thành phố khác bằng máy bay. Chứng minh khi hủy 4 đường bay tùy ý mà không có 2 đường bay thuộc cùng một hãng hàng không thì người ta vẫn có thể đi lại giữa các thành phố bằng máy bay. Bài 3. Trong 1 cuộc thi có 25 bạn nam và 1 số bạn nữ. Biết rằng với mỗi nhóm m bạn nam  10m  đều có đúng 1m  bạn nữ mà mỗi bạn nữ quen ít nhất 1 bạn nam trong nhóm này. Chứng minh tồn tại 1 bạn nữ quen với ít nhất 16 bạn nam trong cuộc thi. Bài 4. Cho số nguyên dương 3n  và đồ thị đơn vô hướng có n đỉnh thỏa mãn điều kiện: Với mỗi 2 k n  , số cạnh của 1 đồ thị con bất kì k đỉnh của G không vượt quá 2 2k  . Chứng minh có thể tô màu tất cả các cạnh, mỗi cạnh bởi 1 trong 2 màu xanh đỏ mà không tồn tại chu trình mà các cạnh được tô cùng màu. Bài 5 (IMO Shortlist 2002, C7). Trong nhóm có 120 người, một số người là bạn của nhau. Một nhóm được gọi là bộ bốn yếu là nhóm có 4 người chứa đúng 1 cặp là bạn của nhau. Tìm giá trị lớn nhất số bộ bốn yếu. Bài 6. Cho số nguyên 3n  . Trong không gian cho n điểm mà không có 3 điểm nào thẳng hàng và không có 4 điểm nào đồng phẳng. Tại mỗi điểm ghi 1 số nguyên dương sao cho không có 2 điểm nào được ghi bởi cùng 1 số và n số được ghi là n số nguyên dương đầu tiên. Gọi điểm được ghi số i là điểm i. Nối tất cả các cặp điểm  ,p q với p q bởi 1 mũi tên từ p đến q. Tô mỗi mũi tên bởi 1 trong 2 màu xanh, đỏ. Một cách tô màu được gọi là “tốt” nếu tồn tại 2 điểm j, k sao cho có thể đi từ j đến k qua các mũi tên xanh và cũng có thể đi từ j đến k qua các mũi tên đỏ. Hỏi có tất cả bao nhiêu cách tô màu “tốt”. CuuDuongThanCong.com https://fb.com/tailieudientucntt
  • 41. --------------------------------------------------------------------------------------------------------------------------- 41 Bài 7 (Poland 2000). Cho số nguyên dương 2n  . Tìm số k nhỏ nhất có tính chất: với bất kì tập k ô đơn vị của bảng n n luôn tồn tại 1 tập con khác rỗng A sao cho trên mỗi dòng, trên mỗi cột có chẵn ô đơn vị trong A. Bài 8 (Thụy Điển 2010). Một thành phố có 3n công dân. Hai người bất kì trong thành phố có ít nhất 1 người bạn chung trong thành phố này. Chứng minh có thể chọn một nhóm có n người sao cho bất kì người nào trong số 2n người còn lại có người quen trong nhóm n người. Bài 9 (Generalization of USAMO 2007). Xét graph liên thông G với V đỉnh, mỗi đỉnh có bậc nhiều nhất là d. Chứng minh G có thể phân hoạch thành 2 graph con liên thông, mỗi graph có ít nhất 1V d  đỉnh. Bài 10 (Hefetz, Krivelevich, Stojakovic, Szabo). Xét số nguyên dương d và graph G mà mỗi đỉnh đều có bậc d. Maker và Breaker chơi 1 trò chơi với cạnh của G. Hai người lần lượt, mỗi bước chọn đúng 1 cạnh của graph và dời cạnh này, Breaker là người đi đầu tiên. Trò chơi kết thúc khi tất cả các cạnh được chọn. Maker thắng nếu sau khi kết thúc trò chơi, với mỗi đỉnh anh ấy chọn được ít nhất 4 d     cạnh chứa đỉnh này, ngược lại anh ấy thua cuộc. Chứng minh Maker có chiến thuật thắng. Bài 11. Người ta muốn mời một số em học sinh tới dự một buổi gặp mặt, mà trong số đó mỗi em chưa quen với ít nhất là 56 em khác và với mỗi cặp hai em chưa quen nhau thì đều có ít nhất một em quen với cả hai em đó. Hỏi số học sinh được mời dự buổi gặp mặt nói trên có thể là 65 em được hay không? Bài 12 (IMOSL 2015, C7). Trong một công ty có một số người là đối thủ của nhau. Một nhóm người được gọi là khó gần nếu số người trong nhóm này là lẻ và nhóm có ít nhất 3 người, hơn nữa có thể xếp những người trong nhóm quanh bàn tròn sao cho hai người kề nhau là đối thủ của nhau. Biết rằng có nhiều nhất 2015 nhóm khó gần, chứng minh có thể chia công ty này thành 11 nhóm sao cho không có 2 đối thủ trong cùng 1 nhóm. Bài 13 (China TST 2011). Xét graph G có  2 3 1n n  đỉnh và không có đỉnh nào có bậc lớn hơn 4n. Giả sử tồn tại 1 đỉnh bậc 1 và với 2 đỉnh bất kì luôn tồn CuuDuongThanCong.com https://fb.com/tailieudientucntt
  • 42. --------------------------------------------------------------------------------------------------------------------------- 42 tại đường đi có độ dài không vượt quá 3 nối chúng. Chứng minh G có ít nhất 2 7 3 2 n n cạnh. Bài 14 (IMO Shortlist 2013, C6). Trong graph G, mỗi đỉnh v có nhiều nhất 2k đỉnh có khoảng cách 3 đến nó. Chứng minh với bất kì đỉnh u, tồn tại nhiều nhất  1k k  đỉnh mà khoảng cách đến nó là 4. Bài 15 (IMO Shortlist 2004, C3). Xét số nguyên 4n  và 1 graph đầy đủ có n đỉnh. Thực hiện công việc như sau: mỗi bước chọn bất kì chu trình có độ dài 4 (nếu tồn tại), chọn cạnh tùy ý trong chu trình này và xóa cạnh này. Quá trình dừng lại khi không còn chu trình 4. Tìm số cạnh nhỏ nhất có thể bị xóa. CuuDuongThanCong.com https://fb.com/tailieudientucntt
  • 43. --------------------------------------------------------------------------------------------------------------------------- 43 Graph hai phần Định nghĩa : Graph  ,G V E được gọi là 2 phần nếu tập đỉnh có thể phân hoạch thành 2 tập con khác rỗng A, B sao cho không có cạnh nối 2 đỉnh trong cùng 1 tập. Định lý : Một graph là hai phần nếu và chỉ nếu nó không có chu trình lẻ. Chứng minh Giả sử  , ,G V E V A B  là 1 graph 2 phần. Nếu có chu trình thì độ dài của chu trình phải là chẵn. Thật vậy, giả sử chu trình 1 2 1... kv v v v thì nếu có 1 cạnh nối 1 đỉnh trong A với 1 đỉnh trong B thì ngay sau đó có 1 cạnh nối 1 đỉnh trong B với 1 đỉnh trong A. Giả sử G không có chu trình độ dài lẻ. Ta có thể giả sử G là liên thông, vì nếu G không liên thông thì nó là hợp của các thành phần liên thông rời nhau, khi đó ta thực hiện với từng thành phần liên thông và ghép chúng lại. Gọi A là 1 đỉnh bất kì của G. Xét 2 tập X, Y tương ứng chứa các đỉnh mà độ dài tới A là chẵn, lẻ tương ứng. Suy ra X, Y chính là 1 phân hoạch của tập đỉnh V. Ta chứng minh mỗi tập X, Y các đỉnh là độc lập. Giả sử có 2 đỉnh B và C trong X được nối với nhau, ta xét đường đi ngắn nhất nối A, B sau đó nối B, C, cuối cùng là đường đi ngắn nhất nối C, A (có thể các đường đi có đỉnh lặp hoặc cạnh trùng thì ta sẽ loại bỏ đỉnh, cạnh đó). Khi đó có chu trình có độ dài lẻ, mâu thuẫn. Tương tự với tập Y. Vậy ta có điều chứng minh. Bài 1 (ShortlistIMO 1983 C1). Một đất nước có 1983 thành phố, mỗi cặp 2 thành phố có một đường đi nối chúng. Mỗi đường đi thuộc quản lí của đúng 1 trong 10 công ty. Chứng minh tồn tại một tour du lịch theo vòng khép kín qua lẻ con đường mà các con đường này thuộc quyền quản lí của một công ty. Giải Xét graph G có 1983 đỉnh ứng với 1983 thành phố, 2 đỉnh được nối với nhau và tô cạnh đó bởi 1 trong 10 màu 1, 2, …, 10 ứng với 1 trong 10 hãng hàng không. CuuDuongThanCong.com https://fb.com/tailieudientucntt
  • 44. --------------------------------------------------------------------------------------------------------------------------- 44 Giả sử phản chứng không có một tour du lịch theo vòng khép kín qua lẻ con đường mà các con đường này thuộc quyền quản lí của một công ty, nghĩa là graph G không có chu trình lẻ khi xét các đỉnh và các cạnh cùng màu. Suy ra khi xét các đỉnh và các cạnh cùng màu thì G là 2 phần. Xét các đỉnh và các cạnh được tô màu 1 thì G là graph 2 phần, có 1 phần có ít nhất 1983 1 992 2       đỉnh, đặt đó là 1G . Trong 1G chỉ xét các cạnh được tô màu 2, đây là graph 2 phần, có 1 phần có ít nhất 992 496 2      đỉnh, đặt là 2G . Tiếp tục quá trình có các graph 3G có ít nhất 496 248 2      đỉnh, graph 4G có ít nhất 248 124 2      đỉnh, graph 5G có ít nhất 124 62 2      đỉnh, graph 6G có ít nhất 62 31 2      đỉnh, graph 7G có ít nhất 31 1 16 2       đỉnh, graph 8G có ít nhất 16 8 2      đỉnh, graph 9G có ít nhất 4 đỉnh, graph 10G có ít nhất 2 đỉnh. Cần chú ý rằng, các đỉnh trong 10G không được nối với nhau bởi bất kì cạnh nào vì nếu nối bởi 1 cạnh thì cạnh này được tô màu 1 màu i, khi đó trái với 1iG  là 2 phần khi ta xét các đỉnh trong 1iG  và các cạnh được tô màu i. Bài 2 (IMC 1999). Trong mặt phẳng tọa độ cho n đường thẳng song song với Ox (ta gọi là các dòng) và n đường thẳng song song với Oy (ta gọi là các cột). Xét 2n giao điểm được tạo ra bởi các đường thẳng trên. Chứng minh tồn tại  2 2k k n  điểm 1 2 2, ,..., ka a a trong 2n điểm trên thỏa mãn với mỗi 1,2,..., 1i k  : 2 1ia  và 2ia thuộc cùng 1 dòng nhưng 2ia và 2 1ia  thuộc cùng 1 cột. Giải Xét graph 2 phần G A B  , với các đỉnh trong A tương ứng các dòng và các đỉnh trong B tương ứng các cột. Cặp điểm được nối với nhau nếu có 1 điểm được xét là giao của dòng và cột tương ứng. Nếu 2 cạnh có cùng 1 đầu mút thì 2 CuuDuongThanCong.com https://fb.com/tailieudientucntt